"Psychiatric/Mental Health Nursing - Psychobiological Disorders + Foundations"

Pataasin ang iyong marka sa homework at exams ngayon gamit ang Quizwiz!

Arrives early and waits quietly to be called for the tests

A client is scheduled for several diagnostic studies. Which behavior best indicates to the nurse that the client has received adequate preparation?

Do not want to repeat the ritual but feel compelled to do so

A nurse has been assigned to care for a client with the diagnosis of obsessive-compulsive disorder (OCD). Before providing care for this client, the nurse should consider that clients with OCD:

Checking the client's blood pressure

A nurse is aware that after the administration of alprazolam (Xanax) is started, it is important to observe the client for side effects. What is the nurse's initial action?

Having a coherent sense of self and plans for self-actualization

A nurse is caring for a 20-year-old client. According to Erikson's developmental psychosocial theory, what is expected by 20 years of age?

3.Having a coherent sense of self and plans for self-actualization

A nurse is caring for a 20-year-old client. According to Erikson's developmental psychosocial theory, what is expected by 20 years of age? 1.Having the capacity for love and a commitment to work 2.Being creative and productive and having concern for others 3.Having a coherent sense of self and plans for self-actualization 4.Accepting the worth, integrity, and uniqueness of one's past and present life

Imitating and participating in the child's activities

A nurse is caring for a child with autism. Which intervention is most appropriate in an attempt to promote socialization for this child?

Constant one-to-one supervision

A nurse is caring for a client admitted to a mental health unit because of suicidal ideation. Which intervention provides the greatest safety for this client?

Seizures

A nurse is caring for a client who has abruptly stopped taking a barbiturate. What should the nurse anticipate that the client may experience?

A defense against anxiety

A nurse is caring for a client who has been experiencing delusions. According to psychodynamic theory, delusions are:

Maintaining a calm, consistent approach with the client

A nurse is caring for a client who is angry and agitated. What is the best approach for the nurse to use with this client?

Sympathetic nervous system

A nurse is caring for a client who is experiencing a crisis. Which nervous system is primarily responsible for the clinical manifestations that the nurse is likely to identify?

3. Sympathetic nervous system **The sympathetic nervous system reacts to stress by releasing epinephrine, which prepares the body to fight or flee by increasing the heart rate, constricting peripheral vessels, and increasing oxygen supply to muscles.

A nurse is caring for a client who is experiencing a crisis. Which nervous system is primarily responsible for the clinical manifestations that the nurse is likely to identify? 1. Central nervous system 2. Peripheral nervous system 3. Sympathetic nervous system 4. Parasympathetic nervous system

Fluvoxamine (Luvox) -Fluvoxamine (Luvox) blocks the uptake of serotonin, which leads to a decrease in obsessive-compulsive behaviors. Benztropine (Cogentin) is an antiparkinsonian agent, not an antianxiety agent. Amantadine is an antiparkinsonian agent, not an antianxiety agent. Diphenhydramine (Benadryl) is an antihistamine, not an antianxiety agent.

A nurse is caring for a client who uses ritualistic behavior. What common antiobsessional medication does the nurse anticipate will be prescribed?

An older adult, accompanied by family members, is admitted to a long-term care facility with symptoms of dementia. During the admission procedure the initial statement by the nurse most helpful to this client is:

"Don't be afraid. I'm your nurse, and everyone here in the hospital is here to help you."

An older client with a diagnosis of dementia is living in a long-term care facility. The client's daughter, who lives 300 miles away, calls the unit to speak to the nurse about her upcoming visit. What should the nurse say in response to her question about the best time of day to visit?

"Around 2:30 in the afternoon is the best time to visit."

1 Calm 2 Matter-of-fact

A nurse is caring for a client with the diagnosis of somatoform disorder, conversion type. What type of affect does the nurse expect this client to exhibit? (Select all that apply.)

To reduce the positive symptoms of psychosis

A nurse is caring for several clients who have severe psychiatric disorders. What is the major reason that a health care provider prescribes an antipsychotic medication for these clients?

The nurse notes that a client has been experiencing a somatic delusion. Which statement led to this conclusion?

"My stomach has disintegrated." A somatic delusion is a false belief that one has a disease or a physical defect. A delusion about being a person of importance is a grandiose delusion. A delusion about death is a nihilistic delusion. A delusion that others are out to cause personal harm is a paranoid delusion.

1.Dissociation rational 1. Talking in the third person reflects poor ego boundaries and a dissociation from the real self.

A nurse greets a client who had been experiencing delusions of persecution and auditory hallucinations by saying, "Good evening. How are you?" The client, who has been referring to himself as "man," answers, "The man is bad." Of what is this an example? 1.Dissociation 2.Transference 3.Displacement 4.Identification

Dissociation -Speaking in the third person reflects poor ego boundaries and dissociation from the real self. Transference is the movement of emotional energy and feelings that one has for one person to another person

A nurse greets a client who has been experiencing delusions of persecution and auditory hallucinations by saying, "Good evening. How are you?" The client, who has been referring to himself as "the man," answers, "The man is bad." Of what is this an example?

Loosened associations and hallucinations

A nurse is caring for an adolescent client with the diagnosis of schizophrenia, undifferentiated type. Which signs and symptoms should the nurse expect the client to experience?

Bargaining

A terminally ill client repeatedly tells the nurse all the details of a daughter's wedding that will take place in 6 months and how important it is for her to attend. What Kübler-Ross stage of grieving does the nurse identify?

4."Let's talk about your feeling regarding your child's diagnosis."

A parent of a 13-year-old adolescent who was recently diagnosed with Hodgkin disease tells a nurse, "I don't want my child to know the diagnosis." How should the nurse respond? 1."It is best if your child knows the diagnosis." 2."Did you know the cure rate for Hodgkin is high?" 3."Would you like someone with Hodgkin to talk with you?" 4."Let's talk about your feeling regarding your child's diagnosis."

A client in the mental health clinic tells the nurse, "The FBI is out to kill me." What should the nurse document that the client is experiencing?

Delusion of persecution

In her eighth month of pregnancy, a 24-year-old client is brought to the hospital by the police, who were called when she barricaded herself in a ladies' restroom of a restaurant. During admission the client shouts, "Don't come near me! My stomach is filled with bombs, and I'll blow up this place if anyone comes near me." The nurse concludes that the client is exhibiting:

Delusional thinking

An adolescent with a conduct disorder is undergoing behavioral therapy in an attempt to limit behaviors that violate societal norms. A specific outcome criterion unique to adolescents with this problem is:

Demonstration of respect for the rights of others

An older adult who lives alone tells a nurse at the community health center, "I really don't need anyone to talk to. The TV is my best friend." The nurse identifies the defense mechanism known as:

Denial

A nurse is planning activities for a withdrawn client who is hallucinating. Which activity will be most therapeutic for the client?

Going for a walk with the nurse

A man with bipolar disorder, manic episode, has been traveling around the country, dating multiple women, and buying his dates expensive gifts. He is admitted to the hospital when he becomes exhausted and runs out of money. The nurse anticipates that during a manic episode the client is most likely experiencing feelings of:

Grandeur

Provide the neutral environment the client needs to work through conflicts

Hospitalization or day-treatment centers are often indicated for the treatment of a client with obsessive-compulsive disorder because these settings:

By limiting unnecessary interactions with the client -Limiting unnecessary interactions will decrease stimulation and therefore agitation

How can a nurse minimize agitation in a disturbed client?

8 to 12 hours

How long after the last dose should the nurse schedule to have a client's blood drawn to evaluate the serum lithium level?

4. 8 to 12 hours

How long after the last dose should the nurse schedule to have a client's blood drawn to evaluate the serum lithium level? 1. 2 to 4 hours 2. 4 to 6 hours 3. 6 to 8 hours 4. 8 to 12 hours

4. 8 to 12 hours

How long after the last dose should the nurse schedule to have a client's blood drawn to evaluate the serum lithium level? 1. 2 to 4 hours 2. 4 to 6 hours 3. 6 to 8 hours 4. 8 to 12 hours

By fostering a sense of contentment when the client looks back on her achievements

How should a nurse at an assisted living facility encourage a client to effectively complete the tasks of older adulthood?

Toilet the client more frequently with supervision

How should a nurse intervene when a confused and anxious client voids on the floor in the sitting room of the mental health unit?

Offering high-calorie snacks frequently that the client can hold

How should the nursing staff fulfill the nutritional needs of a client experiencing periods of extreme mania and hyperactivity?

1.Empathy

In an attempt to remain objective and support a client during a crisis, the nurse uses imagination and determination to project the self into the client's emotions. This technique is known as: 1.Empathy 2.Sympathy 3.Projection 4.Acceptance

Antipsychotics

In conjunction with which classification of medication are trihexyphenidyl, biperiden (Akineton), and benztropine (Cogentin) often prescribed?

3. Antipsychotics

In conjunction with which classification of medication are trihexyphenidyl, biperiden (Akineton), and benztropine (Cogentin) often prescribed? 1. Anxiolytics 2. Barbiturates 3. Antipsychotics 4. Antidepressants

A client with a history of sleeplessness, lack of interest in eating, and excessive purchases on charge accounts is seen in the mental health clinic. The adaptation that the nurse should expect the client to exhibit is:

Intrusive involvement with environmental activities

A nurse is caring for a group of children with the diagnosis of autism. Which signs and symptoms are associated with this disorder? (Select all that apply.)

Repetitive activities Self-injurious behaviors Lack of communication with others

A client who is hallucinating actively approaches the nurse and reports, "I'm hearing voices that are saying bad things about me." What should the nurse do?

Reply, "I'll stay with you for a while because you seem frightened."

A 20-year-old woman is brought to an emergency department after having been raped. She is very anxious and cannot recall any of the circumstances surrounding the assault or provide the police with a description of the rapist. The nurse knows that the defense mechanism being utilized by this woman is:

Repression

Establishing a patent airway

After a cocaine binge an individual is found unconscious and is admitted to the hospital with acute cocaine toxicity. What should the initial nursing action be directed toward?

Focusing

After a traumatic event, a client is extremely upset and exhibits pressured and rambling speech. What therapeutic technique can the nurse use when a client's communication rambles?

Repression

After an automobile collision involving a fatality and a subsequent arrest for speeding, a client has amnesia regarding the events surrounding the accident. Which defense mechanism is being used by the client?

A client who has been hospitalized with schizophrenia tells the nurse, "My heart has stopped and my veins have turned to glass!" What should the nurse conclude that the client is experiencing?

Somatic delusion

An anxious client reports experiencing pain in the abdomen and feeling empty and hollow. A diagnostic workup reveals no physical causes of these clinical findings. What term best reflects what the client is experiencing?

Somatization

Introverted and emotionally withdrawn -These clients usually display social inadequacy and lack of emotional contact with others. Rigid and controlling behaviors reflect an obsessive-compulsive personality disorder. Submissive and immature behaviors reflect a dependent personality disorder. Arrogant and attention-seeking behaviors probably reflect a narcissistic personality disorder.

An adult is found to have schizotypal personality disorder. How should a nurse describe the client's behavior?

When planning nursing care for clients who are grieving the potential death of a family member, it is helpful to draw on the understanding of the five stages of grieving identified and described by Elisabeth Kübler-Ross. Place these stages in order of progression from first to last.

Denial Anger Bargaining Depression Acceptance

The nurse is caring for a client with vascular dementia. What does the nurse identify as the cause of this problem?

Disruptions in cerebral blood flow, resulting in thrombi or emboli

A nurse greets a client who has been experiencing delusions of persecution and auditory hallucinations by saying, "Good evening. How are you?" The client, who has been referring to himself as "the man," answers, "The man is bad." Of what is this an example?

Dissociation

A client who is in a manic phase of bipolar disorder threatens staff and clients on a psychiatric acute care unit. Place the following interventions in priority order, from the least to the most restrictive.

Diversional activities Limit-setting Medication administration Seclusion Restraints

What therapeutic nursing intervention may redirect a hyperactive, manic client?

Encouraging the client to tear pictures out of magazines for a scrapbook

When a client is expressing severe anxiety by sobbing in the fetal position on her bed, the nurse's priority is:

Ensuring a safe therapeutic milieu

1. Industry vs inferiority 2. Identity vs role confusion 3. Intimacy vs isolation 4. Integrity vs despair 5. Generativity vs. stagnation

Erik Erikson posited life as a sequence of achievements. Place the levels of development in their order of achievement according to Erikson's theory.

What should a nurse consider when planning care for a client who is using ritualistic behavior?

Clients do not want to repeat their rituals but feel compelled to do so.

Electroconvulsive therapy (ECT) is a mode of treatment that is used primarily to treat:

Clinical depression

A man is admitted to the psychiatric unit after attempting suicide. The client's history reveals that his first child died of sudden infant death syndrome 2 years ago, that he has been unable to work since the death of the child, and that he has attempted suicide before. When talking with the nurse he says, "I hear my son telling me to come over to the other side." What should the nurse conclude that the client is experiencing?

Command hallucination

A client with a long history of alcohol dependence spends 28 days in an alcohol-rehabilitation unit. What type of referral does the nurse anticipate will be included in the discharge plan?

Community-based self-help group

A client on the psychiatric unit tells the nurse, "I'm a movie star, and the other clients are my audience." What is an appropriate conclusion for the nurse to document about what the client is experiencing?

Delusion of grandeur

An acutely ill client with the diagnosis of schizophrenia has just been admitted to the mental health unit. What is the most therapeutic initial nursing intervention?

Spending time with the client to build trust and demonstrate acceptance

A client with a diagnosis of borderline personality disorder (BPD) has negative feelings toward the other clients on the unit and considers them all "bad." Which defense is the client using when identifying the other clients thusly?

Splitting

A nurse is conducting the Mini-Mental Status examination on an older client. What should the nurse ask the client to do when testing short-term memory?

State three random words mentioned earlier in the exam

A delusional client refuses to eat because she believes that the food is poisoned. What is the most appropriate initial nursing intervention?

Stating that the food is not poisoned

A client believes that doorknobs are contaminated and refuses to touch them except with a paper tissue. What nursing intervention will be most therapeutic for this client?

Supplying the client with tissues to maintain function until the anxiety eases

What is the best nursing intervention during the working phase of the therapeutic relationship with which to meet the needs of individuals who demonstrate obsessive-compulsive behavior?

Supporting rituals while setting realistic limits

Isolation

Survivors of a major earthquake are being interviewed on admission to the hospital. The nurse notes that they exhibit a flattened affect, make minimal eye contact, and speak in a monotone. These behaviors are indicative of the defense mechanism known as:

Which client characteristic is an initial concern for the nurse when caring for a client with the diagnosis of paranoid schizophrenia?

Suspicious feelings

A nurse is caring for a client who is addicted to opioids and who has undergone major surgery. The client is receiving methadone. What is the purpose of this medication?

Switches the user from illicit opioid use to use of a legal drug

A client is responding within an hour of receiving naloxone to combat respiratory depression from an overdose of heroin. Why should a nurse continue to closely monitor this client's status?

Symptoms of the heroin overdose may return after the naloxone is metabolized.

Sitting down in a chair by the client and saying, "I'm here to spend time with you."

What is the most appropriate intervention for the nurse to implement after finding a disturbed client in bed in the fetal position?

1. Sitting down in a chair by the client and saying, "I'm here to spend time with you."

What is the most appropriate intervention for the nurse to implement after finding a disturbed client in bed in the fetal position? 1. Sitting down in a chair by the client and saying, "I'm here to spend time with you." 2. Touching the client gently on the shoulder and saying, "I'm going to sit with you for a while." 3. Going to the client and saying, "I'll be waiting for you in the community room, so please get up and join me." 4. Leaving the client alone because the behavior demonstrates that the client has regressed too far to benefit from talking with the nurse

Escorting the client from the room -Escorting the client from the room accepts the client but rejects the behavior. The nurse should set limits on this behavior when it is not performed in a private area

What is the most appropriate nursing intervention when a client is seen openly masturbating in the recreation room?

1.Realistic limits are set.

What should a nurse ensure when creating an environment that is conducive to psychological safety? 1.Realistic limits are set. 2.Passive acceptance is promoted. 3.The client's physical needs are met. 4.The physical environment is kept in order.

Assisting the individual with plans for the future

What is the most therapeutic nursing intervention to help a late-middle-aged individual cope with the emotional aspects of aging?

Realistic limits are set.

What should a nurse ensure when creating an environment that is conducive to psychological safety?

3.Compensation

When a person who is nonathletic and uncoordinated is successful in a musical career, it may be related to the defense mechanism of: 1.Sublimation 2.Transference 3.Compensation 4.Rationalization

The client's perception of the circumstances surrounding the crisis

Which is the most important information for a nurse to gather from the client in crisis?

Milieu therapy

Which nursing intervention involves providing, structuring, and maintaining a safe and therapeutic environment in collaboration with patients, families, and other health care providers?

A nurse is caring for a client with a bipolar disorder depressive episode. What should the nurse's objective for this client be?

Feeling comfortable with the nurse

A client arrives at the mental health clinic disheveled, agitated, and demanding that the nurse "do something to make these feeling stop." What clinical manifestation is evident?

Feelings of panic

A client is admitted to the psychiatric service with a diagnosis of severe depression. When approached by the nurse, the client says, "You know I'm a sorry, lazy person. I don't deserve a job. I'm just stupid and no good." What does the nurse conclude that the client is experiencing?

Feelings of self-deprecation

By changing the topic to reality-based events

A client describes his delusions in minute detail to the nurse. How should the nurse respond?

A 2½-year-old child is admitted for treatment of injuries supposedly sustained in a fall down a flight of stairs. Child abuse is suspected. What statements might the nurse expect from a parent who engages in child abuse? (Select all that apply.)

"Every time I turn around the kid is falling over something." "I can't understand it. He didn't have a problem using the stairs without my help before this."

A male client with the diagnosis of a bipolar disorder, depressed episode, is found lying on the floor in his room in the psychiatric unit. He states, "I don't deserve a comfortable bed; give it to someone else." The best response response by the nurse is:

"Everyone has a bed. This one is yours."

A client in the outpatient clinic is denying that he is addicted to alcohol. He tells the nurse that he is not an alcoholic and that it is his nagging wife who causes him to drink. What is the most therapeutic response by the nurse?

"Everyone is responsible for his own actions."

A nurse uses the CAGE screening test for alcoholism to determine an individual's potential for a drinking problem. What is one of the four questions included on this test?

"Have you ever felt bad or guilty about your drinking?"

A nurse moves into the working phase of a therapeutic relationship with a depressed client who has a history of suicide attempts. What question should the nurse ask the client when exploring alternative coping strategies?

"How will you manage the next time your problems start piling up?"

The nurse finds a client with schizophrenia lying under a bench in the hall. The client says, "God told me to lie here." What is the best response by the nurse?

"I didn't hear anyone talking; come with me to your room."

A nurse enters a client's room and notes that the client appears preoccupied. Turning to the nurse, the client says, "They're saying terrible things about me. Can't you hear them?" What is the most therapeutic response by the nurse?

"I don't hear anyone else talking, but I can see that you're upset."

A client has been on the psychiatric unit for several days. The client arouses anxiety and frustration in the staff and manipulates them so well that staff members are afraid to approach the client. One morning the client shouts at the nurse, "You've worked it so I can't go for a walk with the group today. You're as cunning as a fox. I hate you! Get out, or I'll hit you!" What is the best response by the nurse?

"I don't like hearing your threats, but tell me more about your feelings."

A client is admitted to the hospital because of incapacitating obsessive-compulsive behavior. The statement that best describes how clients with obsessive-compulsive behavior view this disorder is:

"I know there's no reason to do these things, but I can't help myself."

A nurse has been assigned to work with a depressed client on a one-on-one basis. The next morning the client refuses to get out of bed, saying, "I'm too sick to be helped, and I don't want to be bothered." What is the best response by the nurse?

"I know you don't feel like getting up, but you might feel better if you did. Let me help you get started."

A client tells the nurse, "The voices say I'll be safe only if I stay in this room, wear these clothes, and avoid stepping on the cracks between the floor tiles." What is the best initial response by the nurse?

"I understand that these voices are real to you, but I want you to know that I don't hear them."

A client with schizophrenia is admitted to a psychiatric unit. The client is talking while walking in the hall, is unkempt, and obviously has not washed in several days. What should the nurse say when trying to help this client shower?

"I'll help you take your shower now."

A 17-year-old teenager is found to have leukemia. Which statements by the teenager reflect Piaget's cognitive processes associated with adolescence? (Select all that apply.)

"I'm going to do my best to fight this awful disease." "Now I can't go to the prom because I have this stupid disease." "This illness is serious, but with treatment I think I have a chance to get better."

A confused hallucinating client says, "My arms are turning to stone." What is the most therapeutic response by the nurse?

"It can be frightening to feel that way."

The parent of a child with a tentative diagnosis of attention deficit-hyperactivity disorder (ADHD) arrives at the pediatric clinic insisting on getting a prescription for medication that will control the child's behavior. What is best response by the nurse?

"It must be frustrating to deal with your child's behavior."

A client with schizophrenia plans an activity schedule with the help of the treatment team. A written copy is posted in the client's room. What should the nurse say when it is time for the client to go for a walk?

"It's time for you to go for a walk now."

During a routine yearly physical an older adult says to a nurse, "I haven't had sex lately because I can't get an erection anymore!" What should the nurse's initial response be?

"Let's discuss this concern a little more."

A client with alcohol dependence problem asks whether the nurse can see the bugs that are crawling on the bed. What is the nurse's initial reply?

"No, I don't see any bugs."

An adolescent female with an antisocial personality disorder plans to live with her parents after discharge. The parents request advice on how to respond to their daughter's unruly behavior. What is the most therapeutic response by the nurse?

"Set clear limits, explain the consequences if she disregards them, and firmly and consistently apply them."

A nurse determines that the information about falling down the stairs given by a parent suspected of child abuse contradicts the information given by the child. What should the nurse say to the parent?

"Tell me again how your child fell down the stairs."

A client is admitted to the surgical unit with superficial wounds of both wrists, the result of a suicide attempt. When the nurse enters the room, the client says, "I suppose you're going to ask me about my suicide attempt." What is the best response by the nurse?

"Tell me how you feel about it."

The parents of an overweight 12-year-old bring their child to the mental health clinic. One parent says, "You've got to do something to help us—just look how huge he is." The child tells the nurse, "I hate school. The other kids tease me about my weight. I'm always last when they pick teams in gym." What is the most therapeutic response by the nurse?

"That hurts a lot when you want to be liked."

A client is lonely and extremely depressed, and the health care provider prescribes a tricyclic antidepressant. The client asks the nurse what the medication will do. What is the best response by the nurse?

"The medication will increase your appetite and make you feel better."

Certain questions are applicable in determining nursing negligence. (Select all that apply.)

"Was reasonable care provided?" "Was there a breach of nursing duty?" "Was there an act of omission that resulted in harm? "Except for the nurse's action, would the injury have occurred?"

A client who has a history of a conversion reaction that involves weakness in the right arm that periodically progresses to paralysis is hospitalized on the mental health unit of the local community hospital. While listening to instructions for a group project, the client experiences a feeling of weakness and is unable to move the right arm. After evaluating the client, what should the nurse ask?

"What emotion were you feeling before you felt the weakness?"

A client with a diagnosis of panic disorder who had a panic attack on the previous day says to the nurse, "That was a terrible feeling I had yesterday. I'm so afraid to talk about it." What is the most therapeutic response by the nurse?

"What were you doing yesterday when you first noticed the feeling?"

One day the nurse and a young adult client sit together and draw. The client draws a face with horns and says, "This is me. I'm a devil." What is the best response by the nurse?

"When I look at you I see a person, not a devil."

A delusional client has refused to eat for the past 24 hours because, he says, "the food is poisoned." How should the nurse respond?

"You feel worried that someone wants to poison you?"

One morning a client with the diagnosis of acute depression says to the nurse, "God is punishing me for my past sins." What is the best response by the nurse?

"You really seem to be upset about this."

An older widower who is sitting by himself in a lounge in the nursing home, says, "I'm all alone; no one has any use for me." Which response by the nurse is most therapeutic?

"You seem upset. Let's talk about what's bothering you."

A 67-year-old man with type 2 diabetes sadly confides in the nurse that he has been unable to have an erection for several years. What is the best response by the nurse?

"You sound upset about not being able to have an erection."

An adult client charged with molesting a child is admitted for psychiatric evaluation. When a nurse invites the client to come to dinner, the client refuses and says, "I don't want anyone to see me. Leave me alone." What is the best response by the nurse?

"You sound upset; let's talk about it."

A depressed client tells a nurse, "I want to die." What is the most therapeutic response by the nurse?

"You would rather not live."

One evening a nurse finds a client who has been experiencing persecutory delusions trying to get out the door. The client begs, "Please let me go. I trust you. The Mafia is going to kill me tonight." Which response is most therapeutic?

"You're frightened. Come with me to your room, and we can talk about it."

One day while shaving, a male client with the diagnosis of bipolar disorder tells the nurse, "I've hidden a razor blade, and tonight I'm going to kill myself." What is the best reply by the nurse?

"You're going to kill yourself?"

A nurse is evaluating a child with suspected autism. At what age does the nurse determine that the signs of autism initially may be evident?

2 years

Engage in parallel play while sitting next to the child

A 6-year-old child with autism is nonverbal and makes limited eye contact. What should the nurse do initially to promote social interaction?

Repression

A 20-year-old woman is brought to an emergency department after having been raped. She is very anxious and cannot recall any of the circumstances surrounding the assault or provide the police with a description of the rapist. The nurse knows that the defense mechanism being utilized by this woman is:

Scapegoating -When all members of a family blame one member for all their problems, scapegoating is occurring. There are no data to indicate a controlling, patronizing, or overburdening pattern of relating.

A 12-year-old child who has a history of school failure and destructive acting out is admitted to a child psychiatric unit with the diagnosis of conduct disorder. The youngest of three children, the child is identified by both the parents and the siblings as the family problem. The nurse recognizes the family's pattern of relating to the child as:

Refer the mother to the psychiatrist -It is the responsibility of the psychiatrist, who is the primary care provider, to discuss the test results with the mother. Explaining to the mother the results of the tests is beyond the scope of the nurse's role. The mother should be referred to the psychiatrist, not the psychologist, because the psychiatrist is the leader of this health team. Teaching about the tests should have been done before, not after, the tests were administered.

A 13-year-old boy who recently was suspended from school for consistently bullying other children is brought to the pediatric mental health clinic by his mother. The child is assessed by the psychiatrist and referred to a psychologist for psychological testing. The day after the tests are completed, the mother returns to the clinic and asks the nurse for results of the tests. The nurse should:

Providing a safe, nonjudgmental environment

A 13-year-old girl is brought to the emergency department by her mother, who tells the nurse that she just found out that her daughter has been sexually abused by her grandfather for almost 2 years. What is the nurse's priority intervention?

Rewarding positive behavior -In behavior modification, positive behavior is reinforced and negative behavior is punished or not reinforced. Deconditioning the client's fears, decreasing the number of unnecessary restrictions, and reducing the number of anxiety-causing situations may each be a part of the program, but none is a major component.

A 17-year-old client is admitted to the hospital because of weight loss and malnutrition, and the health care provider diagnoses anorexia nervosa. After the client's physical condition is stabilized, the provider, in conjunction with the client and parents, decides to institute a behavior-modification program. What component of behavior modification verbalized by one of the parents leads the nurse to conclude that the parent has an understanding of the therapy?

Integrity versus despair

A 65-year-old man is admitted to the hospital with a history of depression. The client, who speaks little English and has had few outside interests since retiring, says, "I feel useless and unneeded." The nurse concludes that the client is in Erikson's developmental stage of:

2. "I'm going to do my best to fight this terrible disease." 3. "Now I can't go to the prom because I have this stupid illness." 5. "This illness is serious, but with treatment I think I will have a chance to get better." 1. At 2 to 7 years of age children are in the preoperational stage of cognitive development. They believe that external, unrelated, concrete phenomena cause illness. 2. At 17 years of age the adolescent is in the formal operational stage of cognitive development and therefore is able to understand the seriousness of leukemia. 3. At 17 years of age the adolescent is in the formal operational stage of cognitive development and therefore understands the seriousness of the illness. The statement also reflects an adolescent's preoccupation with peer socialization. 4. At 7 to 10 years of age children are in the concrete operational stage of cognitive development. Because of their egocentrism, they believe that they are responsible for situations, such as illnesses, and are being punished. 5. At 17 years of age the adolescent is in the formal operational stage of cognitive development and therefore is able to comprehend the seriousness of leukemia and the need for treatment.

A 17-year-old teenager is diagnosed with leukemia. Which statements by the teenager reflect Piaget's cognitive processes associated with adolescence? Select all that apply. 1. "My smoking pot probably caused the leukemia." 2. I'm going to do my best to fight this terrible disease." 3. "Now I can't go to the prom because I have this stupid illness." 4. "I know I got sick because I've been causing a lot of problems at home." 5. "This illness is serious, but with treatment I think I will have a chance to get better."

Superego

A 19 year-old, arrested for assault and robbery, has a history of truancy and prostitution but is unconcerned that her behavior has caused emotional distress to others. The diagnosis of antisocial personality disorder is made. According to psychoanalytical theory, the client's lack of remorse and repetitive behavior probably are related to an underdeveloped:

3.Superego

A 19 year-old, arrested for assault and robbery, has a history of truancy and prostitution but is unconcerned that her behavior has caused emotional distress to others. The diagnosis of antisocial personality disorder is made. According to psychoanalytical theory, the client's lack of remorse and repetitive behavior probably are related to an underdeveloped: 1.Id 2.Ego 3.Superego 4.Limbic system

3. Superego

A 19 year-old, arrested for assault and robbery, has a history of truancy and prostitution but is unconcerned that her behavior has caused emotional distress to others. The diagnosis of antisocial personality disorder is made. According to psychoanalytical theory, the client's lack of remorse and repetitive behavior probably are related to an underdeveloped: 1. Id 2. Ego 3. Superego 4. Limbic system

Asking the client about the events of his day A broad opening encourages communication that may elicit the client's perception of the day's events. Obtaining a prescription for a tranquilizer is premature. What is most important is the client's, not the parents', perception of what has occurred. Assigning a nursing assistant to remain with the client is premature; there are no data to indicate that the client may harm himself or others.

A 22-year-old male client with the diagnosis of schizophrenia has been in a mental health facility for approximately 2 weeks. After his parents visit he is seen pacing in the hall, talking loudly to himself. What should the nurse's initial intervention be? Obtaining a prescription for a tranquilizer Asking the client about the events of his day Calling the parents to find out what happened Assigning a nursing assistant to remain with the client

Projection

A 23-year-old woman is admitted to a psychiatric unit after several episodes of uncontrolled rage at her parents' home, and borderline personality disorder is diagnosed. While watching a television newscast describing an incident of violence in the home, the client says, "People like that need to be put away before they kill someone." The nurse concludes that the client is using:

2. Projection **Projection is the process of attributing one's thoughts about one's self to others.

A 23-year-old woman is admitted to a psychiatric unit after several episodes of uncontrolled rage at her parents' home, and borderline personality disorder is diagnosed. While watching a television newscast describing an incident of violence in the home, the client says, "People like that need to be put away before they kill someone." The nurse concludes that the client is using: 1. Denial 2. Projection 3. Introjection 4. Sublimation

1 Impulsivity 2 Panic attacks 3 Unemployment 4 Substance abuse -Impulsivity, panic attacks, unemployment, and substance abuse have all been linked with an increased risk for suicide. A sense of responsibility to family and religious beliefs are considered protective factors that may lessen the risk of suicide.

A 25-year-old woman is seeking outpatient counseling after thinking about suicide. The nurse realizes that there some factors place individuals at a higher risk for suicide. Which of these factors increases the risk for suicide? (Select all that apply.)

2 "Every time I turn around the kid is falling over something." 4"I can't understand it. He didn't have a problem using the stairs without my help before this."

A 2½-year-old child is admitted for treatment of injuries supposedly sustained in a fall down a flight of stairs. Child abuse is suspected. What statements might the nurse expect from a parent who engages in child abuse? (Select all that apply.)

2. "Every time I turn around the kid is falling over something." 4. "I can't understand it. He didn't have a problem using the stairs without my help before this."

A 2½-year-old child is admitted for treatment of injuries supposedly sustained in a fall down a flight of stairs. Child abuse is suspected. What statements might the nurse expect from a parent who engages in child abuse? (Select all that apply.) 1. "Kids have to learn to be careful on the stairs." 2. "Every time I turn around the kid is falling over something." 3. "He tends to be adventurous and doesn't understand about getting hurt on the stairs." 4. "I can't understand it. He didn't have a problem using the stairs without my help before this." 5. "I try to keep an eye on him, but little kids are always on the go and I just can't keep running after him."

Responsiveness to the parents -One of the symptoms that an autistic child displays is lack of responsiveness to others; there is little or no extension to the external environment. Music is nonthreatening, comforting, and soothing. Repetitive behavior provides comfort. Repetitive visual stimuli, such as a spinning top, are nonthreatening and soothing.

A 3-year-old child is found to have a pervasive developmental disorder not otherwise specified (autistic disorder). What should the nurse consider most unusual for the child to demonstrate?

Avoids eye-to-eye contact Performs repetitive activities

A 3-year-old child is found to have autism. Which behaviors should the nurse expect when observing this child? (Select all that apply.)

2 Situational

A 44-year-old client has been unable to function since her husband asked for a divorce 2 weeks ago. She is brought to the crisis intervention center by a friend. What type of crisis is this situation?

A desire to control her life

A 5-foot 5-inch 15-year-old girl who weighs 80 lb is admitted to a mental health facility with a diagnosis of anorexia nervosa. The nurse recognizes that her problem most likely is caused by:

Use another activity to distract the child

A 6-year-old child who has autism exhibits frequent spinning and hand-flapping behaviors. What should the nurse teach the parents to do to limit these actions?

Reassuring the client with the frequent presence of staff -The client needs constant reassurance because forgetfulness blocks previous explanations; frequent presence of staff serves as a continual reminder.

A 54-year-old client has demonstrated increasing forgetfulness, irritability, and antisocial behavior. After being found walking down a street, disoriented and semi-naked, the client is admitted to the hospital, and a diagnosis of dementia of the Alzheimer type is made. The client expresses fear and anxiety. What is the best approach for the nurse to take?

"I am not the devil! Stop calling me those names!" The client is responding to messages that he is hearing, which are auditory hallucinations. The responses regarding the snakes and the spaceship are examples of visual hallucinations because they describe what the client sees. The accusation of poisoning is the statement of a client who is suspicious and paranoid but not hallucinating.

A 56-year-old man is admitted to the inpatient unit after family members report that he seems to be experiencing auditory hallucinations. The man has a history of schizophrenia and has had several previous admissions. Which statement indicates to the nurse that the client is experiencing auditory hallucinations? "Get these horrible snakes out of my room!" "I am not the devil! Stop calling me those names!" "The food on this plate has poison in it, so take it away—I won't eat it." "I did see an alien spaceship last night outside in my yard, and I've felt worse ever since."

"Taking insulin for your diabetes is like getting new batteries for your superhero toys." The child is in Piaget's stage of preoperational thought, which is manifested by magical thinking; therefore, teaching should also use magical thinking.

A 6-year-old child is diagnosed with type 1 diabetes. Considering the child's cognitive developmental level, which explanation of the illness is most appropriate? "Diabetes is caused by not having any insulin in your body." "Diabetes will require you to take insulin shots for the rest of your life." "You will be taught how to give yourself insulin now that you have diabetes." "Taking insulin for your diabetes is like getting new batteries for your superhero toys."

"Taking insulin for your diabetes is like getting new batteries for your superhero toys."

A 6-year-old child is found to have type 1 diabetes. In light of the child's cognitive developmental level, which explanation of the illness is most appropriate?

2.Integrity versus despair

A 65-year-old man is admitted to the hospital with a history of depression. The client, who speaks little English and has had few outside interests since retiring, says, "I feel useless and unneeded." The nurse concludes that the client is in Erikson's developmental stage of: 1.Initiative versus guilt 2.Integrity versus despair 3.Intimacy versus isolation 4.Identity versus role confusion

The child may be blaming himself for his parents' breakup.

A 7-year-old boy is brought to the clinic by the mother, who tells the nurse that her child has been having trouble in school, has difficulty concentrating, and is falling behind in schoolwork since she and her husband separated 6 months ago. The mother reports that lately her child has not been eating dinner, and she often hears him crying when he is alone. What basis for these behaviors should the nurse consider?

Open communication

A child has been hospitalized repeatedly for illnesses of unknown origin. Finally the health care provider makes the diagnosis of Munchausen syndrome by proxy. What is the most therapeutic approach by the nurse to the involved parent?

Developmental level of the child

A child in the first grade is murdered, and counseling is planned for the other children in the school. What should a nurse identify first before evaluating a child's response to a crisis?

1. Developmental level of the child **Rationale: Knowledge of the developmental level is essential to understanding a child's response to a crisis situation; the variety of coping abilities usually increases as the child progresses through the stages of growth and development.

A child in the first grade is murdered, and counseling is planned for the other children in the school. What should a nurse identify first before evaluating a child's response to a crisis? 1. Developmental level of the child 2. Quality of the child's peer relationships 3. Child's perception of the crisis situation 4. Child's communication patterns with family members

Feelings of panic -The client can no longer control or tolerate these overwhelming feelings and is seeking help.

A client arrives at the mental health clinic disheveled, agitated, and demanding that the nurse "do something to make these feeling stop." What clinical manifestation is evident?

"These medications affect the chemicals used in communication between nerve cells."

A client asks the nurse how psychotropic medications work. The nurse correctly replies:

3. "These medications affect the chemicals used in communication between nerve cells."

A client asks the nurse how psychotropic medications work. The nurse correctly replies: 1. "These medications decrease the metabolic needs of your brain." 2. "These medications increase the production of healthy nervous tissue." 3. "These medications affect the chemicals used in communication between nerve cells." 4. "These medications regulate the sensory input received from the external environment."

Quickly, with an attitude of concern

A client begins fighting and biting other clients. The practitioner prescribes a stat injection of haloperidol (Haldol). How should the nurse implement this prescription?

1. Quickly, with an attitude of concern **Quickness is used for safety; an attitude of concern may help reduce the client's anxiety.

A client begins fighting and biting other clients. The practitioner prescribes a stat injection of haloperidol (Haldol). How should the nurse implement this prescription? 1. Quickly, with an attitude of concern 2. Before the client realizes what is happening 3. After the client agrees to receive the injection 4. Quietly, without any explanation of the reason for it

Absence of memory in relation to drinking episodes

A client being admitted for alcoholism reports having had alcoholic blackouts. The nurse knows that an alcoholic blackout is best described as:

Supplying the client with tissues to maintain function until the anxiety eases -The client is using this compulsive behavior to control anxiety and needs to continue with it until the anxiety is reduced and more acceptable methods are developed to handle it.

A client believes that doorknobs are contaminated and refuses to touch them except with a paper tissue. What nursing intervention will be most therapeutic for this client?

4 Is responding to the antidepressant therapy

A client has become increasingly depressed, and the practitioner prescribes an antidepressant. After 20 days of therapy, the client returns to the clinic. The client appears relaxed and smiles at the nurse. The most significant conclusion that the nurse can draw from this behavior is that the client:

Increased risk for suicide

A client has been hospitalized for 3 weeks while receiving a tricyclic medication for severe depression. One day the client says to the nurse, "I'm really feeling better; my energy level is up." After the encounter an aide tells the nurse that the client has given away his favorite jacket. What should the nurse conclude that the client's statement indicates?

Consults with his provider regarding alternative medication therapies Although erectile dysfunction can result from conventional antipsychotic medication therapy, the provider is often able to prescribe an alternative medication that will help manage the symptoms but is less likely to cause the dysfunction.

A client has been prescribed chlorpromazine (Thorazine) for the management of positive symptoms of schizophrenia. When the client reports difficulty sustaining an erection, the nurse:

2. Consults with his provider regarding alternative medication therapies

A client has been prescribed chlorpromazine (Thorazine) for the management of positive symptoms of schizophrenia. When the client reports difficulty sustaining an erection, the nurse: 1. Reassures him this side effect will resolve in a few weeks 2. Consults with his provider regarding alternative medication therapies 3. Explains that all conventional antipsychotic medications cause impotence 4. Provides additional medication education to explain the medication's side effects in detail

Consults with his provider regarding alternative medication therapies Although erectile dysfunction can result from conventional antipsychotic medication therapy, the provider is often able to prescribe an alternative medication that will help manage the symptoms but is less likely to cause the dysfunction. Education regarding side effects is certainly appropriate, but such information will only confirm that the side effect is not likely to subside with time.

A client has been prescribed chlorpromazine (Thorazine) for the management of positive symptoms of schizophrenia. When the client reports difficulty sustaining an erection, the nurse: Reassures him this side effect will resolve in a few weeks Consults with his provider regarding alternative medication therapies Explains that all conventional antipsychotic medications cause impotence Provides additional medication education to explain the medication's side effects in detai

3. Regularly testing the level of the drug in the client's blood

A client has been prescribed lithium. What important nursing intervention must be implemented while this medication is being administered? 1. Restricting the client's daily sodium intake 2. Testing the client's urine specific gravity weekly 3. Regularly testing the level of the drug in the client's blood 4. Withholding the client's other medications for several days

Constricted pupils -Pupil constriction is a physical response to opioid intoxication; the pupils will dilate with opioid overdose. Opioids cause apathy or a depressed, sad mood (dysphoria); lability of mood is associated with the use of anabolic-androgenic steroids. Opioids cause drowsiness and psychomotor retardation; alertness is associated with the use of stimulants such as caffeine and amphetamines. Opioids depress the respiratory center of the brain, causing slow, shallow respirations; increases in temperature, pulse, respirations, and blood pressure are associated with cocaine use.

A client has been receiving oxycodone (OxyContin) for moderate pain associated with multiple injuries sustained in a motor vehicle collision. The client has returned three times for refills of the prescription. What behavior, in addition to the client's slurred speech, leads the nurse to suspect opioid intoxication?

"It can take 1 to 4 weeks to see an improvement."

A client has been taking escitalopram (Lexapro) for treatment of a major depressive episode. On the fifth day of therapy the client refuses the medication, stating, "It doesn't help, so what's the use of taking it?" What is the best response by the nurse?

Take the client's vital signs and arrange for immediate transfer to a hospital

A client has been taking prescribed risperidone (Risperdal) 3 mg twice a day for the past 8 days. A friend brings the client to the outpatient clinic. The client reports tremors, shortness of breath, a fever, and sweating. What should the nurse do?

Express anger or frustration -Typically, recurrent self-mutilation is an expression of intense anger, helplessness, or guilt or is a form of self-punishment.

A client has just been admitted with the diagnosis of borderline personality disorder. There is a history of suicidal behavior and self-mutilation. The nurse remembers that the main reason that clients use self-mutilation is to:

Refocus the conversation on the client's fears, frustrations, and anger about the client's condition

A client has just spent five minutes complaining to the nurse about numerous aspects of the client's hospital stay. Which is the best initial response by the nurse?

Benztropine mesylate (Cogentin) -Benztropine (Cogentin), an anticholinergic, helps balance neurotransmitter activity in the central nervous system (CNS) and helps control extrapyramidal tract symptoms.

A client has recently started taking a new neuroleptic drug, and the nurse notes extrapyramidal effects. Which drug does the nurse anticipate will be prescribed to limit these side effects?

Spending time with depressed clients demonstrates that they are worthy of attention. -A severely depressed client has low self-esteem; this intervention demonstrates that the client is important and worthy of attention.

A client in an acute mental health unit appears severely depressed. The client does not initiate conversations or perform personal care. Questions are answered with a barely audible one- or two-word response. The nurse sits with the client and makes no demands. On what premise is the nurse's intervention for this client based?

1 Weighing the client once a week 2 Having specialized rehabilitation equipment available 3 Establishing a schedule with periods of rest after activities

A client in the early dementia stage of Alzheimer's disease is admitted to a long-term care facility. Which activities must the nurse initiate? (Select all that apply.)

Imagery

A client in the mental health clinic has a phobia about closed spaces. Which desensitization method should the nurse expect to be used successfully with this client?

Elimination of benzodiazepines for nighttime sedation

A client is scheduled for a 6-week electroconvulsive therapy (ECT) treatment program. What intervention is important during the course of treatment?

Introjection

A client in the mental health clinic who has been seeing a therapist for more than 6 months begins to talk and act like a therapist who is analyzing coworkers. What defense mechanism does the nurse identify?

3. Introjection **Introjection is treating something outside the self as if it is actually inside the self; it is unconsciously incorporating the wishes, values, and attitudes of another as if they were one's own.

A client in the mental health clinic who has been seeing a therapist for more than 6 months begins to talk and act like a therapist who is analyzing coworkers. What defense mechanism does the nurse identify? 1. Undoing 2. Projection 3. Introjection 4. Intellectualization

Rationalization

A client in the mental health clinic who has concerns about getting married says to the nurse, "I guess I'd better get married. All the plans are made and paid for, and the invitations have all been mailed." What defense mechanism is the client using?

Clomipramine (Anafranil)

A client is admitted for treatment of obsessive-compulsive disorder that is interfering with activities of daily living. Which medication should the nurse anticipate the health care provider will prescribe?

3. Clomipramine (Anafranil)

A client is admitted for treatment of obsessive-compulsive disorder that is interfering with activities of daily living. Which medication should the nurse anticipate the health care provider will prescribe? 1. Benztropine (Cogentin) 2. Amantadine (Symmetrel) 3. Clomipramine (Anafranil) 4. Diphenhydramine (Benadryl)

Involve the client in activities that promote success

A client is admitted to a mental health facility for depression. What action should a nurse take to help the client develop a positive self-regard?

Helping the client learn to trust the staff through selected experiences

A client is admitted to a psychiatric hospital with the diagnosis of schizoid personality disorder. Which initial nursing intervention is a priority for this client?

Impulsive -Impulsive, potentially self-damaging behaviors are typical of clients with this personality disorder.

A client is found to have a borderline personality disorder. What behavior does the nurse consider is most typical of these clients?

Risk for self-injury -The greatest risk in cocaine withdrawal is risk for self-injury. The risk for seizure is increased while a person is under the influence of cocaine, not during withdrawal. Although dehydration may occur during cocaine use and withdrawal, it is not the priority concern. People in cocaine withdrawal, although irritable, are more apt to hurt themselves than others.

A client is admitted to the drug detoxification unit for cocaine withdrawal. What is the nurse's primary concern while working with clients withdrawing from cocaine?

"I know there's no reason to do these things, but I can't help myself."

A client is admitted to the hospital because of incapacitating obsessive-compulsive behavior. The statement that best describes how clients with obsessive-compulsive behavior view this disorder is:

Saying, "I see that you're crying. Tell me what's going on in your life, and we can work on helping you."

A client is admitted to the mental health unit after attempting suicide. When a nurse approaches, the client is tearful and silent. What is the best initial nursing intervention?

Fluvoxamine (Luvox) -Fluvoxamine (Luvox) inhibits central nervous system neuron uptake of serotonin but not of norepinephrine. Haloperidol (Haldol) is not an SSRI; it is an antipsychotic that blocks neurotransmission produced by dopamine at synapses. Imipramine (Tofranil) is a tricyclic antidepressant, not an SSRI. Benztropine (Cogentin) is an antiparkinsonian agent, not an SSRI.

A client is admitted to the psychiatric hospital with a diagnosis of obsessive-compulsive disorder. The client's anxiety level is approaching a panic level, and the client's ritual is interfering with work and daily living. Which selective serotonin reuptake inhibitor (SSRI) should the nurse anticipate that the health care provider may prescribe?

2. Fluvoxamine (Luvox) **Fluvoxamine (Luvox) inhibits central nervous system neuron uptake of serotonin but not of norepinephrine.

A client is admitted to the psychiatric hospital with a diagnosis of obsessive-compulsive disorder. The client's anxiety level is approaching a panic level, and the client's ritual is interfering with work and daily living. Which selective serotonin reuptake inhibitor (SSRI) should the nurse anticipate that the health care provider may prescribe? 1. Haloperidol (Haldol) 2. Fluvoxamine (Luvox) 3. Imipramine (Tofranil) 4 .Benztropine (Cogentin)

Appearing composed

A client is admitted to the psychiatric unit of the hospital with a diagnosis of conversion disorder. The client is unable to move either leg. Which finding should the nurse consider consistent with this diagnosis?

Exploring ways to verbalize feelings -The priority is getting the client to express feelings appropriately rather than through the use of physical symptoms. Focusing on symptoms will encourage their use by the client. An expression of feelings, not an intellectual understanding of the cause of the symptoms, is required.

A client is admitted with a conversion disorder. What is the primary nursing intervention?

1 Euphoria 2 Agitation 5 Hypervigilance 6 Impaired Judgement

A client is brought to the emergency department by friends because of increasingly bizarre behavior. Which signs does the nurse identify that indicate that the client was using cocaine? (Select all that apply.)

"Why don't you share your feelings with him while you can?" -It is difficult to work through a loss; however, encouraging the sharing of feelings helps both parties to feel better about having to let go.

A client is dying. Hesitatingly, his wife says to the nurse, "I'd like to tell him how much I love him, but I don't want to upset him." Which is the best response by the nurse?

"It will help increase your appetite and make you feel better."

A client is extremely depressed, and the practitioner prescribes a tricyclic antidepressant, imipramine (Tofranil). The client asks the nurse what the medication will do. The nurse responds:

4."It will help increase your appetite and make you feel better."

A client is extremely depressed, and the practitioner prescribes a tricyclic antidepressant, imipramine (Tofranil). The client asks the nurse what the medication will do. The nurse responds: 1."It will help you forget why you are depressed." 2."It will help keep you alert and cure your insomnia." 3."It will help you feel better after taking it for several days." 4."It will help increase your appetite and make you feel better."

Low self-esteem

A client is found to have an adjustment disorder with mixed anxiety and depression. What should the nurse anticipate as the client's primary problem?

Akathisia

A client is found to have paranoid schizophrenia, and the practitioner prescribes a typical antipsychotic medication. After a 1-month hospitalization the client is discharged home with instructions to continue the antipsychotic and a referral for weekly mental health counseling. The picture illustrates the client's physical status as observed by the nurse on the client's first visit to the community mental health clinic. What extrapyramidal side effect has developed?

"The medication will increase your appetite and make you feel better." -Tricyclic antidepressants create a general sense of well-being, increase appetite, and help lift depression.

A client is lonely and extremely depressed, and the health care provider prescribes a tricyclic antidepressant. The client asks the nurse what the medication will do. What is the best response by the nurse?

4. Many prescribed and over-the-counter drugs cannot be taken with this medication

A client is receiving a monoamine oxidase inhibitor (MAOI). What should the nurse teach the client? 1. It is necessary to avoid the sun. 2. Drowsiness is an expected side effect of this medication. 3. The therapeutic and toxic levels of the drug are very close. 4. Many prescribed and over-the-counter drugs cannot be taken with this medication

Mydriasis

A client is receiving doxepin (Sinequan). For which most dangerous side effect of tricyclic antidepressants should a nurse monitor the client?

1. Mydriasis

A client is receiving doxepin (Sinequan). For which most dangerous side effect of tricyclic antidepressants should a nurse monitor the client? 1. Mydriasis 2. Dry mouth 3. Constipation 4. Urine retention

Symptoms of the heroin overdose may return after the naloxone is metabolized.

A client is responding within an hour of receiving naloxone to combat respiratory depression from an overdose of heroin. Why should a nurse continue to closely monitor this client's status?

3.Arrives early and waits quietly to be called for the tests Rationale: 3. The client's early arrival indicates an expected degree of anxiety; the quiet waiting indicates that the client has been told what to expect.

A client is scheduled for several diagnostic studies. Which behavior best indicates to the nurse that the client has received adequate preparation? 1.Requests that the tests be reexplained 2.Checks the appointment card repeatedly 3.Arrives early and waits quietly to be called for the tests 4.Paces up and down the hallway the morning of the tests

Renal Studies -Because of the severity of side effects and the stress lithium places on the renal and cardiovascular systems, its administration is contraindicated in clients with renal or cardiovascular disease. Baseline renal studies can be used for comparison in the future.

A client is to begin lithium carbonate therapy. The nurse should ensure that before the drug's administration the client has baseline:

1. Renal studies

A client is to begin lithium carbonate therapy. The nurse should ensure that before the drug's administration the client has baseline: 1. Renal studies 2. Liver enzyme studies 3. Adrenal function studies 4. Pulmonary function studies 5. New Choice 6. New Choice 7. New Choice

A client is allowed to consent to or refuse potential psychiatric treatments if a future incapacitating mental health crisis occurs.

A client on the psychiatric unit asks a nurse about psychiatric advance directives (PADs). What information should form the basis of the nurse's response?

Having the client verbalize her understanding and the outcomes of the procedure

A client on the psychiatric unit is undergoing a pretreatment evaluation for electroconvulsive therapy (ECT). Because of the client's profoundly depressed behavior, the nurse doubts that the client can provide informed consent. What should the nurse's initial intervention be?

2. Having the client verbalize her understanding and the outcomes of the procedure

A client on the psychiatric unit is undergoing a pretreatment evaluation for electroconvulsive therapy (ECT). Because of the client's profoundly depressed behavior, the nurse doubts that the client can provide informed consent. What should the nurse's initial intervention be? 1. Consulting with the hospital's legal staff and following their recommendation 2. Having the client verbalize her understanding and the outcomes of the procedure 3. Asking the client to sign the consent form because the client has not been declared incompetent 4. Suggesting to the health care provider that a family member sign the consent form for the client

"I'll leave for now, but I'll be back later."

A client on the psychiatric unit sits alone most of the day. The nurse approaches the client. As the nurse gets approximately 3 feet away, the client lets out a string of profanity and shouts, "Leave me alone; I don't want to talk to you!" What is the most appropriate response by the nurse?

1."I'll leave for now, but I'll be back later."

A client on the psychiatric unit sits alone most of the day. The nurse approaches the client. As the nurse gets approximately 3 feet away, the client lets out a string of profanity and shouts, "Leave me alone; I don't want to talk to you!" What is the most appropriate response by the nurse? 1."I'll leave for now, but I'll be back later." 2."Why do you feel the need to greet me like that?" 3."Don't talk to me like that—I'm here to spend time with you." 4."I don't like it when you talk like that—are you trying to push me away?"

"I know that these voices are real to you, but I want you to know that I don't hear them."

A client on the psychiatric unit tells the nurse, "The voices have told me that I'm in danger. They say I'll be safe only if I stay in this room, wear these clothes, and avoid stepping on the cracks between the floor tiles." What is the best initial response by the nurse to this statement?

Suicidal clients may not sign out even if they voluntarily admitted themselves.

A client on the psychiatric unit who has suicidal ideas says to the nurse, "I signed myself in. I'll sign myself out." What concept provides the basis for the nurse's response?

4.Suicidal clients may not sign out even if they voluntarily admitted themselves

A client on the psychiatric unit who has suicidal ideas says to the nurse, "I signed myself in. I'll sign myself out." What concept provides the basis for the nurse's response? 1.Voluntary clients may sign out at any time. 2.Voluntary clients may sign out by following unit procedures. 3.Suicidal clients may sign out if they are able to contract for their safety. 4.Suicidal clients may not sign out even if they voluntarily admitted themselves

4. Suicidal clients may not sign out even if they voluntarily admitted themselves. **The priority is to keep the client safe; a client admitted on a voluntary basis may be kept involuntarily if professional judgment indicates that the client may harm him- or herself or others.

A client on the psychiatric unit who has suicidal ideas says to the nurse, "I signed myself in. I'll sign myself out." What concept provides the basis for the nurse's response? 1. Voluntary clients may sign out at any time. 2. Voluntary clients may sign out by following unit procedures. 3. Suicidal clients may sign out if they are able to contract for their safety. 4. Suicidal clients may not sign out even if they voluntarily admitted themselves.

Withhold the medication

A client receiving the medication buspirone hydrochloride (Buspar) is admitted to the hospital with the diagnosis of possible hepatitis. The nurse identifies that the client's sclerae look yellow. What should be the nurse's initial action?

Providing false reassurance

A client says, "Since my husband died I've got nothing to live for. I just want to die." The nurse hears the nursing assistant say, "Things will get better soon." The nurse identifies this response as:

Regressive

A client sits huddled in a chair and leaves it only to assume the fetal position in a corner. The nurse, observing this, identifies the behavior as

2. Regressive **Curling up in a corner reflects the early fetal position; the individual curls up for both protection and security

A client sits huddled in a chair and leaves it only to assume the fetal position in a corner. The nurse, observing this, identifies the behavior as: 1. Reactive 2. Regressive 3. Dissociative 4. Hallucinatory

1."Have you been thinking about suicide?"

A client who appears dejected, barely responds to questions, and walks very slowly about the mental health unit tells the nurse in a barely audible voice that life is no longer worth living. What is the most therapeutic response to this statement by the nurse? 1."Have you been thinking about suicide?" 2."What could be so bad to make you feel that way?" 3."We'll talk about your feelings after you've rested." 4."Let's talk about something pleasant to make you feel better."

4.Encouraging the client to replace negative thoughts with positive thoughts 5.Helping the client to modify the belief that anything less than perfection is unacceptable Rationale: 4. Cognitive therapy seeks to find underlying self-defeating beliefs and replace them with more reality-based positive beliefs. 5. Cognitive therapy encourages the use of cognitive restructuring (cognitive reframing) through positive self-talk and rational mindset.

A client states, "I get down on myself when I make a mistake." When a cognitive therapy approach is used, which nursing interventions are most appropriate? Select all that apply. 1.Teaching the client relaxation exercises to diminish stress 2.Exploring with the client past experiences that caused distress 3.Providing the client with mastery experiences to boost self-esteem 4.Encouraging the client to replace negative thoughts with positive thoughts 5.Helping the client to modify the belief that anything less than perfection is unacceptable

"Negative thoughts can precipitate anxiety."

A client tells the nurse in the mental health clinic that the practitioner said that the cornerstone of therapy used in the clinic is cognitive therapy. The client asks what this therapy entails. What concept should the nurse explain as the basis of cognitive therapy?

"Huffing paint can damage your lungs, kidneys, and liver."

A client tells the nurse, "All my friends experiment with drugs. I like the high I get when I huff paint. Nothing bad is going to happen to me." What is the best response by the nurse?

"I understand that these voices are real to you, but I want you to know that I don't hear them." -The statement "I understand that these voices are real to you, but I want you to know that I don't hear them" demonstrates recognition and acceptance of the client's feelings; it also points out reality

A client tells the nurse, "The voices say I'll be safe only if I stay in this room, wear these clothes, and avoid stepping on the cracks between the floor tiles." What is the best initial response by the nurse?

"Have you been thinking about suicide?"

A client who appears dejected, barely responds to questions, and walks very slowly about the mental health unit tells the nurse in a barely audible voice that life is no longer worth living. What is the most therapeutic response to this statement by the nurse?

Reply, "I'll stay with you for a while because you seem frightened."

A client who is hallucinating actively approaches the nurse and reports, "I'm hearing voices that are saying bad things about me." What should the nurse do?

1. Obtaining vital signs 2. Assessing for suicidal thoughts 3. INstituting continuous monitoring 4. Initiating a therapeutic relationship 5. Inspecting the bandages for bleeding

A client who attempted suicide by slashing her wrists is transferred from the emergency department to a mental health unit. What important nursing interventions must be implanted when the client arrives on the unit? (Select all that apply.)

"It seems unfair that you should have this disease."

A client who has recently been found to be infected with HIV comments to the nurse, "There are so many terrible people around. Why couldn't one of them get HIV instead of me?" What is the best response by the nurse?

Verbalizes that a substance abuse problem exists

A client who is a polysubstance abuser is mandated to seek drug and alcohol counseling. What is an appropriate initial outcome criterion for this client?

Reacting to the planned discharge

A client who is being treated in a mental health clinic is to be discharged after several months of therapy. The client anxiously tells the nurse, "I don't know what I'll do when I can't see you anymore." The nurse determines that the client is:

2.Reacting to the planned discharge

A client who is being treated in a mental health clinic is to be discharged after several months of therapy. The client anxiously tells the nurse, "I don't know what I'll do when I can't see you anymore." The nurse determines that the client is: 1.Expressing thanks to the nurse 2.Reacting to the planned discharge 3.Attempting to manipulate the nurse 4.Indicating a need for further treatment

2. Reacting to the planned discharge

A client who is being treated in a mental health clinic is to be discharged after several months of therapy. The client anxiously tells the nurse, "I don't know what I'll do when I can't see you anymore." The nurse determines that the client is: 1. Expressing thanks to the nurse 2. Reacting to the planned discharge 3. Attempting to manipulate the nurse 4. Indicating a need for further treatment

3 Diaphoresis 4 Tachycardia 5 Hypertension -As withdrawal from alcohol progresses, autonomic hyperactivity occurs, resulting in profuse diaphoresis, a heart rate faster than 100 beats/min, and increases in temperature, respiratory rate, and blood pressure. Polydipsia, excessive intake of oral fluids, is one of the signs of diabetes mellitus. Hyperalertness, not drowsiness, may occur.

A client who is on the third day of detoxification therapy becomes agitated and restless. What are the signs and symptoms that indicate impending alcohol withdrawal delirium? (Select all that apply.) 1 Polydipsia 2 Drowsiness 3 Diaphoresis 4 Tachycardia 5 Hypertension

Discontinue the medication and, if the health care provider is unavailable today, go to the emergency department for evaluation

A client who is taking clozapine (Clozaril) calls the nurse in the psychiatric clinic to report the sudden development of a sore throat and a high fever. What should the nurse instruct the client to do?

Probably associated with a toxic level of lithium

A client who is taking lithium arrives at the mental health center for a routine visit. The client has slurred speech, has an ataxic gait, and complains of nausea. The nurse knows that these signs and symptoms are:

Invite another client to take part in a joint activity with the nurse and the client

A client whose depression is beginning to lift remains aloof from the other clients on the mental health unit. How can a nurse help the client participate in an activity?

Telling the client that it is frustrating not to get the correct tray but that throwing the tray at the dietitian is unacceptable behavior

A client with a borderline personality disorder receives the wrong meal tray for lunch and angrily states, "The next time I see the dietitian, I'm going to throw this tray at her!" What is the most appropriate response by the nurse?

Place the client in a private room to provide a quiet atmosphere

A client with a diagnosis of bipolar disorder, manic episode, is admitted to the mental health unit. Because the environment is important, what should the nurse do?

Splitting -Splitting is the compartmentalization of opposite-affect states and failure to integrate the positive and negative aspects of self or others.

A client with a diagnosis of borderline personality disorder (BPD) has negative feelings toward the other clients on the unit and considers them all "bad." Which defense is the client using when identifying the other clients thusly?

1. Splitting

A client with a diagnosis of borderline personality disorder (BPD) has negative feelings toward the other clients on the unit and considers them all "bad." Which defense is the client using when identifying the other clients thusly? 1. Splitting 2. Ambivalence 3. Passive aggression 4. Reaction formation

1.Splitting Rationale: 1. Splitting is the compartmentalization of opposite-affect states and failure to integrate the positive and negative aspects of self or others.

A client with a diagnosis of borderline personality disorder has negative feelings toward the other clients on the unit and considers them all to be "bad." Which defense was the client using when this statement was made? 1.Splitting 2.Ambivalence 3.Passive aggression 4.Reaction formation

Accepting the client's feelings about activities calmly while setting firm limits

A client with a diagnosis of major depression refuses to participate in unit activities, saying that she is "just too tired." What is the best nursing approach?

2. Arrange a unit meeting to discuss what has just happened

A client with a diagnosis of paranoid schizophrenia throws a chair across the room and starts screaming at the other clients. Several of these clients have frightened expressions, one starts to cry, and another begins to pace. A nurse removes the agitated client from the room. What should the nurse remaining in the room do next? 1. Continue the unit's activities as if nothing has happened 2. Arrange a unit meeting to discuss what has just happened 3. Refocus clients' negative comments to more positive topics 4. Have a private talk with the clients who cried and started to pace

4. Fluphenazine **Fluphenazine can be given intramuscularly every 2 to 3 weeks to clients who are unreliable about taking oral medications; it allows them to live in the community while keeping the disorder under control.

A client with a diagnosis of schizophrenia is discharged from the hospital. At home the client forgets to take the medication, is unable to function, and must be rehospitalized. What medication may be prescribed that can be administered on an outpatient basis every 2 to 3 weeks? 1. Lithium 2. Diazepam 3. Fluvoxamine 4. Fluphenazine

Realistic limits and controls are set -Setting realistic limits and controls makes the environment as emotionally nonthreatening as is realistically possible. All needs cannot be met; the person must learn how to cope with delaying gratification. It is not possible or realistic to meet all of a person's requests. Order in the environment is of less importance; providing a nonthreatening environment is the priority action.

A client with a generalized anxiety disorder is hospitalized. The nurse determines that an environment conducive to reducing emotional stress and providing psychological safety for this client is one in which:

Increasing tension in facial expression Having difficulty waiting to take turns during a group project Pacing in the hall Engaging in verbal abuse toward the nurse Pushing another client while waiting in line to the dining room

A client with a history of aggressive, violent behavior is admitted to the psychiatric unit involuntarily. The nurse, who understands the need to use deescalation approaches during the preassaultive stage of the violence cycle, monitors the client's behavior closely for progression of signs of impending violence. List these client behaviors in order of escalating aggression, from the lowest risk to the highest.

2.Increasing tension in facial expression 5.Having difficulty waiting to take turns during a group project 1.Pacing in the hall 3.Engaging in verbal abuse toward the nurse 4.Pushing another client while waiting in line to the dining room

A client with a history of aggressive, violent behavior is admitted to the psychiatric unit involuntarily. The nurse, who understands the need to use deescalation approaches during the preassaultive stage of the violence cycle, monitors the client's behavior closely for progression of signs of impending violence. List these client behaviors in order of escalating aggression, from the lowest risk to the highest. 1.Pacing in the hall 2.Increasing tension in facial expression 3.Engaging in verbal abuse toward the nurse 4.Pushing another client while waiting in line to the dining room 5.Having difficulty waiting to take turns during a group project

Intramuscular injections of thiamine -Thiamine is a coenzyme necessary for the production of energy from glucose. If thiamine is not present in adequate amounts, nerve activity is diminished and damage or degeneration of myelin sheaths occurs. A traditional phenothiazine is a neuroleptic antipsychotic that should not be prescribed because it is hepatotoxic. Antipsychotics are avoided; the use of these has a higher risk for toxic side effects in older or debilitated persons. Chlorpromazine, a neuroleptic, will not be used because it is severely toxic to the liver.

A client with a history of alcoholism is found to have Wernicke encephalopathy associated with Korsakoff syndrome. What does the nurse anticipate will be prescribed?

Marked loss of memory -Alcoholic clients have loss of memory and adapt to this by unconsciously filling in with false information areas that cannot be remembered.

A client with a history of chronic alcoholism is admitted to the mental health unit. What does the nurse identify as the cause of a client's use of confabulation?

Teaching relaxation

A client with a history of violence is becoming increasingly agitated. Which nursing intervention will most likely increase the risk of acting-out behavior?

"No, I don't see any bugs."

A client with alcohol dependence problem asks whether the nurse can see the bugs that are crawling on the bed. What is the nurse's initial reply?

Denial

A client with cancer is told by a health care provider that the cancer has metastasized to other organs and is untreatable. The client tells the nurse, "I think they made a mistake. I don't think I have cancer. I feel too good to be dying." Which stage of grief does the nurse conclude that the client is experiencing?

Ingesting adequate fluid and food with assistance A client in a vegetative state may not eat or drink without assistance; fluids and foods are basic physiologic needs that are necessary to prevent malnutrition and starvation; therefore the intake of adequate fluid and food is a priority short-term goal. The client is in total withdrawal; talking with peers, performing activities of daily living, and completing activities and assignments are not priority outcomes at this time.

A client with catatonic schizophrenia who is in a vegetative state is admitted to the psychiatric hospital. The nurse identifies short- and long-term outcomes in the client's clinical pathway. What is the priority short-term outcome of care that the client should be able to attain? Talking with peers Performing her own activities of daily living Completing unit activities and assignments Ingesting adequate fluid and food with assistance

Tardive dyskinesia Tardive dyskinesia occurs as a late and persistent extrapyramidal complication of long-term antipsychotic therapy. It is most often manifested by abnormal movements of the lips, tongue, and mouth. The other side effects are reversible with administration of an anticholinergic (e.g., benztropine [Cogentin]) or an antihistamine (e.g., diphenhydramine [Benadryl]) or cessation of the medication.

A client with chronic undifferentiated schizophrenia is receiving an antipsychotic medication. For which potentially irreversible extrapyramidal side effect should a nurse monitor the client? Torticollis Oculogyric crisis Tardive dyskinesia Pseudoparkinsonism

The blood level may not be sufficient to cause noticeable improvement for 2 to 4 weeks.

A client with depression is to be given fluoxetine (Prozac). What precaution should the nurse consider when initiating treatment with this drug?

3. The blood level may not be sufficient to cause noticeable improvement for 2 to 4 weeks. **Fluoxetine (Prozac) does not produce an immediate effect; nursing measures must be continued to reduce the risk for suicide

A client with depression is to be given fluoxetine (Prozac). What precaution should the nurse consider when initiating treatment with this drug? 1. It must be given with milk and crackers to avoid hyperacidity and discomfort. 2. Eating cheese or pickled herring or drinking wine may cause a hypertensive crisis. 3. The blood level may not be sufficient to cause noticeable improvement for 2 to 4 weeks. 4. The blood level should be checked weekly for 3 months to monitor for an appropriate level.

Intellectualization -Intellectualization is the avoidance of a painful emotion with the use of a rational explanation that removes the event from any personal significance.

A client with diabetes mellitus is able to discuss in detail the diabetic metabolic process while eating a piece of chocolate cake. What defense mechanism does the nurse identify when evaluating this behavior?

4. Intellectualization Rationale: 4. Intellectualization occurs when a painful emotion is avoided by means of a rational explanation that removes the event from any personal significance.

A client with diabetes mellitus is able to discuss in detail the diabetic metabolic process while eating a piece of chocolate cake. What defense mechanism does the nurse identify when evaluating this behavior? 1. Projection 2. Dissociation 3. Displacement 4. Intellectualization

4. "Work on identifying and developing coping strategies."

A client with generalized anxiety disorder says to the nurse, "What can I do to keep myself from overreacting to stress?" What is the best response by the nurse? 1. "Work on problem-solving skills." 2. "Improve your time-management skills." 3. "Ignore situations that you cannot change." 4. "Work on identifying and developing coping strategies."

Protecting the client against any suicidal impulses

A client with major depression that includes psychotic features tells the nurse, "All of my relatives have been killed because I've been sinful and need to be punished." What is the primary focus of nursing interventions?

Denying this activity may precipitate an increased level of anxiety.

A client with obsessive-compulsive disorder performs a specific ritual. Why should the nurse give the client time to perform the ritual?

The client eats the food provided on the hospital tray. Because the client was admitted while complaining that the food was poisoned, eating the food on the tray indicates that the client feels safe. Discussing discharge plans with the staff does not provide adequate behavioral assessment with which the nurse can evaluate reality testing. Questioning each medication when it is administered indicates that the client still does not completely trust the staff. Asking permission to make phone calls to the hospital administration seems to indicate that the client still does not trust the staff and is attempting to intimidate the staff by calling the administration.

A client with paranoid schizophrenia tells the nurse, "My neighbors are spying on me because they want to rob me and take money." While hospitalized, the client complains of being poisoned by the food and of being given the wrong medication. The nurse evaluates the client's response to medications and therapy. Which assessment finding leads the nurse to conclude that the client's reality testing has improved? The client eats the food provided on the hospital tray. The client discusses his discharge plans with the staff. The client questions each medication when it is administered. The client asks permission to make phone calls to the hospital administration

The client eats the food provided on the hospital tray.

A client with paranoid schizophrenia tells the nurse, "My neighbors are spying on me because they want to rob me and take money." While hospitalized, the client complains of being poisoned by the food and of being given the wrong medication. The nurse evaluates the client's response to medications and therapy. Which finding leads the nurse to conclude that the client's reality testing has improved?

Ziprasidone (Geodon) -Ziprasidone (Geodon) is a neuroleptic, which will reduce psychosis by affecting the action of both dopamine and serotonin.

A client with schizophrenia is actively psychotic, and a new medication regimen is prescribed. A student nurse asks the nurse, "Which of the medications will be the most helpful against the psychotic signs and symptoms?" What response should the nurse give?

Ziprasidone (Geodon) Ziprasidone (Geodon) is a neuroleptic, which will reduce psychosis by affecting the action of both dopamine and serotonin. Citalopram (Celexa) is a selective serotonin reuptake inhibitor antidepressant. Benztropine (Cogentin) is an anticholinergic. Acetaminophen with hydrocodone (Lortab) is an analgesic/opioid.

A client with schizophrenia is actively psychotic, and a new medication regimen is prescribed. A student nurse asks the primary nurse, "Which of the medications will be the most helpful against the psychotic signs and symptoms?" What response should the nurse give? Citalopram (Celexa) Ziprasidone (Geodon) Benztropine (Cogentin) Acetaminophen with hydrocodone (Lortab)

"I'll help you take your shower now." -The client is displaying a self-care deficit; stating the intention of helping the client shower is direct, does not require the client to make a decision, provides help, and meets the client's physiological and psychological needs. The client may or may not be capable of making a decision; if the client says no, the nurse will be confronted with a dilemma:

A client with schizophrenia is admitted to a psychiatric unit. The client is talking while walking in the hall, is unkempt, and obviously has not washed in several days. What should the nurse say when trying to help this client shower?

Play

A nurse is caring for a preschool-aged child with a history of physical and sexual abuse. What type of therapy will be the most advantageous for this child?

Passive range-of-motion exercises three times a day for effective joint health Waxy flexibility is an excessive and extended maintenance of posture that can lead to a variety of problems, including joint trauma. Passive range-of-motion exercises focus on the effective management of joint mechanics. Although aspiration precautions, documentation of intake and output, and staff role modeling may address issues experienced by a client with schizophrenia, passive range-of-motion exercises address waxy flexibility.

A client with schizophrenia is demonstrating waxy flexibility. Which intervention is the best way to manage the possible outcome of this behavior? Providing thickened liquids to minimize the risk of aspiration Documenting intake and output each shift to monitor hydration Reinforcing appropriate social boundaries through staff role modeling Passive range-of-motion exercises three times a day for effective joint health

Make the client more receptive to psychotherapy -Antipsychotic/neuroleptic medications help control anxiety, improve cognition, and decrease acting-out behavior, rendering the client better able to participate in therapy. Although the medication may keep the client quiet and relaxed, control the client's behavior and reduce stress, or prevent the need for restraints, none of these is the primary purpose of administration.

A client with schizophrenia is started on an antipsychotic/neuroleptic medication. The nurse explains to a family member that this drug primarily is used to:

4. Make the client more receptive to psychotherapy

A client with schizophrenia is started on an antipsychotic/neuroleptic medication. The nurse explains to a family member that this drug primarily is used to: 1. Keep the client quiet and relaxed 2. Control the client's behavior and reduce stress 3. Reduce the client's need for physical restraints 4. Make the client more receptive to psychotherapy

Take a dose as soon as possible, up to 2 hours before the next dose. Taking a dose as soon as possible is the advised intervention when a dose is missed; interruption of the medication may precipitate signs of withdrawal such as anxiety and tachycardia. Taking 2 pills at the next regularly scheduled dose will provide an excessive amount of the medication at one time. Notifying the health care provider about the missed dose immediately is unnecessary. Skipping a dose is not advised if the next regularly scheduled dose is due within 2 hour

A client with schizophrenia is taking benztropine (Cogentin) in conjunction with an antipsychotic. The client tells a nurse, "Sometimes I forget to take the Cogentin." What should the nurse teach the client to do if this happens again? Take 2 pills at the next regularly scheduled dose. Notify the health care provider about the missed dose immediately. Take a dose as soon as possible, up to 2 hours before the next dose. Skip the dose, then take the next regularly scheduled dose 2 hours early.

"It's time for you to go for a walk now."

A client with schizophrenia plans an activity schedule with the help of the treatment team. A written copy is posted in the client's room. What should the nurse say when it is time for the client to go for a walk?

"Try to ignore the voices." -Clients can sometimes learn to push auditory hallucinations aside, particularly within the framework of a trusting relationship; it may provide the client with a sense of power to manage the voices.

A client with schizophrenia reports having ongoing auditory hallucinations that he describes as "voices telling me that I'm a bad person" to the nurse. What is the best response by the nurse?

Loose association

A client with schizophrenia says to the nurse, "I've been here 5 days. There are 5 players on a basketball team. I like to play the piano." How should the nurse document this cognitive disorder?

Loose association These ideas are not well connected and there is no clear train of thought. This is an example of loose association. Word salad is incoherent expressions containing jumbled words. This client's thoughts are coherent but not connected. Thought blocking occurs when the client loses the train of thinking and ideas are not completed. Each of the client's thoughts is complete but not linked to the next thought. These statements are reality based and not reflective of delusional thinking

A client with schizophrenia says to the nurse, "I've been here 5 days. There are five players on a basketball team. I like to play the piano." How should the nurse document this cognitive disorder? Word salad Loose association Thought blocking Delusional thinking

3 More interest is shown in unit activities 5 The client performs activities of daily living independently

A client with schizophrenia who has type II (negative) symptoms is prescribed risperidone (Risperdal). Which outcomes indicate that the medication has minimized these symptoms? (Select all that apply.)

3.More interest is shown in unit activities. 5.The client performs activities of daily living independently

A client with schizophrenia who has type II (negative) symptoms is prescribed risperidone (Risperdal). Which outcomes indicate that the medication has minimized these symptoms? (Select all that apply.) 1.There is less agitation. 2.There are fewer delusions. 3.More interest is shown in unit activities. 4.The client reports that the hallucinations have stopped. 5.The client performs activities of daily living independently

3. More interest is shown in unit activities. 5. The client performs activities of daily living independently.

A client with schizophrenia who has type II (negative) symptoms is prescribed risperidone (Risperdal). Which outcomes indicate that the medication has minimized these symptoms? (Select all that apply.) 1.There is less agitation. 2.There are fewer delusions. 3. More interest is shown in unit activities. 4. The client reports that the hallucinations have stopped. 5. The client performs activities of daily living independently.

Invite the client to play a game of cards or board game. Activities that require limited interpersonal contact are less threatening. Individuals with schizophrenia, paranoid type, usually do not respond to an authoritarian approach because they do not trust others, particularly those who act in an aggressive manner. Group activities require interaction with other people, which is threatening to individuals with paranoid feelings.

A client with schizophrenia, paranoid type, is delusional, withdrawn, and negativistic. The nurse should plan to: Invite the client to play a game of cards or board game. Explain to the client the benefits of joining a group activity. Encourage the client to become involved in group activities. Mention to the client that the psychiatrist has ordered increased activity.

Rationalization -The attempt to justify a behavior by giving it acceptable motives is an example of rationalization

A client with the diagnosis of alcoholism explains to the nurse that alcohol has a calming effect and states, "I function better when I'm drinking than when I'm sober." What defense mechanism does the nurse identify?

Talking with the nurse several times during the day -Involving the client in a one-on-one conversation provides individualized, low-anxiety-producing attention and gives the message that the client is important, which supports self-esteem.

A client with the diagnosis of bipolar disorder, depressive episode, has been hospitalized on a psychiatric unit for 1 week. What is the most appropriate activity for this client?

Dehydration -The nurse should be alert for dehydration caused by fluid loss through vomiting in the binge-purge cycle.

A client with the diagnosis of bulimia nervosa, purging type, is admitted to the mental health unit after an acute episode of bingeing. Which clinical manifestation is most important for the nurse to evaluate?

Walking around the facility with a nurse -Walking around the facility with a nurse does not involve an element of competition and still allows the client to channel excess energy safely.

A client with the diagnosis of manic episode of bipolar disorder attends a mental health day treatment program. What supervised activity will be most therapeutic for this client during the early phase of treatment?

Tardive dyskinesia

A client with the diagnosis of schizophrenia is given one of the antipsychotic drugs. The nurse understands that antipsychotic drugs can cause extrapyramidal side effects. Which effect is cause for the greatest concern?

2.Tardive dyskinesia

A client with the diagnosis of schizophrenia is given one of the antipsychotic drugs. The nurse understands that antipsychotic drugs can cause extrapyramidal side effects. Which effect is cause for the greatest concern? 1.Akathisia 2.Tardive dyskinesia 3.Parkinsonian syndrome 4.Acute dystonic reaction

Tardive dyskinesia Tardive dyskinesia, an extrapyramidal response characterized by vermicular movements and protrusion of the tongue, chewing and puckering movements of the mouth, and puffing of the cheeks, is often irreversible, even when the antipsychotic medication is withdrawn. Akathisia, motor restlessness, usually can be treated with antiparkinsonian or anticholinergic drugs while the antipsychotic medication is continued. Parkinsonian syndrome (a disorder featuring signs and symptoms of Parkinson disease such as resting tremors, muscle weakness, reduced movement, and festinating gait) can usually be treated with antiparkinsonian or anticholinergic drugs while the antipsychotic medication is continued. Dystonia, impairment of muscle tonus, can usually be treated with antiparkinsonian or anticholinergic drugs while the antipsychotic medication is continued.

A client with the diagnosis of schizophrenia is given one of the antipsychotic drugs. The nurse understands that antipsychotic drugs can cause extrapyramidal side effects. Which effect is cause for the greatest concern? Akathisia Tardive dyskinesia Parkinsonian syndrome Acute dystonic reaction

Undoing

A client's hands are raw and bloody from a ritual involving frequent handwashing. Which defense mechanism does the nurse identify?

Having a staff member sit with the client in a quiet area during mealtimes -By sitting with the client during mealtimes the nurse can evaluate how much the client is eating; this encourages the client to eat and begins the construction of a trusting relationship. Fewer distractions may help the client focus on eating.

A client with the diagnosis of schizophrenia refuses to eat meals. Which nursing action is most beneficial for this client?

Having a staff member sit with the client in a quiet area during mealtimes By sitting with the client during mealtimes the nurse can evaluate how much the client is eating; this encourages the client to eat and begins the construction of a trusting relationship. Fewer distractions may help the client focus on eating. The client will not follow directions to eat because of the nature of the illness. Explaining the importance of eating and allowing the client to eat when ready are both unrealistic and will not ensure adequate intake.

A client with the diagnosis of schizophrenia refuses to eat meals. Which nursing action is most beneficial for this client? Directing the client repeatedly to eat the food Explaining to the client the importance of eating Waiting and allowing the client to eat whenever the client is ready Having a staff member sit with the client in a quiet area during mealtimes

Say, "You sound frightened. Is there something else I can give you to take your medication with? The response "You sound frightened" reflects the client's feelings and avoids focusing on the delusion; following up with "Is there something else I can give you to take your medication with?" encourages the client to take the medication. The response "The juice is not poisoned" will not change the client's feelings because the belief is real to the client. Pouring the client a glass of juice from a full pitcher will not change the client's feelings because the other pitcher also may be perceived as poisoned. Taking a drink of the juice to show the client that it is safe will not change the client's feelings; the client will believe that the nurse was not really drinking the juice.

A client with the diagnosis of schizophrenia watches the nurse pour juice for the morning medication from an almost-empty pitcher and screams, "That juice is no good! It's poisoned." What is the most therapeutic response by the nurse? Assure the client, "The juice is not poisoned." Pour the client a glass of juice from a full pitcher. Take a drink of the juice to show the client that it is safe. Say, "You sound frightened. Is there something else I can give you to take your medication with?

By setting realistic limits on the client's maladaptive behavior

A client's methods of coping are maladaptive. How can the nurse best help the client develop healthier coping mechanisms?

Provide the client with sunscreen

A client with the diagnosis of schizophrenia, paranoid type, has been receiving a phenothiazine drug. The daycare center is planning a fishing trip. It is important that the nurse:

1.Provide the client with sunscreen

A client with the diagnosis of schizophrenia, paranoid type, has been receiving a phenothiazine drug. The daycare center is planning a fishing trip. It is important that the nurse: 1.Provide the client with sunscreen 2.Caution the client to limit exertion during the trip 3.Give the client an extra dose of medication to take after lunch 4.Take the client's blood pressure before allowing him to participate in the outing

1. Provide the client with sunscreen

A client with the diagnosis of schizophrenia, paranoid type, has been receiving a phenothiazine drug. The daycare center is planning a fishing trip. It is important that the nurse: 1. Provide the client with sunscreen 2. Caution the client to limit exertion during the trip 3. Give the client an extra dose of medication to take after lunch 4. Take the client's blood pressure before allowing him to participate in the outing

Decreased control of the diabetes

A client with type 1 diabetes is found to have a psychosis and is to receive haloperidol (Haldol). Which response should a nurse anticipate with this drug combination?

3.Decreased control of the diabetes

A client with type 1 diabetes is found to have a psychosis and is to receive haloperidol (Haldol). Which response should a nurse anticipate with this drug combination? 1.Depressed respiration 2.Intensified action of both drugs 3.Decreased control of the diabetes 4.Increased danger of extrapyramidal side effects

Telling the father that it is nothing he has done and sharing the nurse's observations of the child

A clinic nurse observes a 2-year-old girl sitting alone, rocking and staring at a small, shiny top that she is spinning. Later the father relates his concerns, stating, "She pushes me away. She doesn't speak, and she only shows feelings when I take her top away. Is it something I've done?" What is the most therapeutic initial response by the nurse?

1. Impulsiveness 2. Lability of mood 5. Self-destructive behavior

A college student is brought to the mental health clinic by his parents. The diagnosis is borderline personality disorder. Which factors in the client's history support this diagnosis? (Select all that apply.) 1. Impulsiveness 2. Lability of mood 3. Ritualistic behavior 4. Psychomotor retardation 5. Self-destructive behavior

Identity versus role confusion -The client is demonstrating a search for self and has not resolved the developmental conflict of adolescence, identity versus role confusion.

A college student visits the health center and describes anxiety about having to declare an academic major. What developmental conflict, according to Erikson, is this client still attempting to resolve?

"It can be frightening to feel that way."

A confused hallucinating client says, "My arms are turning to stone." What is the most therapeutic response by the nurse?

Role experimentations -Adolescents learn about who they are by assuming and experiencing a variety of roles; experimentation results in the retention or rejection of behavior and roles.

A constructive but lengthy method of confronting the stress of adolescence and preventing a negative and unhealthy developmental outcome is:

"A neutral atmosphere facilitates the working through of conflicts." -These clients can better work through their underlying problems when the environment is structured, demands are reduced, and the routine is simple

A daycare environment is recommended for a client with incapacitating behaviors resulting from an obsessive-compulsive personality disorder. The client's partner asks the nurse why this approach is necessary. What is the best response by the nurse?

An illusion -An illusion is a misperception or misinterpretation of an actual external stimulus. A delusion is a false belief that cannot be changed even by evidence; it is associated with psychosis. A hallucination results from an imaginary, not real, stimulus. An idea of reference is a belief that others are talking about the person. Topics

A delirious client sees a design on the wallpaper and perceives it as an animal. How should a nurse communicate what the client perceived in the change-of-shift report?

Projection

A delusional client is actively hallucinating and worried about being stalked by a terrorist group. What defense mechanism does the nurse identify as the most prominent in this situation?

Stay with the client during meals -Active support is demonstrated when the nurse sits with the client during meals.

A depressed client has been sitting alone in a chair most of the day and displays no interest in eating. How should the nurse plan to meet this client's nutritional needs?

"You must have been upset to try to take your life."

A depressed client is brought to the emergency department after taking an overdose of a sedative. After lavage the client says, "Let me die. I'm no good." What is the most appropriate response by the nurse?

Restricting the client's access to the bedroom

A depressed client often sleeps past the expected time of awakening and spends excessive time resting and sleeping. Which nursing intervention is appropriate for this client?

1. Restricting the client's access to the bedroom **The goal is 6 to 8 hours of rest at night; too much time spent sleeping in the daytime will defeat the goal of adequate rest at night.

A depressed client often sleeps past the expected time of awakening and spends excessive time resting and sleeping. Which nursing intervention is appropriate for this client? 1. Restricting the client's access to the bedroom 2. Offering the client a series of relaxation tapes 3. Rescheduling the client's bedtime to an earlier hour 4. Suggesting that the client exercise before going to bed

"I care about you. What are some foods you especially like?" -The statement "I care about you. What are some foods you especially like?" is a direct response to the client's concern and permits some exploration of food choices

A depressed older client has not been eating well since her admission to the hospital. The client repeatedly states, "No one cares." What is the most appropriate response by the nurse?

Echolalia

A disturbed client starts to repeat phrases that others have just said. How should the nurse document this speech?

He has some feelings of self-worth.

A disturbed client who has been out of touch with reality has been hospitalized for several weeks. One day the nurse notes that the client's hair is dirty and asks whether the client wants to wash it. The client answers, "Yes, and I'd like to shower and change my clothes, too." What can the nurse conclude about the client in relation to this response?

1.He has some feelings of self-worth.

A disturbed client who has been out of touch with reality has been hospitalized for several weeks. One day the nurse notes that the client's hair is dirty and asks whether the client wants to wash it. The client answers, "Yes, and I'd like to shower and change my clothes, too." What can the nurse conclude about the client in relation to this response? 1.He has some feelings of self-worth. 2.He is open to suggestions from others. 3.He may be entering a hyperactive phase. 4.He has a need for social reassurance from others

Having the client sit with a staff member in whom he trusts -The client needs someone with whom he has a working and trusting relationship; this individual must observe, protect, anticipate, and prevent the client from acting out destructive impulses.

A disturbed male client, unprovoked, attacks another client. A short-term initial plan for this client should include:

3 "I'd like to end it all with sleeping pills." 4 "The voices say I should kill all prostitutes."

A family member brings a relative to the local community hospital because the relative "has been acting strange." Which statements meet involuntary hospitalization criteria? (Select all that apply.)

3. "I'd like to end it all with sleeping pills." 4 . "The voices say I should kill all prostitutes." **Rationale: The statement about ending it all is a suicide threat; it is a direct expression of intent without action. Likewise, the threat to harm others must be heeded.

A family member brings a relative to the local community hospital because the relative "has been acting strange." Which statements meet involuntary hospitalization criteria? (Select all that apply.) 1. "I cry all the time; I'm just so sad." 2 . "Since I retired I've been so depressed." 3. "I'd like to end it all with sleeping pills." 4 . "The voices say I should kill all prostitutes." 5 . "My boss makes me so angry—he's always picking on me."

Countertransference

A female nurse has been caring for a depressed 75-year-old woman who reminds her of her grandmother. The nurse spends extra time with her every day and brings her home-baked cookies. The nurse's behavior reflects:

3."I would like to end it all with sleeping pills." 4."Voices say it is okay for me to kill all prostitutes." Rationale: 3. This statement indicates a suicide threat; it is a direct expression of intent but without action. 4. The threat to harm others must be heeded; the client must be protected from self harm as well as harming others.

A family member brings a relative to the local community hospital because the relative "has been acting strange." Which statements meet involuntary hospitalization criteria? Select all that apply. 1."I cry all the time, I am so sad." 2."Since I retired I have been so depressed." 3."I would like to end it all with sleeping pills." 4."Voices say it is okay for me to kill all prostitutes." 5."My boss makes me so angry by always picking on me."

Battery -Battery is the intentional touching of one person by another without permission of the person being touched.

A female client in the terminal stage of cancer is admitted to the hospital in severe pain. The client refuses the prescribed intramuscular analgesic for pain because it puts her to sleep and she wants to be awake. One day, despite the client's objection, a nurse administers the pain medication saying, "You know that this will make you more comfortable." The nurse in this situation could be charged with:

Dependence versus independence

A female client who is severely incapacitated by obsessive-compulsive behavior has been admitted to the mental health hospital. The client's compulsive ritual involves changing her clothing 8 to 12 times a day. She continually asks the nurse for advice regarding her problems but then ignores it. This is an example of the conflict of:

"Tell me more about this."

A female client with acute schizophrenia tells the nurse, "Everyone hates me." What is the best response by the nurse?

4. Countertransference **With countertransference the professional provider of care exhibits an emotional reaction to a client based on a previous relationship or on unconscious needs or conflicts.

A female nurse has been caring for a depressed 75-year-old woman who reminds her of her grandmother. The nurse spends extra time with her every day and brings her home-baked cookies. The nurse's behavior reflects: 1. Affiliation 2. Displacement 3. Compensation 4. Countertransference

A nurse in a community therapeutic recreation program is working with a client with dysthymia. The treatment plan suggests group activities when possible for this client. What is the priority rationale for this intervention?

A group can offer increased support.

Observing the clients' abilities to cope with a more complex society

A group of clients from a psychiatric unit, accompanied by staff members, are going to a professional baseball game. The purpose of visits into the community under the supervision of staff members is:

3. Observing the clients' abilities to cope with a more complex society

A group of clients from a psychiatric unit, accompanied by staff members, are going to a professional baseball game. The purpose of visits into the community under the supervision of staff members is: 1. Helping clients adjust to stressors in the community 2. Helping clients return to reality under controlled conditions 3. Observing the clients' abilities to cope with a more complex society 4. Broadening the clients' experiences by providing exposure to cultural activities

Management of manic episodes of bipolar disorder

A health care provider prescribes divalproex (Depakote). What does the nurse consider an appropriate indication for the use of this drug?

3.Management of manic episodes of bipolar disorder

A health care provider prescribes divalproex (Depakote). What does the nurse consider an appropriate indication for the use of this drug? 1.Control of acute agitation of schizophrenia 2.Treatment of the agitated phase of a paranoid state 3.Management of manic episodes of bipolar disorder 4.Modification of the depressive phase of major depression

Staying in the sun

A health care provider prescribes haloperidol (Haldol) for a client. What should the nurse teach the client to avoid while taking this medication?

2.Staying in the sun

A health care provider prescribes haloperidol (Haldol) for a client. What should the nurse teach the client to avoid while taking this medication? 1.Driving at night 2.Staying in the sun 3.Ingesting aged cheeses 4.Taking medications containing aspirin

2. Staying in the sun **Haloperidol (Haldol) causes photosensitivity. Severe sunburn may occur on exposure to the sun.

A health care provider prescribes haloperidol (Haldol) for a client. What should the nurse teach the client to avoid while taking this medication? 1. Driving at night 2. Staying in the sun 3. Ingesting aged cheeses 4. Taking medications containing aspirin

Hypochondriac disorder -Preoccupation with fears of getting or having a serious disease is called hypochondriasis. The condition usually exists for 6 months or longer, persists despite negative medical tests and reassurance, and results in social or occupational impairment. Conversion disorder is characterized by the presence of one or more symptoms related to a neurological problem that has no organic cause. Somatization disorder is characterized by the reporting of many physical problems by the client, usually beginning before age 30; physical problems may include pain, gastrointestinal symptoms, sexual or reproductive problems, and at least one symptom that suggests a neurological disorder. Body dysmorphic disorder is characterized by preoccupation with some imagined defect in appearance that causes marked distress and significant impairment in social and occupational function.

A health care provider refers a 52-year-old man to the mental health clinic. The history reveals that the man lost his wife to colon cancer 6 months ago and that since that time he has seen his health care provider seven times with the concern that he has colon cancer. All tests have had negative results. Recently the client stopped seeing friends, dropped his hobbies, and stayed home to rest. Which disorder should the nurse identify as consistent with the client's preoccupation with the fear of having a serious disease?

3. Hypochondriac disorder **Preoccupation with fears of getting or having a serious disease is called hypochondriasis

A health care provider refers a 52-year-old man to the mental health clinic. The history reveals that the man lost his wife to colon cancer 6 months ago and that since that time he has seen his health care provider seven times with the concern that he has colon cancer. All tests have had negative results. Recently the client stopped seeing friends, dropped his hobbies, and stayed home to rest. Which disorder should the nurse identify as consistent with the client's preoccupation with the fear of having a serious disease? 1. Conversion disorder 2. Somatization disorder 3. Hypochondriac disorder 4. Body dysmorphic disorder

Keep the client under closer observation - As the client's motivation and energy return, the likelihood that suicidal ideation will be acted out increases.

A hospitalized, depressed, suicidal client has been taking a mood-elevating medication for several weeks. The client's energy is returning and the client no longer talks about suicide. What should the nurse do in response to this client's behavior?

Recognizing that PRN orders for restraints are unacceptable

A health care provider writes a prescription of "Restraints PRN" for a client who has a history of violent behavior. What is the nurse's responsibility in regard to this order?

2.Recognizing that PRN orders for restraints are unacceptable

A health care provider writes a prescription of "Restraints PRN" for a client who has a history of violent behavior. What is the nurse's responsibility in regard to this order? 1.Asking that the order indicate the type of restraint 2.Recognizing that PRN orders for restraints are unacceptable 3.Implementing the restraint order when the client begins to act out 4.Ensuring that the entire staff is aware of the order for the restraints

2. Recognizing that PRN orders for restraints are unacceptable

A health care provider writes a prescription of "Restraints PRN" for a client who has a history of violent behavior. What is the nurse's responsibility in regard to this order? 1. Asking that the order indicate the type of restraint 2. Recognizing that PRN orders for restraints are unacceptable 3. Implementing the restraint order when the client begins to act out 4. Ensuring that the entire staff is aware of the order for the restraints

Change the child's bed while he changes his pajamas

A hospitalized 7-year-old boy wakes up crying because he has wet his bed. It is most appropriate for the nurse to:

2. Change the child's bed while he changes his pajamas **Changing the child's bed while the child changes his pajamas will not call attention to the accident and will minimize the child's embarrassment.

A hospitalized 7-year-old boy wakes up crying because he has wet his bed. It is most appropriate for the nurse to: 1. Allow the child to change his bed and pajamas 2. Change the child's bed while he changes his pajamas 3. Take the child to the bathroom and change his pajamas 4. Remind the child to call the nurse next time to avoid the need to change his pajamas

"I see that you're worried. We're using medication to ease your wife's discomfort." -Recognizing the spouse's feelings and giving simple factual information help to allay anxiety.

A husband is upset that his wife's alcohol withdrawal delirium has persisted for a second day. What is the most appropriate initial response by the nurse?

Talking with the child about the importance of using a seat belt

A hyperactive 9-year-old child with a history of attention deficit-hyperactivity disorder is admitted for observation after a motor vehicle collision. On what should nursing actions be focused when the nurse is teaching about personal safety?

Presenting a united, consistent staff approach

A male client with a diagnosis of antisocial personality disorder is admitted to the mental health hospital. What is the priority nursing intervention?

"Everyone has a bed. This one is yours." -A matter-of-fact approach helps avoid a cycle in which the nurse expresses concern to a client who feels unworthy, which increases feelings of unworthiness

A male client with the diagnosis of a bipolar disorder, depressed episode, is found lying on the floor in his room in the psychiatric unit. He states, "I don't deserve a comfortable bed; give it to someone else." The best response response by the nurse is:

By telling the client that the behavior is unacceptable and to stop -Exposing the genitals and masturbating in a public place are unacceptable behaviors. Unacceptable behavior should be pointed out to the client and the client should be instructed to stop.

A male client with the diagnosis of pedophilia is admitted to the psychiatric hospital because of repeated episodes of exhibitionism. In the recreation room the client exposes himself to a nurse and begins to masturbate. How should the nurse respond?

Compensation -By developing skills in one area, the individual compensates for a real or imagined deficiency in another, thereby maintaining a positive self-image.

A male college student who is smaller than average and unable to participate in sports becomes the life of the party and a stylish dresser. What defense mechanism does the nurse determine that the client is using?

3.Compensation

A male college student who is smaller than average and unable to participate in sports becomes the life of the party and a stylish dresser. What defense mechanism does the nurse determine that the client is using? 1.Introjection 2.Sublimation 3.Compensation 4.Reaction formation

3. Compensation **By developing skills in one area, the individual compensates for a real or imagined deficiency in another, thereby maintaining a positive self-image.

A male college student who is smaller than average and unable to participate in sports becomes the life of the party and a stylish dresser. What defense mechanism does the nurse determine that the client is using? 1. Introjection 2. Sublimation 3. Compensation 4. Reaction formation

Compensation

A male college student who is smaller than average and unable to participate in sports becomes the life of the party and a stylish dresser. What defense mechanism should the nurse determine that the client is using? Introjection Sublimation Compensation Reaction formation

Displacement -Displacement is the discharging of pent-up feelings on a less threatening object, in this case the locker door.

A male long-distance jumper improves his distance by 3½ inches (7 cm) and earns the praise of his coach, but on another day, when he does not reach his mark, he forcefully kicks the door of his locker. What defense mechanism does his outburst demonstrate?

2 "How do you feel about having a male nurse?"

A male nurse is caring for a client. The client states, "You know, I've never had a male nurse before." What is the best reply by the nurse?

Grandeur

A man with bipolar disorder, manic episode, has been traveling around the country, dating multiple women, and buying his dates expensive gifts. He is admitted to the hospital when he becomes exhausted and runs out of money. The nurse anticipates that during a manic episode the client is most likely experiencing feelings of:

Allowing the client to externalize her feelings, especially anger, in a safe manner

A middle-age female client who has lost 20 lb over the last 2 months cries easily, sleeps poorly, and refuses to participate in any family or social activities that she previously enjoyed. What is the most important nursing intervention?

Suppression

A mother and her 5-year-old daughter have been referred to a child advocacy center for a forensic pediatric sexual examination. Before the child is examined or interviewed, the mother gives a detailed history, relaying her suspicion that the child's maternal grandfather sexually assaulted her. As the interview progresses, the mother suddenly says, "My father sexually molested me when I was a child, but I try not to think about it." What defense mechanism does the nurse recognize that the mother's statement demonstrates?

2.Suppression

A mother and her 5-year-old daughter have been referred to a child advocacy center for a forensic pediatric sexual examination. Before the child is examined or interviewed, the mother gives a detailed history, relaying her suspicion that the child's maternal grandfather sexually assaulted her. As the interview progresses, the mother suddenly says, "My father sexually molested me when I was a child, but I try not to think about it." What defense mechanism does the nurse recognize that the mother's statement demonstrates? 1.Introjection 2.Suppression 3.Passive aggression 4.Reaction formation

Shorten the rest of the story -Shortening the story nonjudgmentally limits the activity while supporting the child's self-esteem; the child with ADHD cannot control his inattention and hyperactivity. The mother should select activities that are more interactive or interesting for the child to engage his attention.

A mother of a 6-year-old boy with the diagnosis of attention deficit-hyperactivity disorder (ADHD) tells the nurse that when she is reading storybooks to her son, about halfway through the story he becomes distracted, fidgets, and stops paying attention. The nurse suggests that the mother:

Say, "I'll be back in 15 minutes, and then we can talk."

A nurse enters the room of an agitated, angry client to administer the prescribed antipsychotic medication. The client shouts, "Get out of here!" The nurse's best approach is to:

Asking the client whether he is hearing voices Because the client is newly admitted, the nurse needs to conduct a thorough assessment before intervening. Encouraging the client to engage in unit activities may eventually be done but is not the priority. Telling the client that the voices he is hearing are not real assumes that the client is hallucinating. The client's behavior does not indicate the need for extra medication at this time. Some clients with schizophrenia have hallucinations throughout their lives.

A newly admitted male client with schizophrenia appears to be responding to internal stimuli when laughing and talking to himself. What is the best initial response by the nurse? Asking the client whether he is hearing voices Encouraging the client to engage in unit activities Telling the client that the voices he is hearing are not real Giving the client his prescribed PRN antipsychotic medication

Unintentional tremor

A nurse administers an antipsychotic medication to a client. For which common manageable side effect should the nurse evaluate the client?

4.Unintentional tremor

A nurse administers an antipsychotic medication to a client. For which common manageable side effect should the nurse evaluate the client? 1.Jaundice 2.Melanocytosis 3.Drooping eyelids 4.Unintentional tremor

Makes the client more amenable to psychotherapy

A nurse administers prescribed anxiolytics to clients with severe emotional disorders. What is the goal of this treatment?

Respecting the client's need for social isolation

A nurse begins a therapeutic relationship with a client with the diagnosis of schizotypal personality disorder. What is the best initial nursing action?

Initiative

A nurse concludes that a 6-year-old child who has attained an acceptable level of psychosocial development has achieved Erikson's developmental conflicts related to trust, autonomy, and:

Perform a relaxation exercise

A nurse concludes that a client has successfully achieved the long-term goal of mobilizing effective coping responses when the client states that when he feels himself getting anxious he will:

Directing pent-up emotions at someone other than the primary source

A nurse concludes that a client is using displacement. Which behavior has the nurse identified?

Directing pent-up emotions at someone other than the primary source -When acting out against the primary source of anxiety creates even further anxiety or danger, the individual may use displacement to express feelings toward a "safer" person or object

A nurse concludes that a client is using displacement. Which behavior has the nurse identified?

4. Directing pent-up emotions at someone other than the primary source

A nurse concludes that a client is using displacement. Which behavior has the nurse identified? 1. Ignoring unpleasant aspects of reality 2. Resisting any demands made by others 3. Using imaginative activity to escape reality 4. Directing pent-up emotions at someone other than the primary source

Defense against anxiety -Withdrawal provides a temporary defense against anxiety because it limits contact with reality and reduces the client's world.

A nurse concludes that a client's withdrawn behavior may temporarily provide a:

1 Clarifying

A nurse counseling a female client on the inpatient psychiatric unit responds to a statement made by the woman by stating, "I'm confused about exactly what is upsetting you. Would you go over that again, please?" The nurse is using:

4. "You say you're not a good parent, but you were effective when you were talking with your son today."

A nurse determines that confrontation is an appropriate tool for use with a client. What is an example of therapeutic confrontation? 1. "I find that hard to believe." 2. "I noticed that you're not wearing any makeup today." 3. "You feel frustrated because you think your mother doesn't understand you." 4. "You say you're not a good parent, but you were effective when you were talking with your son today."

Foster changes in behavior

A nurse encourages a client to attend Alcoholics Anonymous (AA) meetings after discharge. What do self-help groups such as AA help their members do?

3. Foster changes in behavior

A nurse encourages a client to attend Alcoholics Anonymous (AA) meetings after discharge. What do self-help groups such as AA help their members do? 1. Set long-term goals 2. Limit excessive drinking 3. Foster changes in behavior 4. Identify underlying causes of behavior

Support

A nurse encourages a client to join a self-help group after being discharged from a mental health facility. What is the purpose of having people work in a group?

A group can offer increased support.

A nurse in a community therapeutic recreation program is working with a client with dysthymia. The treatment plan suggests group activities when possible for this client. What is the priority rationale for this intervention?

Imagination is used to fill in memory gaps

A nurse in the mental health clinic concludes that a client is using confabulation when:

2.Imagination is used to fill in memory gaps

A nurse in the mental health clinic concludes that a client is using confabulation when: 1.The flow of thoughts is interrupted 2.Imagination is used to fill in memory gaps 3.Speech flits from one topic to another with no apparent meaning 4.Connections between statements are so loose that only the speaker understands them

2. Imagination is used to fill in memory gaps **Using imagination to fill in memory gaps is the definition of confabulation; it is a defense mechanism used by people experiencing memory deficits.

A nurse in the mental health clinic concludes that a client is using confabulation when: 1. The flow of thoughts is interrupted 2. Imagination is used to fill in memory gaps 3. Speech flits from one topic to another with no apparent meaning 4. Connections between statements are so loose that only the speaker understands them

Drowsiness and dry mouth

A nurse is administering hydroxyzine (Vistaril) to a client. For which common side effects of this drug should the nurse monitor the client?

Drowsiness and dry mouth -This drug suppresses activity in key regions of the subcortical area of the central nervous system; it also has antihistaminic and anticholinergic effects

A nurse is administering hydroxyzine (Vistaril) to a client. For which common side effects of this drug should the nurse monitor the client?

It combats the extrapyramidal side effects of the other drug.

A nurse is administering medications to clients on a psychiatric unit. What does the nurse identify as the reason that so many psychiatric clients are given the drug benztropine (Cogentin) or trihexyphenidyl in conjunction with the phenothiazine derivatives neuroleptic medications?

3.It combats the extrapyramidal side effects of the other drug.

A nurse is administering medications to clients on a psychiatric unit. What does the nurse identify as the reason that so many psychiatric clients are given the drug benztropine (Cogentin) or trihexyphenidyl in conjunction with the phenothiazine derivatives neuroleptic medications? 1.It reduces postural hypotension. 2.It potentiates the effects of the other drug. 3.It combats the extrapyramidal side effects of the other drug. 4.It ameliorates the depression that may accompany schizophrenia

Apathy Flatness Apathy (indifference) is common among people with chronic schizophrenia because negative symptoms are more apparent. Flatness, with few extremes of emotion, is common among people with chronic schizophrenia because negative symptoms are more apparent. Extremes in emotions are not associated with chronic schizophrenia. Sadness is related more to mood disorders, such as a depressive episode of bipolar disorder or major depression. Hostility may be seen in some forms of schizophrenia, such as paranoid schizophrenia, but it is rarely seen in the chronic stages. Happiness and elation are associated with manic episodes of bipolar disorder, not chronic schizophrenia. Depression is related to mood disorders, such as a depressive episode of bipolar disorder or major depression.

A nurse is assessing a client with chronic schizophrenia. Which effects will the client most likely exhibit? Select all that apply. Apathy Sadness Flatness Hostility Happiness Depression

1 Meditation 2 Mental imagery 5 Deep-breathing exercises

A nurse is assigned to lead a relaxation group. Which techniques should the nurse incorporate? (Select all that apply.)

Confusion immediately after the treatment

A nurse is assisting with the administration of electroconvulsive therapy (ECT) to a severely depressed client. What side effect of the therapy should the nurse anticipate?

The co-worker may need help with grieving.

A nurse is aware that a co-worker's mother died 16 months ago. The co-worker cries every time someone says the word "mother" and when the mother's name is mentioned. What does the nurse conclude about this behavior?

Feeling comfortable with the nurse -Before therapy can begin, a trusting relationship must be developed. A client with major depression will not have the impetus or energy to investigate new leisure activities. Participating in small group activities is not appropriate initially; the client does not have the physical or emotional energy to interact with a small group of people. Initiating conversations about feelings will not be successful unless the client develops a trusting, comfortable relationship with the nurse.

A nurse is caring for a client with a bipolar disorder depressive episode. What should the nurse's objective for this client be?

Redirect the conversation with the nurse to physical symptoms

A nurse is caring for a client with a somatoform disorder. What should the nurse anticipate that this client will do?

Unconscious control of unacceptable feelings

A nurse is caring for a client with an obsessive-compulsive disorder. What is the basis for the obsessions and compulsions?

Assuring the client that the symptoms are part of the withdrawal syndrome

A nurse is caring for a client with the diagnosis of alcohol withdrawal delirium. Which action is most appropriate for the nurse to implement?

By providing a nonthreatening environment

A nurse is caring for a client with the diagnosis of schizophrenia, paranoid type. How should the nurse plan for the client's initial care?

Disordered thinking The schizophrenic individual has neurobiological changes that cause disorders in thought process and perceiving reality. Chronic confusion and disorientation are not usually associated with this disorder. Illogical thinking and impaired judgment are associated with schizophrenia. Individuals with the diagnosis of schizophrenia often have personal boundary difficulties. They lack a sense of where their bodies end in relation to where others begin. Loss of ego boundaries can result in depersonalization and derealization. Most clients with schizophrenic disorders are not violent.

A nurse is caring for a client with the diagnosis of schizophrenia. What is a common problem for clients with this diagnosis? Chronic confusion Disordered thinking Rigid personal boundaries Violence directed toward others

3 Repetitive activities 4 Self-injurious behaviors 5 Lack of communication with others -Perseveration (repetition of a behavior pattern) is commonly demonstrated by children with autism; this behavior provides comfort. Self-stimulation through injurious behavior is associated with autism. Children with autism have difficulty communicating or do not communicate at all with others. There may be unusual eating habits and food preferences, but lack of appetite is not associated with autism. Mood disorders are usually not associated with autism.

A nurse is caring for a group of children with the diagnosis of autism. Which signs and symptoms are associated with this disorder? (Select all that apply.)

An uncomplicated daily schedule

A nurse is caring for a group of depressed clients. What should the nurse attempt to provide?

Is too busy to take the time to eat

A nurse is caring for a hyperactive, manic client who exhibits flight of ideas and is not eating. What may be the reason why the client is not eating?

1. "I want him to get a second opinion." 2. "He shouldn't have gotten this because he doesn't smoke or drink." 3. "His grandchildren need to get to know him." 4. "All I do is cry, because I can't live without him." 5. "If he can't be cured, I just want him to be comfortable."

A nurse is caring for a man who has inoperable cancer of the pancreas. His wife is trying to cope with the diagnosis. Place the wife's statements in order as the woman progresses through the grieving process, from the first stage to the last:

1."I want him to get a second opinion." 2."He shouldn't have gotten this because he doesn't smoke or drink." 3."His grandchildren need to get to know him." 4."All I do is cry, because I can't live without him." 5."If he can't be cured, I just want him to be comfortable." Tip: Denial,Anger,Bargaining,Depression,Acceptance

A nurse is caring for a man who has inoperable cancer of the pancreas. His wife is trying to cope with the diagnosis. Place the wife's statements in order as the woman progresses through the grieving process, from the first stage to the last: 1."He shouldn't have gotten this because he doesn't smoke or drink." 2."I want him to get a second opinion." 3."All I do is cry, because I can't live without him." 4."If he can't be cured, I just want him to be comfortable." 5."His grandchildren need to get to know him."

3 Increasing food intake gradually 4 Limiting mealtime to half an hour 5 Providing privileges for goal achievement

A nurse is caring for an adolescent who has anorexia nervosa. The nutritional treatment of anorexia is composed of several guidelines. Which guidelines should the nurse emphasize? (Select all that apply.)

3. Increasing food intake gradually 4. Limiting mealtime to half an hour 5. Providing privileges for goal achievement

A nurse is caring for an adolescent who has anorexia nervosa. The nutritional treatment of anorexia is composed of several guidelines. Which guidelines should the nurse emphasize? (Select all that apply.) 1. Increasing high-fiber foods 2. Eating just three meals a day 3. Increasing food intake gradually 4. Limiting mealtime to half an hour 5. Providing privileges for goal achievement

1 Resistance to change 2 Inability to recognize familiar objects 4 Inability to concentrate on new activities or interests 5 Tendency to dwell on the past and ignore the present

A nurse is caring for an older adult with the diagnosis of dementia. Which manifestations are expected in this client? (Select all that apply.)

2 Client

A nurse is caring for clients who are undergoing therapy for dependence on alcohol. Which member of the health team has the primary responsibility for their rehabilitation?

1 Bipolar Disease 2 Paranoid schizophrenia -Individuals with manic-depressive illness may have psychotic episodes during which they are unable to perceive and respond to reality appropriately. Mania diminishes judgment and insight, which in turn reduces a client's ability to make decisions. Individuals with paranoid schizophrenia may have psychotic episodes during which they are unable to perceive and respond to reality appropriately. Paranoia makes a client overly suspicious, which diminishes judgment and insight. Individuals with narcissistic personality disorder are usually in contact with reality and able to make reasonable decisions.

A nurse is caring for clients with a variety of psychiatric illnesses. For which diagnoses is the establishment of a psychiatric advance directive (PAD) most beneficial? (Select all that apply.)

Are dependent on it

A nurse is caring for several clients who are going through withdrawal from alcohol. The primary reason for the ingestion of alcohol by clients with a history of alcohol abuse is that they:

4. To reduce the positive symptoms of psychosis **Antipsychotics are used to decrease positive signs and symptoms associated with psychoses, including hallucinations, delusions, paranoia, and disorganized speech.

A nurse is caring for several clients who have severe psychiatric disorders. What is the major reason that a health care provider prescribes an antipsychotic medication for these clients? 1. To improve judgment 2. To promote social skills 3. To diminish neurotic behavior 4. To reduce the positive symptoms of psychosis

Guilt

A nurse is caring for several clients with the diagnosis of bulimia nervosa. What primary feeling does the nurse anticipate that these clients experience after an episode of bingeing?

"Call the contact number we gave you if you have an emergency."

A nurse is discharging a client from the mental health unit who has been treated for major depression. Which statement is most therapeutic at this time?

Active membership in Alcoholics Anonymous

A nurse is discussing plans with a client who has decided to withdraw from alcohol. What should the nurse recommend as one of the most effective treatments for alcoholism?

2 years By 2 years of age the child should demonstrate an interest in others, communicate verbally, and possess the ability to learn from the environment. Before the age when these skills develop, autism is difficult to diagnose. Usually by 3 years the signs of autism become more profound.

A nurse is evaluating a child with suspected autism. At what age does the nurse determine that the signs of autism initially may be evident?

Intimacy versus isolation The major task of young adulthood is centered on human closeness and sexual fulfillment; lack of love results in isolation.

A nurse is evaluating a young adult for evidence of achievement of the age-related developmental stage set forth in Erikson's developmental theory. What developmental crisis is associated with this age group?

2.Intimacy versus isolation

A nurse is evaluating a young adult for evidence of achievement of the age-related developmental stage set forth in Erikson's developmental theory. What developmental crisis is associated with this age group? 1.Trust versus mistrust 2.Intimacy versus isolation 3.Industry versus inferiority 4.Generativity versus stagnation

2 Verbalizes difficulty identifying personal strengths 4 Acknowledges the effects of the addiction on the family 5 Addresses how the addiction has contributed to family distress

A nurse is in the process of developing a therapeutic relationship with a client who has an addiction problem. What client communication permits the nurse to conclude that they are making progress in the working stage of the relationship? (Select all that apply.)

3 Loneliness 4 Hopelessness

A nurse is interacting with a depressed, suicidal client. What themes in the client's conversation are of most concern to the nurse? (Select all that apply.)

Attends Alcoholics Anonymous meetings daily

A nurse is interviewing a client newly admitted to an outpatient program after withdrawal from alcohol. What behavior best indicates that the client has accepted that drinking is a problem?

"Those boys are so cute. I hope their room's next to mine!"

A nurse is interviewing an 8-year-old girl who has been admitted to the pediatric unit. Which statement by the child needs to be explored?

3."Those boys are so cute. I hope their room is next to mine!" 3. A 7-year-old child should be more concerned with same-gender relationships. A child demonstrating a strong attraction to opposite-gender relations should be questioned further regarding the possibility of sexual abuse.

A nurse is interviewing an 8-year-old girl who was admitted to the pediatric unit. Which statement by the child needs to be explored? 1."Wow! This place has bright colors." 2."Is my mother allowed to visit me tonight?" 3."Those boys are so cute. I hope their room is next to mine!" 4."I am scared about being here. Can you stay with me awhile?"

Ambivalence Ambivalence is the existence of two conflicting emotions, impulses, or desires. Double bind is two conflicting messages, not emotions, in a single communication. Loose associations are not two conflicting emotions but instead the loosening of connections between thoughts. Inappropriate affect is not two conflicting emotions but instead the inappropriate expression of emotions.

A nurse is monitoring a client with the diagnosis of schizophrenia who is experiencing opposing emotions simultaneously. When providing a change-of-shift report, how should the nurse refer to this emotional experience of the client? Double bind Ambivalence Loose association Inappropriate affect

By saying to the interrupting client, "I'll be back to talk with you after I orient this new client."

A nurse is orienting a new client to the unit when another client rushes down the hallway and asks the nurse to sit down to talk. The client requesting the nurse's attention is manipulative and uses acting-out behaviors when demands go unmet. How should the nurse intervene?

3 Boundary violations

A nurse is speaking with a client who was sexually abused as a child. The client does not know what constitutes inappropriate touch by another person. What issue will have to be addressed with this client?

4. Displacement is a defense mechanism in which one's pent-up feelings toward a threatening person are discharged on less-threatening others.

A nurse is teaching a class about child abuse. What defense mechanism most often used by the physically abusive individual should the nurse include? 1.Repression 2.Transference 3.Manipulation 4.Displacement

Chlorpromazine -Clients taking chlorpromazine should be instructed to stay out of the sun. Photosensitivity makes the skin more susceptible to burning. Photosensitivity is not a side effect of lithium, sertraline, or methylphenidate.

A nurse is teaching a client about side effects of medications. Which drug will cause a heightened skin reaction to sunlight?

4.Chlorpromazine

A nurse is teaching a client about side effects of medications. Which drug will cause a heightened skin reaction to sunlight? 1.Lithium 2.Sertraline 3.Methylphenidate 4.Chlorpromazine

4. Chlorpromazine

A nurse is teaching a client about side effects of medications. Which drug will cause a heightened skin reaction to sunlight? 1. Lithium 2. Sertraline 3. Methylphenidate 4. Chlorpromazine

Occipital headaches

A nurse is teaching clients about dietary restrictions during monoamine oxidase inhibitor (MAOI) therapy. What response does the nurse tell them to anticipate if they do not follow these restrictions?

Occipital headaches -Occipital headaches are the beginning of a hypertensive crisis resulting from an excess of tyramine

A nurse is teaching clients about dietary restrictions during monoamine oxidase inhibitor (MAOI) therapy. What response does the nurse tell them to anticipate if they do not follow these restrictions?

1. Occipital headaches

A nurse is teaching clients about dietary restrictions during monoamine oxidase inhibitor (MAOI) therapy. What response does the nurse tell them to anticipate if they do not follow these restrictions? 1. Occipital headaches 2. Generalized urticaria 3. Severe muscle spasms 4. Sudden drop in blood pressure

Just after breakfast -Methylphenidate (Ritalin) should be given just after breakfast to avoid appetite suppression. Giving the medication when the child arrives at school or immediately before lunch would not allow enough time for the medication to be effective during school hours. Giving the medication when the child arrives home from school would not allow the medication to be effective during school hours, and would likely interfere with the child's sleep times.

A nurse is teaching the parents of a school-aged child with attention deficit-hyperactivity disorder (ADHD) about the prescribed medication methylphenidate (Ritalin). When should the daily dose be administered?

Blurs reality -The addict tries to avoid stress and reality. The drug produces a blurring of these feelings to the point that the addict becomes dependent on it. The psychological effect is usually more important than the ability to ease pain. Large doses of opioids, not cocaine, can cause a dreamlike state. Cocaine can increase, not decrease, motor activity.

A nurse is working in a clinic that provides services to clients who abuse drugs. What effect of cocaine should the nurse consider as the reason that it easily causes dependence?

Couple's marital problems are affecting their children -The parents' ongoing marital problems appear to have interfered with their parental roles, resulting in their children's behavioral problems. At this time the children need support, not limits. The son's and daughter's behaviors are not the priority nursing concern because there are no data to support these assumptions.

A nurse is working with a couple and their two children. The 14-year-old son has been in trouble at school because of truancy and poor grades. The 16-year-old daughter is quiet and withdrawn and refuses to talk to her parents. The parents have had severe marital problems for the past 10 years. The priority nursing concern at this time is how the:

2."Most woman attempt to leave about six times before they are able to do so." Nurses who work with clients who are victims of partner abuse need to be supportive and patient. It takes time and several attempts for most victims to be able to leave abusive relationships.

A nurse is working with a married woman who has come to the emergency department several times with injuries that appear to be related to domestic violence. While talking with the nurse manager, a nurse expresses disgust that the woman returns to the same situation. What is the nurse manager's best response? 1."She must not have the financial resources to leave her husband." 2."Most woman attempt to leave about six times before they are able to do so." 3."There is nothing the staff can do because people are free to choose their own life." 4."These women should be told how foolish they are to remain in their current situation."

2 Provide physical outlets for aggressive feelings 3 Establish a contract regarding manipulative behavior 4 Develop activities that provide opportunities for success

A nurse is working with an adolescent client with conduct disorder. Which strategies should the nurse implement while working on the goal of increasing the client's ability to meet personal needs without manipulating others? (Select all that apply.)

Blunt reality

A nurse knows individuals who are alcoholics use alcohol to:

Experience perceptual difficulties that interfere with learning -ADHD interferes with the ability to perceive and respond to sensory stimuli, resulting in a deficit in interpreting new sensory data. This makes learning difficult. It is not true that children with ADHD have intellectual deficits that interfere with learning; there is no cognitive impairment present.

A nurse knows that children with attention deficit-hyperactivity disorder (ADHD) may be learning disabled. This means that these children:

3. "It annoys me when people call me 'Dearie,' so I told him not to do it anymore." This is an assertive statement; it clearly states what the problem is and sets limits on undesired behavior without being demeaning.

A nurse leads an assertiveness training program for a group of clients. Which client statement demonstrates that the treatment has been effective? 1. "I know I should put the needs of others before mine." 2. "I won't stand for it, so I told my boss he's a jerk and to get off my back." 3. "It annoys me when people call me 'Dearie,' so I told him not to do it anymore." 4. "It is easier for me to agree up front and then just do enough so that no one notices."

"It annoys me when people call me 'sweetie,' so I told him not to do it anymore."

A nurse leads an assertiveness training program for a group of clients. Which statement by a client indicates that the treatment has been effective?

3."It annoys me when people call me 'sweetie,' so I told him not to do it anymore."

A nurse leads an assertiveness training program for a group of clients. Which statement by a client indicates that the treatment has been effective? 1."I know that I should put the needs of others before mine." 2."I won't stand for it, so I told my boss he's a jerk and to get off my back." 3."It annoys me when people call me 'sweetie,' so I told him not to do it anymore." 4."It's easier for me to agree up front and then do just enough so that no one notices."

3. "It annoys me when people call me 'sweetie,' so I told him not to do it anymore."

A nurse leads an assertiveness training program for a group of clients. Which statement by a client indicates that the treatment has been effective? 1. "I know that I should put the needs of others before mine." 2. "I won't stand for it, so I told my boss he's a jerk and to get off my back." 3. "It annoys me when people call me 'sweetie,' so I told him not to do it anymore." 4. "It's easier for me to agree up front and then do just enough so that no one notices."

1 Tremors 2 Anorexia -Hand tremors, related to dysfunction of the nervous system, are an early sign of withdrawal from alcohol; alcohol depresses the central nervous system, interferes with nerve conduction, and results in peripheral neuropathy. Signs and symptoms of alcohol withdrawal begin within 12 hours of cessation or decrease in alcohol consumption, peak in 48 to 72 hours, and usually begin to ease after 4 or 5 days. Anorexia, nausea, and vomiting are early signs of withdrawal from alcohol; alcohol affects the gastrointestinal system and can cause gastritis, pancreatitis, hepatitis, and cirrhosis. Psychomotor agitation is a late, not an early, sign of alcohol withdrawal. Transient visual, auditory, and tactile hallucinations, rather than delusions, are associated with alcohol withdrawal. Confusion, disorientation, and impaired cognition are not early signs of alcohol withdrawal; alcohol withdrawal delirium occurs in less than 10% of those who experience the alcohol withdrawal syndrome.

A nurse notes that a client in the detoxification unit is exhibiting early signs of alcohol withdrawal. What clinical manifestations might the nurse have noticed? (Select all that apply.)

Overactive

A nurse notes that haloperidol (Haldol) is most effective for clients who exhibit behavior that is:

2. Overactive

A nurse notes that haloperidol (Haldol) is most effective for clients who exhibit behavior that is: 1. Depressed 2. Overactive 3. Withdrawn 4. Manipulative

Being prompt for their scheduled meetings

A nurse on the psychiatric unit is assigned to work with a male client who appears reclusive and distrustful of everyone. The nurse can help the client develop trust by:

Being prompt for their scheduled meetings -Being prompt for their scheduled meetings helps the client feel important because the nurse remembers their meetings and is on time.

A nurse on the psychiatric unit is assigned to work with a male client who appears reclusive and distrustful of everyone. The nurse can help the client develop trust by:

The entire family is involved because what happens to one member affects all

A nurse plans to use family therapy as a means of helping a family cope with their child's terminal illness. The nurse bases this choice on the principle that:

Echolalia -The autistic child repeats sounds or words spoken by others

A nurse recalls that language development in the autistic child resembles:

Transference

A nurse reminds a client that it is time for group therapy. The client responds by shouting, "You're always telling me what to do, just like my father!" What defense mechanism is the client using?

2.Transference With transference a client assigns to someone the feelings and attitudes originally associated with an important significant other.

A nurse reminds a client that it is time for group therapy. The client responds by yelling at the nurse, "You are always telling me what to do, just like my father!" What defense mechanism is the client using? 1.Regression 2.Transference 3.Reaction formation 4.Cognitive distortion

"It must be very difficult for you to be on a psychiatric unit."

A nurse stops by the room of a newly admitted depressed client and offers to walk with the tearful client to the evening meal. The client looks intently at the nurse but says nothing. What is the best response by the nurse?

Client is assisted in investigating alternative approaches to solving the identified problem

A nurse suggests a crisis intervention group to a client experiencing a developmental crisis. The nurse knows that these groups are successful because the:

2 Pepperoni pizza 3 Bologna sandwich

A nurse teaches dietary guidelines to a client who will be receiving tranylcypromine sulfate (Parnate), a monoamine oxidase inhibitor (MAOI). The client compiles a list of foods to avoid. Which foods included on the list indicate that the teaching has been effective? (Select all that apply.)

2.Pepperoni pizza 3.Bologna sandwich

A nurse teaches dietary guidelines to a client who will be receiving tranylcypromine sulfate (Parnate), a monoamine oxidase inhibitor (MAOI). The client compiles a list of foods to avoid. Which foods included on the list indicate that the teaching has been effective? (Select all that apply.) 1.French fries 2.Pepperoni pizza 3.Bologna sandwich 4.Hamburger on a bun 5.Hash brown potatoes

2. Pepperoni pizza 3. Bologna sandwich

A nurse teaches dietary guidelines to a client who will be receiving tranylcypromine sulfate (Parnate), a monoamine oxidase inhibitor (MAOI). The client compiles a list of foods to avoid. Which foods included on the list indicate that the teaching has been effective? (Select all that apply.) 1. French fries 2. Pepperoni pizza 3. Bologna sandwich 4 . Hamburger on a bun 5 . Hash brown potatoes

Offer a therapeutic regimen compatible with the lifestyle of the family

A nurse understands that when a client is a member of a different ethnic community it is important to:

Hug with praise -Secondary reinforcers involve social approval; a hug meets this requirement.

A nurse uses behavior modification to foster toilet-training efforts in a cognitively impaired child. What reward should the nurse provide to reinforce appropriate use of the toilet?

"Have you ever felt bad or guilty about your drinking?" -The CAGE screening test for alcoholism contains four questions, corresponding to the letters CAGE: Have you ever felt you ought to Cut down on your drinking? Have people Annoyed you by criticizing your drinking? Have you ever felt bad or Guilty about your drinking? Have you ever had a drink first thing in the morning (as an "Eye-opener") to steady your nerves or get rid of a hangover? "How often did you have a drink containing alcohol in the past year?" is one of the 10 questions on the Alcohol Use Disorders Identification Test (AUDIT). "Do you feel that you are a normal drinker" and "Are you always able to stop drinking when you want to" are two of the 26 questions on the Michigan Alcohol Screening Test (MAST).

A nurse uses the CAGE screening test for alcoholism to determine an individual's potential for a drinking problem. What is one of the four questions included on this test?

24 to 72 -Alcohol withdrawal delirium, a life-threatening central nervous system response to alcohol withdrawal, occurs in 1 to 3 days when the blood alcohol levels drops as alcohol is detoxified and excreted. Jitteriness, nervousness, and insomnia may occur 8 to 12 hours after withdrawal; these are not life-threatening issues. Nervousness, insomnia, nausea, vomiting, and increased blood pressure and pulse may occur after 12 to 24 hours; these are not life-threatening problems. Withdrawal symptoms will have begun to subside after 72 to 96 hours, and the risk for complications is diminished.

A nurse, planning care for a client who is an alcoholic, knows that the most serious life-threatening effects of alcohol withdrawal usually begin after a specific time interval. How many hours after the last drink do they occur?

Thiamine deficiency -The deficiency of thiamine (vitamin B1) is thought to be a primary cause of alcohol-induced amnestic disorder.

A nurse, understanding the possible cause of alcohol-induced amnestic disorder, should take into consideration that the client is probably experiencing:

"Let's talk about how you're feeling about your child's diagnosis."

A parent of a 13-year-old adolescent with recently diagnosed Hodgkin disease tells a nurse, "I don't want her to know about the diagnosis." How should the nurse respond?

1. By denying beating the son 2. By avoiding talking about the situation 5. Emotional response is inconsistent with degree of injury

A parent of four is remanded to the psychiatric unit by the court for observation. The client was arrested and charged with abusing a 2-year-old son, who is in the pediatric intensive care unit in critical condition. The nurse approaches the client for the first time. How should the nurse anticipate that the client will likely respond? (Select all that apply.)

1.By denying beating the son 2.By avoiding talking about the situation 5.Emotional response is inconsistent with degree of injury

A parent of four is remanded to the psychiatric unit by the court for observation. The client was arrested and charged with abusing a 2-year-old son, who is in the pediatric intensive care unit in critical condition. The nurse approaches the client for the first time. How should the nurse anticipate that the client will likely respond? (Select all that apply.) 1.By denying beating the son 2.By avoiding talking about the situation 3.By expressing excessive concern for the son 4.By asking where the other three children are 5.Emotional response is inconsistent with degree of injury

1. By denying beating the son 2. By avoiding talking about the situation 5. Emotional response is inconsistent with degree of injury

A parent of four is remanded to the psychiatric unit by the court for observation. The client was arrested and charged with abusing a 2-year-old son, who is in the pediatric intensive care unit in critical condition. The nurse approaches the client for the first time. How should the nurse anticipate that the client will likely respond? (Select all that apply.) 1. By denying beating the son 2. By avoiding talking about the situation 3. By expressing excessive concern for the son 4. By asking where the other three children are 5. Emotional response is inconsistent with degree of injury

Bereavement may be of greater intensity and duration.

A parent whose daughter is killed in a school bus accident tearfully tells the nurse, "My daughter was just getting over the chickenpox and didn't want to go to school, but I insisted that she go. It's my fault that she's dead." How should the nurse anticipate that perceiving a death as preventable will likely influence the grieving process?

3. Bereavement may be of greater intensity and duration.

A parent whose daughter is killed in a school bus accident tearfully tells the nurse, "My daughter was just getting over the chickenpox and didn't want to go to school, but I insisted that she go. It's my fault that she's dead." How should the nurse anticipate that perceiving a death as preventable will likely influence the grieving process? 1. The loss may be easier to understand and accept. 2. The mourner may experience pathological grief. 3. Bereavement may be of greater intensity and duration. 4. The grieving process may progress to a psychiatric illness

11 years

A person mowing a lawn is badly disfigured by the lawnmower blade. According to Erikson's theory, which age at the time of injury will be associated with the greatest risk of long-term psychological effects?

4."I am sorry, but I cannot allow that because I must take your blood pressure."

A physician is admitted to the psychiatric unit of a community hospital. The client, who was restless, loud, aggressive, and resistive during the admission procedure, states, "I will take my own blood pressure." What is the nurse's most therapeutic response? 1."Right now you are just another client." 2."If you would rather, I'm sure you will do it correctly." 3."I will get the attendants to assist me if you do not cooperate." 4."I am sorry, but I cannot allow that because I must take your blood pressure."

Drowsiness

A practitioner prescribes Alprazolam (Xanax) 0.25 mg by mouth three times a day for a client with anxiety and physical symptoms related to work pressures. For what most common side effect of this drug should the nurse monitor the client?

Set a contract with the client to verbalize frustrations before acting out

A psychiatric client recently admitted to the inpatient unit has a history of angry outbursts. The client's anger appears to be escalating, although the client still appears to be in control. What should the nurse do first to prevent an incident from developing?

1.Set a contract with the client to verbalize frustrations before acting out

A psychiatric client recently admitted to the inpatient unit has a history of angry outbursts. The client's anger appears to be escalating, although the client still appears to be in control. What should the nurse do first to prevent an incident from developing? 1.Set a contract with the client to verbalize frustrations before acting out 2.Establish firm control and use seclusion before the client acts out impulsively 3.Show the client the seclusion room as a method of deterring acting-out behavior 4.Call the health care provider for a prescription for an intramuscular sedative for the client

1. Hospital policy

A psychiatric nurse is hired to work in the psychiatric emergency department of a large teaching hospital. While reviewing the manuals, the nurse reads, "People with mental health emergencies shall be triaged within 5 minutes of entering the emergency department." What does the nurse consider this statement to represent? 1. Hospital policy 2. Standard of care 3. Hospital procedure 4. Mental Health Bill of Rights

Performing hygiene activities independently

A psychiatric unit uses a behavioral approach to determine a client's level of privileges. Which factor should a nurse use to determine whether an increase in privileges is warranted?

The nurse shares with the entire treatment team vital information the client disclosed in a private session.

A recently hired nurse is caring for several clients on a mental health unit at a local community hospital. The nurse manager is evaluating the nurse's performance. What situation indicates that the nurse-client boundaries of the recently hired nurse are appropriate?

1.The nurse shares with the entire treatment team vital information the client disclosed in a private session.

A recently hired nurse is caring for several clients on a mental health unit at a local community hospital. The nurse manager is evaluating the nurse's performance. What situation indicates that the nurse-client boundaries of the recently hired nurse are appropriate? 1.The nurse shares with the entire treatment team vital information the client disclosed in a private session. 2.The nurse is often busy doing other tasks when the client and nurse are scheduled for a counseling session. 3.A client enters the therapeutic group late with the nurse's permission even though group rules say that this is not allowed. 4.A client's overall behavior is significantly more independent and demonstrates higher function on the days that the nurse is not working

Client requests something to calm her

A recently married 22-year-old woman is brought to the trauma center by the police. She has been robbed, beaten, and sexually assaulted. The client, although anxious and tearful, appears to be in control. The health care provider prescribes alprazolam (Xanax) 0.25 mg for agitation. The nurse should administer this medication when the:

2. Client requests something to calm her **Because a sexual assault is a threat to the sense of control over one's life, some control should be given back to the client as soon as possible.

A recently married 22-year-old woman is brought to the trauma center by the police. She has been robbed, beaten, and sexually assaulted. The client, although anxious and tearful, appears to be in control. The health care provider prescribes alprazolam (Xanax) 0.25 mg for agitation. The nurse should administer this medication when the: 1. Client's crying increases 2. Client requests something to calm her 3. Nurse determines a need to reduce her anxiety 4. Health care provider is getting ready to perform a vaginal examination

Self-help group

A recovering alcoholic joins Alcoholics Anonymous (AA) to help maintain sobriety. What type of group is AA?

By offering choices consistent with the client's heritage

A resident in a nursing home recently immigrated to the United States from Italy. How does the nurse plan to provide emotional support?

Undoing

A secretary in a home health agency gossips about coworkers and then writes them notes to tell them how valuable they are to the organization and how much she likes working with them. What defense mechanism is being used by the secretary?

2.Undoing

A secretary in a home health agency gossips about coworkers and then writes them notes to tell them how valuable they are to the organization and how much she likes working with them. What defense mechanism is being used by the secretary? 1.Denial 2.Undoing 3.Displacement 4.Intellectualization

3.Bargaining

A terminally ill client repeatedly tells the nurse all the details of a daughter's wedding that will take place in 6 months and how important it is for her to attend. What Kübler-Ross stage of grieving does the nurse identify? 1.Anger 2.Denial 3.Bargaining 4.Acceptance

Immediately after the client's admission to the hospital

A widow who is hospitalized for a medical problem has dementia of the Alzheimer type and is no longer able to live alone. The client is to be transferred from the hospital to a long-term care facility. When should the staff begin preparation for the transfer?

Determining a safe place to go in an emergency Memorizing the domestic violence hotline number

A woman who is emotionally and physically abused by her husband calls a crisis hotline for help. The nurse works with the client to develop a plan for safety. What should be included in the safety plan? (Select all that apply.)

2. Determining a safe place to go in an emergency 3. Memorizing the domestic violence hotline number

A woman who is emotionally and physically abused by her husband calls a crisis hotline for help. The nurse works with the client to develop a plan for safety. What should be included in the safety plan? (Select all that apply.) 1. Limiting contact with the abuser 2. Determining a safe place to go in an emergency 3. Memorizing the domestic violence hotline number 4. Obtaining a bank loan to finance leaving the abuser 5. Arranging for a family member to assist her in leaving

"You say that it was your fault—help me understand that."

A woman who is frequently physically abused tells the nurse in the emergency department that it is her fault that her husband beats her. What is the most therapeutic response by the nurse?

4. "You say that it was your fault—help me understand that."

A woman who is frequently physically abused tells the nurse in the emergency department that it is her fault that her husband beats her. What is the most therapeutic response by the nurse? 1. "Maybe it was your husband's fault, too." 2. "I can't agree with that—no one should be beaten." 3. "Tell me why you believe that you deserve to be beaten." 4. "You say that it was your fault—help me understand that."

Obtaining information about her perception of the incident

A woman who was sexually assaulted by a stranger in the elevator of her apartment building is brought by her husband to the emergency department. What is the priority nursing intervention?

Excessive physical activity

A woman with bipolar disorder, manic episode, has been spending thousands of dollars on clothing and makeup. She has been partying in bars every night and rarely sleeps or eats. The nurse in the outpatient clinic, knowing that this client rarely eats, recognizes that her eating problems most likely result from her:

Excessive physical activity -During a manic episode hyperactivity and the inability to sit still long enough to eat are the causes of eating difficulties. Feelings of guilt do not precipitate eating difficulties in clients with the diagnosis of bipolar disorder, manic episode.

A woman with bipolar disorder, manic episode, has been spending thousands of dollars on clothing and makeup. She has been partying in bars every night and rarely sleeps or eats. The nurse in the outpatient clinic, knowing that this client rarely eats, recognizes that her eating problems most likely result from her:

Inquire about her and the children's safety -The safety of the victim and the children must be determined, because research shows that children of an alcoholic parent are frequently abused. If the nurse suspects child abuse, a report must be made to child protective services.

A woman with five children comes to the emergency department with multiple facial injuries. The client says, "My husband is an alcoholic, and he just beat me up." The nurse concludes that the client appears to be a victim of abuse. What should the nurse do next?

3.Inquire about her and the children's safety

A woman with five children comes to the emergency department with multiple facial injuries. The client says, "My husband is an alcoholic, and he just beat me up." The nurse concludes that the client appears to be a victim of abuse. What should the nurse do next? 1.Discuss birth control with her 2.Report her experiences to the police 3.Inquire about her and the children's safety 4.Discuss the possibility of her and the children leaving her husband

"Being incapacitated is difficult for you."

A young adult client is hospitalized with a spinal cord injury. The client, knowing that the paralysis may be permanent, says, "I wish God would end my suffering and take me." What is the most therapeutic initial response by the nurse?

Illusion

A young client is admitted to the hospital with a diagnosis of acute schizophrenia. The family reports that one day the client looked at a linen sheet on a clothesline and thought it was a ghost. What is the most appropriate conclusion to make about what the client was experiencing?

Illusion An illusion is a misinterpretation of an actual sensory stimulus. A delusion is a false, fixed belief. A hallucination is a false sensory perception that occurs with no stimulus. Confabulation is a filling in of blanks in memory.

A young client is admitted to the hospital with a diagnosis of acute schizophrenia. The family reports that one day the client looked at a linen sheet on a clothesline and thought it was a ghost. What is the most appropriate conclusion to make about what the client was experiencing? Illusion Delusion Hallucination Confabulation

1. Primary prevention

A young client who has become a mother for the first time is anxious about her new parenting role. With the nurse's encouragement, she joins the new mothers' support group at the local YMCA. What kind of prevention does this activity reflect? 1. Primary prevention 2. Tertiary prevention 3. Secondary prevention 4. Therapeutic prevention

"I understand that you're hearing voices talking to you and that the voices are very real to you. What are the voices saying to you? Acknowledging that client is hearing voices talking to him and that the voices are very real to him validates the presence of the client's hallucinations without agreeing with them, which communicates acceptance and can form a foundation for trust; it may help the client return to reality. The nurse also needs to assess the content of the voices to determine the risk of self injury or violence against others. The client's contact with reality is too tenuous to explore what they mean. Saying that the client is the only one hearing the voices and asking whether he is sure that he is hearing demeans the client, which blocks the development of a trusting relationship and future communication. Telling the client that the health team members will observe his behavior and that he won't be left alone is condescending and may impair future communication.

A young client with schizophrenia says, "I'm starting to hear voices." What is the nurse's most therapeutic response? "How do you feel about the voices, and what do they mean to you?" "You're the only one hearing the voices. Are you sure you hear them?" "The health team members will observe your behavior. We won't leave you alone." "I understand that you're hearing voices talking to you and that the voices are very real to you. What are the voices saying to you?

Providing a safe environment that permits the ventilation of feelings

A young female client admitted to the trauma center after being sexually assaulted continues to talk about the rape. Toward what goal should the primary nursing intervention be directed?

4. Providing a safe environment that permits the ventilation of feelings

A young female client admitted to the trauma center after being sexually assaulted continues to talk about the rape. Toward what goal should the primary nursing intervention be directed? 1. Getting her involved with a rape therapy group 2. Remaining available and supportive to limit destructive anger 3. Exploring her feelings about men to promote future relationships 4. Providing a safe environment that permits the ventilation of feelings

Manipulating the environment to benefit the client

A young mother of three children, all born 1 year apart, has been hospitalized after trying to hang herself. The client is being treated with milieu therapy. The nurse understands that this therapeutic modality consists of:

4. Manipulating the environment to benefit the client

A young mother of three children, all born 1 year apart, has been hospitalized after trying to hang herself. The client is being treated with milieu therapy. The nurse understands that this therapeutic modality consists of: 1. Providing individual and family therapy 2. Using positive reinforcement to reduce guilt 3. Uncovering unconscious conflicts and fantasies 4. Manipulating the environment to benefit the client

What is the primary reason that the nurse encourages the family of an alcoholic to become involved in the treatment program?

Alcoholism involves the entire family.

A psychologist has been a client on a mental health unit for 3 days. The client has questioned the authority of the treatment team, advised other clients that their treatment plans are wrong, and been disruptive in group therapy. What is the most appropriate nursing intervention?

Accepting that the client is unable to control this behavior and setting appropriate limits

What is the best initial nursing approach to take with a self-accusatory, guilt-ridden client?

Accepting the client's statements as the client's beliefs

Trust Without the development of trust, the child has little confidence that the significant other will return; separation is considered abandonment by the child.

According to Erikson, a child's increased vulnerability to anxiety in response to separation or pending separation from significant others results from failure to complete a developmental stage. What does the nurse call this stage?

1. Trust

According to Erikson, a child's increased vulnerability to anxiety in response to separation or pending separation from significant others results from failure to complete a developmental stage. What does the nurse call this stage? 1. Trust 2. Identity 3. Initiative 4. Autonomy

Generativity versus stagnation -Erikson theorized that how well people adapt to the current stage depends on how well they adapted to the stage immediately preceding it—in this instance, adulthood. Although Erikson believed that the strengths and weaknesses of each stage are present in some form in all succeeding stages, their influence decreases with time

According to Erikson, a person's adjustment to the period of senescence will depend largely on the adjustment the individual made to the earlier developmental stage of:

Intimacy versus isolation

According to Erikson, a young adult must accomplish the tasks associated with the stage known as:

A nurse is discussing plans with a client who has decided to withdraw from alcohol. What should the nurse recommend as one of the most effective treatments for alcoholism?

Active membership in Alcoholics Anonymous

Oral chlordiazepoxide (Librium) 100 mg/ hr is prescribed for a client with a Clinical Institute Withdrawal Assessment (CIWA) score of 25. The client has had 300 mg in 3 hours but is still displaying acute alcohol withdrawal symptoms. What is the next nursing action?

Administering chlordiazepoxide as indicated by the client's CIWA score

"We should talk some more about this."

After 4 days on the inpatient psychiatric unit a client on suicidal precautions tells the nurse, "Hey, look! I was feeling pretty depressed for a while, but I'm certainly not going to kill myself." What is the nurse's best response to this statement?

"Tell me more about what's bothering you."

After a child's visit to a health care provider, the parent tells the nurse, "I'm so upset! The doctor prescribed an antidepressant!" What is the best response by the nurse?

2. Repression

After an automobile collision involving a fatality and a subsequent arrest for speeding, a client has amnesia regarding the events surrounding the accident. Which defense mechanism is being used by the client? 1. Projection 2. Repression 3. Suppression 4. Rationalization

Seeking support from colleagues on the unit -Talking with colleagues who face or who have faced the same problems may provide constructive help with the situation

After caring for a terminally ill client for several weeks, a nurse becomes increasingly aware of a need for a respite from this assignment. What is the best initial action by the nurse?

4 Determine whether the family is in accord with the practitioner while following hospital policy

After speaking with the parents of a child dying of leukemia, the practitioner gives a verbal do-not-resuscitate order but refuses to put it in writing. What should the nurse do?

4. Determine whether the family is in accord with the practitioner while following hospital policy

After speaking with the parents of a child dying of leukemia, the practitioner gives a verbal do-not-resuscitate order but refuses to put it in writing. What should the nurse do? 1. Follow the order as given by the practitioner 2. Refuse to follow the practitioner's order unless the nursing supervisor approves it 3. Ask the practitioner to write the order in pencil on the child's chart before leaving the room 4. Determine whether the family is in accord with the practitioner while following hospital policy

A client is prescribed a monoamine oxidase inhibitor. The nurse teaches the client about what foods to avoid when taking this medication. (Select all that apply.)

Aged cheese Ripe avocados Delicatessen meats

A monoamine oxidase inhibitor (MAOI) is prescribed. What should the nurse instruct the client to avoid while taking this drug?

Aged cheeses

A client with schizophrenia who is being admitted to a psychiatric hospital for evaluation refuses to remove dirty clothing. What should the nurse do to best meet the client's needs?

Allow the client to undress when ready to help maintain identity

A nurse is monitoring a client with the diagnosis of schizophrenia who is experiencing opposing emotions simultaneously. When providing a change-of-shift report, how should the nurse refer to this emotional experience of the client?

Ambivalence

Tricyclics -Amitriptyline (Elavil) is one of several tricyclic antidepressants used to treat anxiety disorders .

Amitriptyline (Elavil) is an antidepressant medication used to treat anxiety disorders. Which class of antidepressant medications does it belong to?

1.Tricyclics

Amitriptyline (Elavil) is an antidepressant medication used to treat anxiety disorders. Which class of antidepressant medications does it belong to? 1.Tricyclics 2.Monoamine oxidase inhibitors (MAOIs) 3.Selective serotonin reuptake inhibitors (SSRIs) 4.Serotonin-norepinephrine reuptake inhibitors (SNRIs)

1. Tricyclics

Amitriptyline (Elavil) is an antidepressant medication used to treat anxiety disorders. Which class of antidepressant medications does it belong to? 1. Tricyclics 2. Monoamine oxidase inhibitors (MAOIs) 3. Selective serotonin reuptake inhibitors (SSRIs) 4. Serotonin-norepinephrine reuptake inhibitors (SNRIs)

2.Health promotion 3.Case management 5.Treating human responses

Among members of the nursing team, which functions are registered nurses legally permitted to perform in a mental health hospital? Select all that apply. 1.Psychotherapy 2.Health promotion 3.Case management 4.Prescribing medication 5.Treating human responses

Identifies the feelings underlying the acting-out behavior

An adolescent client with an antisocial personality disorder has been admitted to the hospital because of drug abuse and repeated sexual acting-out behavior. Which client behavior supports the nurse's conclusion that actions directed toward modifying the behavior of this client have been successful?

Attempting to establish a trusting relationship with the adolescent

An adolescent is admitted to the psychiatric service in stable physical condition with the diagnosis of anorexia nervosa. The adolescent has lost 20 lb in 6 weeks and is very thin but is excessively concerned about being overweight. What is the most important initial nursing intervention?

Attempting to establish a trusting relationship with the adolescent -the problem is psychological. Therefore the nurse's initial approach should be directed toward establishing trust.

An adolescent is admitted to the psychiatric service in stable physical condition with the diagnosis of anorexia nervosa. The adolescent has lost 20 lb in 6 weeks and is very thin but is excessively concerned about being overweight. What is the most important initial nursing intervention?

2 Talking about the situation that precipitated the anger

An adolescent on the psychiatric unit has an angry outburst toward another client who cut in front of people standing in line to get their mail. Later the nurse conducts a one-on-one therapeutic session with the angry client. What is an appropriate short-term goal for the client to strive for?

Preventing violence

An adolescent who has had the diagnosis of conduct disorder since the age of 9 is placed in a residential facility. The adolescent has a history of fighting, stealing, vandalizing property, and running away from home. The adolescent is aggressive, has no friends, and has been suspended from school repeatedly. What is the nurse's priority when planning care?

Demonstration of respect for the rights of others -Demonstrating respect for the rights of others is a specific outcome criterion for children with a risk for violence directed at others; children with the diagnosis of conduct disorder typically present with a repetitive and persistent pattern of behavior that violates the basic rights of others or major age-appropriate societal norms or rules

An adolescent with a conduct disorder is undergoing behavioral therapy in an attempt to limit behaviors that violate societal norms. A specific outcome criterion unique to adolescents with this problem is:

Asking the client, "Have you ever acted on these thoughts?"

An adult client confides to a clinic nurse, "I fantasize about having sex with children, and I get the urge to do it, too." What is the most appropriate response by the nurse?

1.Identify the client's support systems. 4.Encourage the client to talk about the situation. A client in crisis needs to rely on available support systems for assistance; therefore, it is vital for the nurse to identify the client's support system. Talking about the situation helps the individual to put the crisis in perspective.

An adult who has been in a gay relationship for 3 years arrives at the emergency department in a near panic state. The client states, "My partner just left me. I am a wreck." What should the nurse do to help the client cope with this loss? Select all that apply. 1.Identify the client's support systems. 2.Explore the client's psychotic thoughts. 3.Reinforce the client's current self-image. 4.Encourage the client to talk about the situation. 5.Suggest that the client explore personal sexual attitudes.

1 Identify the client's support systems 4 Encourage the client to talk about the situation

An adult who has been in a gay relationship for 3 years arrives at the emergency department in a state of near-panic. The client says, "My partner just left me. I'm a wreck." What should the nurse do to help the client cope with this loss? (Select all that apply.)

1. Identify the client's support systems 4. Encourage the client to talk about the situation

An adult who has been in a gay relationship for 3 years arrives at the emergency department in a state of near-panic. The client says, "My partner just left me. I'm a wreck." What should the nurse do to help the client cope with this loss? (Select all that apply.) 1. Identify the client's support systems 2. Explore the client's psychotic thoughts 3. Reinforce the client's current self-image 4. Encourage the client to talk about the situation 5 . Suggest that the client explore personal sexual attitudes

Seek out the client frequently to spend short periods of time together

An adult with the diagnosis of schizophrenia is admitted to the psychiatric hospital. The client is ungroomed, appears to be hearing voices, is withdrawn, and has not spoken to anyone for several days. What should the nurse do during the first few hospital days?

Seek out the client frequently to spend short periods of time together. Seeking out the client frequently to spend short periods of time together will help the nurse establish trust without unduly increasing anxiety. Seeing that the client bathes and changes clothes daily is not the priority unless the client is extremely dirty; this client is ungroomed, not dirty. A withdrawn client will usually not approach anyone. The client's history reveals a failure to speak.

An adult with the diagnosis of schizophrenia is admitted to the psychiatric hospital. The client is ungroomed, appears to be hearing voices, is withdrawn, and has not spoken to anyone for several days. What should the nurse do during the first few hospital days? See that the client bathes and changes clothes daily. Wait and see whether the client approaches the staff. Conduct an admission assessment interview with the client. Seek out the client frequently to spend short periods of time together.

1. Jaundice 3. Tachycardia -Jaundice signifies liver function interference and requires that the medication be stopped. Tachycardia, QT-interval prolongation, and cardiac arrest are life-threatening cardiovascular effects of haloperidol (Haldol).

An agitated, acting-out, delusional client is receiving large doses of haloperidol (Haldol), and the nurse is concerned because this drug can produce untoward side effects. Which clinical manifestations should alert the nurse to stop the drug immediately? (Select all that apply.)

"Antidepressant therapy requires several weeks before it becomes effective."

An antidepressant is prescribed for a depressed older client. After 1 week the client's son expresses concern that there does not seem to be much improvement. How should the nurse respond?

Somatization

An anxious client reports experiencing pain in the abdomen and feeling empty and hollow. A diagnostic workup reveals no physical causes of these clinical findings. What term best reflects what the client is experiencing?

Sitting with the toddler while watching the spinning top to provide a nonintrusive presence -Autistic children relate best with objects, which can be used as a bridge in interpersonal relationships; this begins at the child's level. Autistic children often become agitated when movement is restricted and personal space is invaded. Autistic children usually have difficulty tolerating being touched. They will not initiate contact or interactions.

An autistic toddler is sitting in a corner, rocking and spinning a top. How can the nurse be most therapeutic when approaching this toddler?

Suppression

An executive, busy at work, receives a phone call from a friend relating bad news. The woman makes a conscious effort to put this information out of her mind and continues to work at the task at hand. The next day she remembers that her friend telephoned her but is unable to recall the message. Which defense mechanism does this behavior represent?

2. Suppression **Suppression is the voluntary exclusion from awareness of anxiety-producing feelings, ideas, and situations.

An executive, busy at work, receives a phone call from a friend relating bad news. The woman makes a conscious effort to put this information out of her mind and continues to work at the task at hand. The next day she remembers that her friend telephoned her but is unable to recall the message. Which defense mechanism does this behavior represent? 1. Regression 2. Suppression 3. Passive aggression 4. Reaction formation

2."I will be here for you to help you figure things out." Clients in crisis need assistance with coping; the nurse must be involved with problem solving.

An extremely anxious client enters a crisis center and asks a nurse for help. Which response best reflects the nurse's role in crisis intervention? 1. "Tell me what you have done to help yourself." 2."I will be here for you to help you figure things out." 3."I understand that in the past you have had problems." 4."Tell me about the things that are bothering you the most."

Self-deprecation -The client's statement is self-derogatory and reflects a low self-appraisal.

An individual whose employment has been terminated because his company has been acquired by another company is brought by a family member to the mental health clinic because of extreme depression. While talking with the nurse the client says, "I'm a useless, worthless person. No wonder I lost my job." What type of delusion does the nurse identify?

Integrity versus despair

An older retired client is visiting the clinic for a regularly scheduled checkup. The client tells the nurse about the great life he has lived and the activities that he enjoys at the senior center. According to Erikson, what developmental conflict has been resolved by this client?

2. Focusing on the child's role in sustaining the injury 3. Changing the story of how the child sustained the injury 5. Giving an explanation of how the injury occurred that is not consistent with the injury

An injured child is brought to the emergency department by the parents. While interviewing the parents, the nurse begins to suspect child abuse. Which parental behaviors might support this conclusion? (Select all that apply.) 1. Demonstrating concern for the injured child 2. Focusing on the child's role in sustaining the injury 3. Changing the story of how the child sustained the injury 4. Asking questions about the injury and the child's prognosis 5. Giving an explanation of how the injury occurred that is not consistent with the injury

Moderate

An obviously upset client comes to the mental health clinic and, after pushing ahead of the other clients, states, "I had an argument with my daughter, and now I'm tense, and worried, and angry." What level of anxiety does the nurse determine that the client is experiencing?

Identify the name of the clinic's town

An older adult is brought to the clinic by a family member because of increasing confusion over the past week. What can the nurse ask the client to do to determine orientation to place?

1 Withholding the next dose of lithium and drawing blood to test it for toxicity

An older adult living in a long-term care facility has been receiving lithium 600 mg twice a day for 3 weeks to ease manic behavior. The client is experiencing nausea and vomiting, diarrhea, thirst, polyuria, slurred speech, and muscle weakness. What is the most appropriate nursing intervention?

Ego integrity versus despair

An older adult tells the nurse, "I regret so many of the choices I've made during my life." Which of Erikson's developmental conflicts has the client probably failed to accomplish?

Denial

An older adult who lives alone tells a nurse at the community health center, "I really don't need anyone to talk to. The TV is my best friend." The nurse identifies the defense mechanism known as:

Giving the client one simple direction at a time in a firm low-pitched voice -Clients who are out of control are seeking control and frequently respond to simple directions stated in a firm voice.

An older adult with a diagnosis of delirium on the mental health unit begins acting out while in the dayroom. What is the initial nursing intervention?

Help the client select appropriate attire and offer to help the client get dressed

An older client is transferred to a nursing home from a hospital with a diagnosis of dementia. One morning, after being in the nursing home for several days, the client is going to join a group of residents in recreational therapy. The nurse sees that the client has laid out several outfits on the bed but is still wearing nightclothes. What should the nurse do?

Say, "You feel she doesn't want you at home."

An older client whose family has been visiting on the psychiatric unit is visibly angry and says to the nurse, "My daughter-in-law says they can't take me home until the doctor lets me go. She doesn't understand how important this is to me; she's not from our culture." What should the nurse do?

2. Say, "You feel she doesn't want you at home.

An older client whose family has been visiting on the psychiatric unit is visibly angry and says to the nurse, "My daughter-in-law says they can't take me home until the doctor lets me go. She doesn't understand how important this is to me; she's not from our culture." What should the nurse do? 1. Ignore the statement for the present 2. Say, "You feel she doesn't want you at home. 3. Reflect on the client's feelings about the cultural differences 4. Respond, "The doctor is the one who makes decisions about discharge."

Receptive aphasia

An older client with vascular dementia has difficulty following simple directions for selecting clothes to be worn for the day. The nurse identifies that these problems as the result of:

Receptive aphasia -Receptive aphasia interferes with interpreting and defining words in addition to following directions and selecting clothes. Following directions does not require skill in judgment or decision-making.

An older client with vascular dementia has difficulty following simple directions for selecting clothes to be worn for the day. The nurse identifies that these problems as the result of:

Projection -The client is assigning to others those feelings and emotions that are unacceptable to him- or herself. Introjection is treating something outside the self as if it is inside the self.

An older depressed person at an independent living facility constantly complains about her health problems to anyone who will listen. One day the client says, "I'm not going to any more activities. All these old crabby people do is talk about their problems." What defense mechanism does the nurse conclude that the client is using?

Reinforcing routines and supporting her usual habits

An older female client is concerned about maintaining her independent living status. What initial intervention strategy is of primary importance?

1. Reinforcing routines and supporting her usual habits

An older female client is concerned about maintaining her independent living status. What initial intervention strategy is of primary importance? 1. Reinforcing routines and supporting her usual habits 2. Helping her secure assistance with cleaning and shopping 3. Writing down and repeating important information for her use 4. Setting clear goals and time limitations for her visits with the nurse

Assure him that everything possible was done for his wife

An older man is widowed suddenly when his wife is killed in an automobile accident. What should the nurse in the emergency department do first to best help the client at this time?

4.Assure him that everything possible was done for his wife

An older man is widowed suddenly when his wife is killed in an automobile accident. What should the nurse in the emergency department do first to best help the client at this time? 1.Ask a member of the clergy to visit him 2.Have the practitioner prescribe a sedative for him 3.Refer him to a support group that meets near his home 4.Assure him that everything possible was done for his wife

4. Assure him that everything possible was done for his wife

An older man is widowed suddenly when his wife is killed in an automobile accident. What should the nurse in the emergency department do first to best help the client at this time? 1. Ask a member of the clergy to visit him 2. Have the practitioner prescribe a sedative for him 3. Refer him to a support group that meets near his home 4. Assure him that everything possible was done for his wife

2. Integrity versus despair

An older retired client is visiting the clinic for a regularly scheduled checkup. The client tells the nurse about the great life he has lived and the activities that he enjoys at the senior center. According to Erikson, what developmental conflict has been resolved by this client? 1. Trust versus mistrust 2. Integrity versus despair 3. Generativity versus self-absorption 4. Autonomy versus shame and doubt

A nurse is caring for a group of depressed clients. What should the nurse attempt to provide?

An uncomplicated daily schedule

A client with bipolar disorder, manic episode, has a superior, authoritative manner and constantly instructs other clients in how to dress, what to eat, and where to sit. The nurse should intervene because these behaviors eventually will cause the other clients to feel:

Angry

1 Rigidity 2 Tremors 5 Bradykinesia

Antipsychotic drugs can cause extrapyramidal side effects. Which responses should the nurse document as indicating pseudoparkinsonism? (Select all that apply.)

1.Rigidity 2.Tremors 5.Bradykinesia

Antipsychotic drugs can cause extrapyramidal side effects. Which responses should the nurse document as indicating pseudoparkinsonism? (Select all that apply.) 1.Rigidity 2.Tremors 3.Mydriasis 4.Photophobia 5.Bradykinesia

1. Rigidity 2. Tremors 5. Bradykinesia

Antipsychotic drugs can cause extrapyramidal side effects. Which responses should the nurse document as indicating pseudoparkinsonism? (Select all that apply.) 1. Rigidity 2. Tremors 3. Mydriasis 4. Photophobia 5. Bradykinesia

A nurse works with school-age children who have conduct disorder, childhood-onset type. The nurse knows that these children are at risk for progression to another disorder during adolescence. For signs of which disorder should the nurse evaluate their current behavior?

Antisocial personality

During the orientation tour for three new staff members, a young, hyperactive manic client greets them by saying, "Welcome to the funny farm. I'm Jo-Jo, the head yo-yo." Which meaning can the nurse assign to the client's statement?

Anxious over the arrival of new staff members

A nurse is caring for several clients who are going through withdrawal from alcohol. The primary reason for the ingestion of alcohol by clients with a history of alcohol abuse is that they:

Are dependent on it

Somatic delusion A somatic delusion is a false feeling about the physical self that is caused by a loss of reality testing. Paranoid ideations are beliefs that the individual is being singled out for unfair treatment. Loose associations are verbalizations that are difficult to understand because the links between thoughts are not apparent. Ideas of reference are false beliefs that the words and actions of others are concerned with or are directed toward the individual.

As a nurse is assisting a client with the diagnosis of schizophrenia with morning care, the client suddenly throws off the covers and starts shouting, "My body is disintegrating! I'm being pinched." What term best describes the client's behavior? Somatic delusion Paranoid ideation Loose association Ieas of referenc

4."You feel you will not be accepted unless you have something to say?" This reflective statement allows the client to either validate or correct the nurse.

As depression begins to lift, a client is asked to join a small discussion group that meets every evening on the unit. The client is reluctant to join because "I have nothing to talk about." What is the best response by the nurse? 1."Maybe tomorrow you will feel more like talking." 2."Could you start off by talking about your family?" 3."A person like you has a great deal to offer the group." 4."You feel you will not be accepted unless you have something to say?"

"You feel you won't be accepted unless you have something to say?" -The statement about the client's feelings of acceptance is a reflective statement that allows the client to either validate the statement or correct the nurse

As depression begins to lift, a client is asked to join a small discussion group that meets every evening on the unit. The client is reluctant to join because, she says, "I have nothing to talk about." What is the best response by the nurse?

4."You feel you won't be accepted unless you have something to say?"

As depression begins to lift, a client is asked to join a small discussion group that meets every evening on the unit. The client is reluctant to join because, she says, "I have nothing to talk about." What is the best response by the nurse? 1."Maybe tomorrow you'll feel more like talking." 2."Could you start off by talking about your family?" 3."A person like you has a great deal to offer the group." 4."You feel you won't be accepted unless you have something to say?"

Attitudes and beliefs -Some attitudes and beliefs include reluctance by older people to seek help because of pride in their independence, stoic acceptance of difficulty, unawareness of resources, and fear of being "put away." Although the client mentions "being put away", that is an attitude

As the nurse is discussing psychiatric care with an older adult client, the client says, "When I was growing up I was taught to accept my lot in life and not complain. I'm proud of the fact that despite my issues I can still function independently. I don't want to be just put away." The nurse understands that the factors that influence the client's mental health are examples of:

3. Attitudes and beliefs

As the nurse is discussing psychiatric care with an older adult client, the client says, "When I was growing up I was taught to accept my lot in life and not complain. I'm proud of the fact that despite my issues I can still function independently. I don't want to be just put away." The nurse understands that the factors that influence the client's mental health are examples of: 1. Setting of care 2. Anxiety disorder 3. Attitudes and beliefs 4. Cultural and ethnic disparities

An adult client confides to a clinic nurse, "I fantasize about having sex with children, and I get the urge to do it, too." What is the most appropriate response by the nurse?

Asking the client, "Have you ever acted on these thoughts?"

A nurse is caring for a client with the diagnosis of alcohol withdrawal delirium. Which action is most appropriate for the nurse to implement?

Assuring the client that the symptoms are part of the withdrawal syndrome

Leaving a dim light on in the client's room at night

At night an older client with dementia sleeps very little and becomes more disoriented. How can the nurse best limit this confusion resulting from sleep deprivation?

Seeking consensual validation -Seeking consensual validation is a technique that prevents misunderstanding so that both the client and the nurse can work toward a common goal in the therapeutic relationship.

At times a client's anxiety level is so high that it blocks attempts at communication and the nurse is unsure of what is being said. To clarify understanding, the nurse says, "Let's see whether we mean the same thing." What communication technique is being used by the nurse?

3 to 5years -Three to 5 years is Freud's phallic stage and Erikson's stage of initiative versus guilt. Adolescence is Freud's genital stage and Erikson's stage of identity versus role confusion. Six to 12 years is Freud's latency stage and Erikson's stage of industry versus inferiority. Birth to 1 year is Freud's oral stage and Erikson's stage of trust versus mistrust.

At what age is a client in Freud's phallic stage of psychosexual development and Erikson's psychosocial phase of initiative versus guilt?

A nurse is assigned to care for a college student who has been talking to unseen people and refusing to get out of bed, go to class, or participate in daily grooming activities. What is the nurse's initial effort toward helping this client?

Attempting to establish a meaningful relationship with the client

The nurse recalls that the major defense mechanism used by an individual with a phobic disorder is:

Avoidance

A nurse is caring for a client with a diagnosis of conversion disorder manifesting as paralysis of the legs. Which is the most therapeutic nursing intervention?

Avoiding focusing on the client's physical symptoms

A client with the diagnosis of obsessive-compulsive disorder uses paper towels to open doors to avoid touching dirty doorknobs. How should the nurse respond initially to this behavior?

By allowing the behavior for the time being

What characteristic of the environment is most therapeutic for clients with the diagnosis of bulimia nervosa?

Based on realistic limits

2 Alternative treatment options 3 The risks and benefits of the treatment 4 The risks involved in refusing the treatment 5 The nature of the problem requiring the treatment

Before a treatment requiring informed consent can be performed, what information must the client be given? (Select all that apply.)

2. Alternative treatment options 3. The risks and benefits of the treatment 4.The risks involved in refusing the treatment 5. The nature of the problem requiring the treatment

Before a treatment requiring informed consent can be performed, what information must the client be given? (Select all that apply.) 1. The cost of the treatment 2. Alternative treatment options 3. The risks and benefits of the treatment 4.The risks involved in refusing the treatment 5. The nature of the problem requiring the treatment

A client is admitted to a psychiatric hospital after a month of unusual behavior that has included eating and sleeping very little, talking and singing constantly, and going on frequent shopping sprees. In the hospital, the client is demanding, bossy, and sarcastic. Which disorder does the nurse associate with these behaviors?

Bipolar disorder, manic phase

An adolescent with a major depressive disorder is prescribed venlafaxine (Effexor). What signs or symptoms related to the medication should the nurse communicate immediately to the prescribing provider? (Select all that apply.)

Blurred vision Suicidal ideation Difficult urination

A nurse is working in a clinic that provides services to clients who abuse drugs. What effect of cocaine should the nurse consider as the reason that it easily causes dependence?

Blurs reality

Clients believe that what they feel that they are experiencing is real. Failure to accept the client and the client's fears is a barrier to effective communication. Today mental health therapy is directed toward returning the client to the community as rapidly as possible. Electroconvulsive therapy is not the treatment of choice for clients with schizophrenia. Family cooperation is helpful but not an absolute necessity.

Breaks with reality such as those experienced by clients with schizophrenia require the nurse to understand that: Extended institutional care is necessary. Clients believe that what they feel that they are experiencing is real. Electroconvulsive therapy produces remission in most clients with schizophrenia. The clients' families must cooperate in the maintenance of the psychotherapeutic plan.

Withdrawal, regressed behavior, and lack of social skills Withdrawal, regressed behavior, and lack of social skills are classic behaviors exhibited by clients with a diagnosis of schizophrenia. Disorientation, forgetfulness, and anxiety are more commonly associated with dementia. Grandiosity, arrogance, and distractibility are more commonly associated with bipolar disorder, manic phase. Slumped posture, pessimistic outlook, and flight of ideas are more commonly associated with depression.

By identifying behaviors commonly exhibited by the client who has a diagnosis of schizophrenia, the nurse can anticipate: Disorientation, forgetfulness, and anxiety Grandiosity, arrogance, and distractibility Withdrawal, regressed behavior, and lack of social skills Slumped posture, pessimistic outlook, and flight of ideas

How can a nurse minimize agitation in a disturbed client?

By limiting unnecessary interactions with the client

A client newly admitted to the psychiatric unit because of an acute psychotic episode is actively hallucinating. The admitting nurse has documented the content of the auditory hallucinations, which center on the theme of powerlessness. Later the primary nurse approaches the client, who appears to be listening to voices, and comments, "You seem to be listening to something. Tell me what you hear." The primary nurse requests feedback from the psychiatric clinical specialist regarding this nursing intervention. How should the clinical specialist respond?

By reminding the nurse that once the content is known, there is no need to focus on the hallucinations because doing so reinforces them

A nurse identifies the establishment of trust as a major nursing goal for a depressed client. How can this goal best be accomplished?

By visiting frequently for short periods with the client each day

A nurse is caring for a client with the diagnosis of somatoform disorder, conversion type. What type of affect does the nurse expect this client to exhibit? (Select all that apply.)

Calm Matter-of-fact (In their head)

1. "Was reasonable care provided?" 2. "Was there a breach of nursing duty?" 3. "Was there an act of omission that resulted in harm? 4. "Except for the nurse's action, would the injury have occurred?"

Certain questions are applicable in determining nursing negligence. (Select all that apply.) 1. "Was reasonable care provided?" 2. "Was there a breach of nursing duty?" 3. "Was there an act of omission that resulted in harm? 4. "Except for the nurse's action, would the injury have occurred?" 5. "Did the nurse fully understand the actions would result in harm?"

A nurse is teaching a client about side effects of medications. Which drug will cause a heightened skin reaction to sunlight?

Chlorpromazine

It reduces their feelings of guilt

Clients addicted to alcohol often use the defense mechanism of denial. What is the reason that this defense is so often used?

As a nurse enters a room and approaches a client who has schizophrenia, the client shouts, "Get out of here before I hit you! Go away!" The nurse concludes that this aggressive behavior is probably related to the fact that the client felt:

Confined when the nurse walked into the room

A nurse is assisting with the administration of electroconvulsive therapy (ECT) to a severely depressed client. What side effect of the therapy should the nurse anticipate?

Confusion immediately after the treatment

A 30-year-old woman reports to the mental health clinic on the recommendation of her primary health care provider. She has been unable to carry out everyday activities because of increased pain in her lower back and legs. Numerous neurological and orthopedic workups indicate that her symptoms seem excessive when compared with the physical problems shown on physical examination and repeated MRIs and x-rays. She says that no one understands how difficult it has been to care for her 32-year-old husband, who has an inoperable brain tumor and is undergoing chemotherapy. In light of the history and symptoms, what disorder should the nurse suspect?

Conversion

Despite repeated nursing interventions to improve reality orientation, a client insists that he is the commander of an alien spaceship. What is the client experiencing?

Delusion

A client with the diagnosis of bulimia nervosa, purging type, is admitted to the mental health unit after an acute episode of bingeing. Which clinical manifestation is most important for the nurse to evaluate?

Dehydration

People who are involved in a bioterrorism attack exhibit immediate reactions to the traumatic event. Which responses can a nurse expect in survivors during the period immediately following a traumatic event? (Select all that apply.)

Denial Confusion Helplessness

A client is using ritualistic behaviors. Why should a nurse give the client ample time in which to perform the ritual?

Denial of this activity may precipitate a panic level of anxiety.

A client with obsessive-compulsive disorder performs a specific ritual. Why should the nurse give the client time to perform the ritual?

Denying this activity may precipitate an increased level of anxiety.

A female client who is severely incapacitated by obsessive-compulsive behavior has been admitted to the mental health hospital. The client's compulsive ritual involves changing her clothing 8 to 12 times a day. She continually asks the nurse for advice regarding her problems but then ignores it. This is an example of the conflict of:

Dependence versus independence

A client with phobias about elevators and large crowds comes to the clinic for help because of feelings of depression related to these fears. What is an appropriate short-term goal for this client?

Describing the thoughts and feelings experienced in terrifying situations

Delusion -A delusion is a fixed false belief. An illusion is a false sense interpretation of an external stimulus. Confabulation is the client's attempt to fill gaps in memory with imaginary events. A hallucination is a false sensory perception with no external stimulus.

Despite repeated nursing interventions to improve reality orientation, a client insists that he is the commander of an alien spaceship. What is the client experiencing?

What is the most difficult initial task in the development of a nurse-client relationship?

Developing an awareness of self and the professional role in the relationship

A client who is on the third day of detoxification therapy becomes agitated and restless. What are the signs and symptoms that indicate impending alcohol withdrawal delirium? (Select all that apply.)

Diaphoresis Tachycardia Hypertension

A nurse is caring for a client with vascular dementia. What does the nurse expect of this client's mental status?

Difficulty recalling recent events related to cerebral hypoxia

A nurse is caring for a client with the diagnosis of schizophrenia. What is a common problem for clients with this diagnosis?

Disordered thinking

A client's history demonstrates a pervasive pattern of unstable and intense relationships, impulsiveness, inappropriate anger, manipulation, offensive behavior, and hostility. The admitting diagnosis is borderline personality disorder. What does the nurse anticipate that this client may attempt to do?

Divide the staff into opposing factions to gain self-esteem

A nurse has been assigned to care for a client with the diagnosis of obsessive-compulsive disorder (OCD). Before providing care for this client, the nurse should consider that clients with OCD:

Do not want to repeat the ritual but feel compelled to do so

A parent of a 17-year-old girl who has been hospitalized for extremely disturbed acting-out behavior leaves a gift for the daughter but says, "I'm too busy to visit today." The daughter becomes upset and tearful after being given the message and opening the package. What does the nurse conclude that the parent's actions represent?

Double-bind message

4 Retention of urine 5 Thoughts of suicide

Doxepin (Sinequan) is prescribed for a 74-year-old man for treatment of a depressive episode that has not responded to several other medications. The nurse in the outpatient clinic reviews with the client the side effects of doxepin. The identification of which side effects by the client as needing to be reported to the health care provider allows the nurse to conclude that the teaching has been effective? (Select all that apply.)

4.Retention of urine 5.Thoughts of suicide

Doxepin (Sinequan) is prescribed for a 74-year-old man for treatment of a depressive episode that has not responded to several other medications. The nurse in the outpatient clinic reviews with the client the side effects of doxepin. The identification of which side effects by the client as needing to be reported to the health care provider allows the nurse to conclude that the teaching has been effective? (Select all that apply.) 1.Diarrhea 2.Loss of appetite 3.Photosensitivity 4.Retention of urine 5.Thoughts of suicide

Stressors that appear to precipitate the client's disruptive behavior

During the first month in a nursing home, an older client with dementia demonstrates numerous disruptive behaviors related to disorientation and cognitive impairment. What should the nurse take into consideration when planning care?

"You're upset because you think we're blaming you for her death?"

During a group discussion regarding the unexpected suicide of a young female client who was on a weekend pass, one of the other clients stands up and shouts, "Oh, I know what you're all thinking. You think that I should've known that she was going to kill herself. You think I helped her plan this." What is the most therapeutic response by the group leader?

"Maybe others in the group have similar feelings that they would share."

During a group meeting a client tells everyone, "I'm about to be discharged from the hospital, and I'm afraid." What is the most appropriate response by the nurse facilitator?

"What's happening right now that prompted you to call?"

During a phone conversation to a crisis hotline a client states, "I'm falling apart and can't put myself together. This goes on and on." What is the most therapeutic response by the nurse?

4."What's happening right now that prompted you to call?"

During a phone conversation to a crisis hotline a client states, "I'm falling apart and can't put myself together. This goes on and on." What is the most therapeutic response by the nurse? 1."Is there anyone there with you?" 2."What do you think this means?" 3."How do you usually handle this type of situation?" 4."What's happening right now that prompted you to call?"

"You seem to be afraid that you'll hurt yourself."

During a special meeting to discuss the unexpected suicide of a recently discharged client, a nurse overhears another client moan softly, "I'm next. Oh my God, I'm next. They couldn't protect him, and they can't protect me, either." What is the most therapeutic response by the nurse?

Anxious over the arrival of new staff members -The client's behavior demonstrates increased anxiety. Because it was directed toward the new staff, it was probably precipitated by their arrival.

During the orientation tour for three new staff members, a young, hyperactive manic client greets them by saying, "Welcome to the funny farm. I'm Jo-Jo, the head yo-yo." Which meaning can the nurse assign to the client's statement?

A nurse understands that autism is a form of a pervasive developmental disorder (PDD). Which factor unique to autism differentiates it from other forms of PDD?

Early onset, before 36 months of age

A nurse recalls that language development in the autistic child resembles:

Echolalia

What treatment should a nurse anticipate will be prescribed for a client with severe, persistent, intractable depression and suicidal ideation?

Electroconvulsive therapy

Clinical depression -ECT is used to treat clinical depression in clients who do not respond well to a trial of psychotropic medications or are so severely depressed that immediate intervention is needed.

Electroconvulsive therapy (ECT) is a mode of treatment that is used primarily to treat:

The day after the birth of their baby, the parents are upset to learn that the baby has a heart defect. At this time it is most helpful for the nurse to:

Encourage the expression of their feelings

A client is brought to the emergency department by friends because of increasingly bizarre behavior. Which signs does the nurse identify that indicate that the client was using cocaine? (Select all that apply.)

Euphoria Agitation Hypervigilance Impaired judgment

The client has gained 6 lb since admission 3 weeks ago. -Weight gain of 6 lb since admission 3 weeks ago is objective proof that the client's eating behaviors have improved

Evaluation of clients with anorexia nervosa requires reassessment of behaviors after admission. Which finding indicates that the therapy is beginning to be effective?

A nurse knows that children with attention deficit-hyperactivity disorder (ADHD) may be learning disabled. This means that these children:

Experience perceptual difficulties that interfere with learning

Risk for assaultive behavior is highest in the mental health client who:

Experiences command hallucinations

A client has just been admitted with the diagnosis of borderline personality disorder. There is a history of suicidal behavior and self-mutilation. The nurse remembers that the main reason that clients use self-mutilation is to:

Express anger or frustration

A client with a history of schizophrenia attends the mental health clinic for a regularly scheduled group therapy session. The client arrives agitated and exhibits behaviors that indicates that she is hearing voices. When a nurse begins to walk toward her, the client pulls out a large knife. What is the best approach by the nurse?

Firm

A client with a diagnosis of schizophrenia is discharged from the hospital. At home the client forgets to take the medication, is unable to function, and must be rehospitalized. What medication may be prescribed that can be administered on an outpatient basis every 2 to 3 weeks?

Fluphenazine

A client is admitted to the psychiatric hospital with a diagnosis of obsessive-compulsive disorder. The client's anxiety level is approaching a panic level, and the client's ritual is interfering with work and daily living. Which selective serotonin reuptake inhibitor (SSRI) should the nurse anticipate that the health care provider may prescribe?

Fluvoxamine (Luvox)

A nurse is caring for a client who uses ritualistic behavior. What common antiobsessional medication does the nurse anticipate will be prescribed?

Fluvoxamine (Luvox)

The nurse is caring for a client with Alzheimer's disease who exhibits behaviors associated with hyperorality. To meet the client's need for a safe milieu, the nurse will instruct staff to monitor the client:

For attempts at eating inedible objects

Yellow sclerae -Yellow sclerae are a sign of jaundice; pancreatitis and hepatic failure are life-threatening adverse effects of valproic acid (Depakene).

For which adverse effect should the nurse continually observe a client who is receiving valproic acid (Depakene)?

1.Yellow sclerae

For which adverse effect should the nurse continually observe a client who is receiving valproic acid (Depakene)? 1.Yellow sclerae 2.Motor restlessness 3.Ringing in the ears 4.Torsion of the neck

1. Yellow sclerae **Yellow sclerae are a sign of jaundice; pancreatitis and hepatic failure are life-threatening adverse effects of valproic acid (Depakene).

For which adverse effect should the nurse continually observe a client who is receiving valproic acid (Depakene)? 1. Yellow sclerae 2. Motor restlessness 3. Ringing in the ears 4. Torsion of the neck

Lack of eye contact -Children with autism usually have a pervasive impairment of reciprocal social interaction. Lack of eye contact is a typical behavior associated with autism.

For which clinical indication should a nurse observe a child in whom autism is suspected?

An older adult with a diagnosis of delirium on the mental health unit begins acting out while in the dayroom. What is the initial nursing intervention?

Giving the client one simple direction at a time in a firm low-pitched voice

A nurse is caring for several clients with the diagnosis of bulimia nervosa. What primary feeling does the nurse anticipate that these clients experience after an episode of bingeing?

Guilt

A client with the diagnosis of schizophrenia refuses to eat meals. Which nursing action is most beneficial for this client?

Having a staff member sit with the client in a quiet area during mealtimes

What is an appropriate way for a nurse to help a client ease anxiety?

Help the client acquire skills with which to face stressful events

What is the most appropriate way for the nurse to help a severely depressed adolescent client accept the realities of daily living?

Helping the client fulfill personal hygiene needs

A client is admitted to a psychiatric hospital with the diagnosis of schizoid personality disorder. Which initial nursing intervention is a priority for this client?

Helping the client learn to trust the staff through selected experiences

A nurse uses behavior modification to foster toilet-training efforts in a cognitively impaired child. What reward should the nurse provide to reinforce appropriate use of the toilet?

Hug with praise

A client with schizophrenia is admitted to an acute care psychiatric unit. Which clinical findings indicate positive signs and symptoms of schizophrenia?

Hyperactivity, auditory hallucinations, loose associations

A client tells the nurse, "That man on the television is talking only to me." What should the nurse document that the client is exhibiting?

Idea of reference

A client with schizophrenia sees a group of visitors sitting together talking. The client tells the nurse, "I know they're talking about me." Which altered thought process should the nurse identify?

Ideas of reference

A college student visits the health center and describes anxiety about having to declare an academic major. What developmental conflict, according to Erikson, is this client still attempting to resolve?

Identity versus role confusion

A young client is admitted to the hospital with a diagnosis of acute schizophrenia. The family reports that one day the client looked at a linen sheet on a clothesline and thought it was a ghost. What is the most appropriate conclusion to make about what the client was experiencing?

Illusion

A client in the mental health clinic has a phobia about closed spaces. Which desensitization method should the nurse expect to be used successfully with this client?

Imagery

Having the client checked for increased intraocular pressure and teaching about symptoms of glaucoma

Imipramine (Tofranil), 75 mg three times per day, is prescribed for a client. What nursing action is appropriate when this medication is being administered?

4.Having the client checked for increased intraocular pressure and teaching about symptoms of glaucoma

Imipramine (Tofranil), 75 mg three times per day, is prescribed for a client. What nursing action is appropriate when this medication is being administered? 1.Telling the client that barbiturates and steroids will not be prescribed 2.Warning the client not to eat cheese, fermented products, and chicken liver 3.Monitoring the client for increased tolerance and reporting when the dosage is no longer effective 4.Having the client checked for increased intraocular pressure and teaching about symptoms of glaucoma

4. Having the client checked for increased intraocular pressure and teaching about symptoms of glaucoma **Glaucoma is one of the side effects of imipramine (Tofranil), and the client should be taught the symptoms.

Imipramine (Tofranil), 75 mg three times per day, is prescribed for a client. What nursing action is appropriate when this medication is being administered? 1. Telling the client that barbiturates and steroids will not be prescribed 2. Warning the client not to eat cheese, fermented products, and chicken liver 3. Monitoring the client for increased tolerance and reporting when the dosage is no longer effective 4. Having the client checked for increased intraocular pressure and teaching about symptoms of glaucoma

When a nurse sits next to a depressed client and begins to talk, the client responds, "I'm stupid and useless. Talk with the other people who are more important." Which response is most therapeutic?

Imitating and participating in the child's activities

A widow who is hospitalized for a medical problem has dementia of the Alzheimer type and is no longer able to live alone. The client is to be transferred from the hospital to a long-term care facility. When should the staff begin preparation for the transfer?

Immediately after the client's admission to the hospital

A depressed client is admitted to the mental health unit. What factor should the nurse consider most important when evaluating the client's current risk for suicide?

Impending anniversary of the loss of a loved one

A client is found to have a borderline personality disorder. What behavior does the nurse consider is most typical of these clients?

Impulsive

A 4-year-old child is found to have attention deficit-hyperactivity disorder (ADHD). What information about the child's behavior should the nurse expect when obtaining a health history from the parents? (Select all that apply.)

Impulsiveness Excessive talking Playing video games for hours on end Failure to follow through or finish tasks

Seek to understand what the behavior means to the client -All behavior has meaning; before planning intervention, the nurse must try to understand what the behavior means to the client.

In addition to hallucinating, a client yells and curses throughout the day. The nurse should:

Empathy

In an attempt to remain objective and support a client during a crisis, the nurse uses imagination and determination to project the self into the client's emotions. This technique is known as:

Delusional thinking

In her eighth month of pregnancy, a 24-year-old client is brought to the hospital by the police, who were called when she barricaded herself in a ladies' restroom of a restaurant. During admission the client shouts, "Don't come near me! My stomach is filled with bombs, and I'll blow up this place if anyone comes near me." The nurse concludes that the client is exhibiting:

"I shouldn't want that."

In response to a question posed during a group meeting, the nurse explains that the superego is that part of the self that says:

3."I shouldn't want that."

In response to a question posed during a group meeting, the nurse explains that the superego is that part of the self that says: 1."I like what I want." 2."I want what I want." 3."I shouldn't want that." 4."I can wait for what I want."

3. "I should not want that." Rational Conscience and a sense of right and wrong are expressed in the superego, which acts to counterbalance the id's desire for immediate gratification.

In response to a question posed during a group meeting, the nurse explains that the superego is that part of the self that says: 1. "I like what I want." 2. "I want what I want." 3. "I should not want that." 4. "I can wait for what I want."

3. "I shouldn't want that." **Conscience and a sense of right and wrong are expressed in the superego, which acts to counterbalance the id's desire for immediate gratification.

In response to a question posed during a group meeting, the nurse explains that the superego is that part of the self that says: 1. "I like what I want." 2. "I want what I want." 3. "I shouldn't want that." 4. "I can wait for what I want."

4. Defense mechanisms **When the individual experiences a threat to self-esteem, anxiety increases, and defense mechanisms are used to protect the self

In the process of development the individual strives to maintain, protect, and enhance the integrity of the self. The nurse determines that this is usually accomplished through the use of: 1. Affective reactions 2. Withdrawal patterns 3. Ritualistic behaviors 4. Defense mechanisms

The client has expressed severe suicidal thoughts.

In which situation is the use of seclusion contraindicated?

1.The client has expressed severe suicidal thoughts.

In which situation is the use of seclusion contraindicated? 1.The client has expressed severe suicidal thoughts. 2.The client appears to want to be placed in seclusion. 3.The client has been voluntarily admitted for treatment. 4.The client had minimal improvement despite being secluded before

Repression

Incidences of child molestation often are revealed years later when the victim is an adult. Which defense mechanism reflects this situation?

2.Repression Rationale: 2. Repression is a coping mechanism in which unacceptable feelings are kept out of conscious awareness; later, under stress or anxiety, thoughts or feelings surface and come into one's conscious awareness.

Incidents of child molestation often are revealed years later when the victim is an adult. Which defense mechanism reflects this situation? 1.Isolation 2.Repression 3.Regression 4.Introjection

A client who is to begin a physical therapy regimen after orthopedic surgery expresses anxiety about starting this new therapy. The nurse responds that some of this apprehension can be an asset because it will:

Increase alertness to the environment

How should a nurse expect a client's anxiety to be manifested physiologically?

Increased blood glucose level

A client has been hospitalized for 3 weeks while receiving a tricyclic medication for severe depression. One day the client says to the nurse, "I'm really feeling better; my energy level is up." After the encounter an aide tells the nurse that the client has given away his favorite jacket. What should the nurse conclude that the client's statement indicates?

Increased risk for suicide

A nurse has been caring for a suicidal client for 3 weeks on an inpatient unit. One morning the client greets the nurse cheerfully and states, "Everything is looking up. I'm not going to have problems for very long." What does the client's behavior and statement indicate?

Increased risk of suicide

The clients on a mental health unit go on a supervised day trip to a baseball game. When returning to the bus, a client with a narcissistic personality disorder insists on leaving the group to get an autograph from a player. What is the most appropriate response by the nurse?

Informing the client in a matter-of-fact tone that everyone must remain with the group

What conflict associated with Erikson's psychosocial stages of development should the nurse remember when caring for a client 30 years of age?

Intimacy versus isolation

A client with a history of alcoholism is found to have Wernicke encephalopathy associated with Korsakoff syndrome. What does the nurse anticipate will be prescribed?

Intramuscular injections of thiamine

A depressed client is very resistive and complains about inabilities and worthlessness. The best nursing approach is to:

Involve the client in activities in which success can be ensured

A nurse is caring for a hyperactive, manic client who exhibits flight of ideas and is not eating. What may be the reason why the client is not eating?

Is too busy to take the time to eat

A nurse is administering medications to clients on a psychiatric unit. What does the nurse identify as the reason that so many psychiatric clients are given the drug benztropine (Cogentin) or trihexyphenidyl in conjunction with the phenothiazine derivatives neuroleptic medications?

It combats the extrapyramidal side effects of the other drug.

What should nurses consider when working with depressed young children?

It is important to include the family in the treatment plan.

A hospitalized, depressed, suicidal client has been taking a mood-elevating medication for several weeks. The client's energy is returning and the client no longer talks about suicide. What should the nurse do in response to this client's behavior?

Keep the client under closer observation

A nurse is caring for a young, hyperactive child with attention deficit-hyperactivity disorder who engages in self-destructive behavior. What is the most important nursing objective in the planning of care for this child?

Keeping the child from inflicting any self-injury

At night an older client with dementia sleeps very little and becomes more disoriented. How can the nurse best limit this confusion resulting from sleep deprivation?

Leaving a dim light on in the client's room at night

A client tells a nurse, "I have been having trouble sleeping and feel wide awake as soon as I get into bed." Which strategies should the nurse teach the client that will promote sleep? (Select all that apply.)

Leaving the bedroom when unable to sleep Exercising in the afternoon rather than in the evening Counting backward from 100 to 0 when his mind is racing

When a newly admitted client with paranoid ideation tells the nurse about people coming through the doors to commit murder, the nurse should:

Listen to what the client is saying

A nurse is interacting with a depressed, suicidal client. What themes in the client's conversation are of most concern to the nurse? (Select all that apply.)

Loneliness Hopelessness

A client with schizophrenia says to the nurse, "I've been here 5 days. There are 5 players on a basketball team. I like to play the piano." How should the nurse document this cognitive disorder?

Loose association

A nurse is caring for an adolescent client with the diagnosis of schizophrenia, undifferentiated type. Which signs and symptoms should the nurse expect the client to experience?

Loosened associations and hallucinations

The nurse anticipates that the medication that will be used to prevent symptoms of withdrawal in clients with a long history of alcohol abuse is:

Lorazepam (Ativan)

A client is found to have an adjustment disorder with mixed anxiety and depression. What should the nurse anticipate as the client's primary problem?

Low self-esteem

A client in a mental health facility is demonstrating manic-type behavior by being demanding and hyperactive. What is the nurse's major objective?

Maintaining a supportive, structured environment

A client has been in an acute care psychiatric unit for 3 days and is receiving haloperidol (Haldol) tablets orally to reduce agitation and preoccupation with auditory hallucinations. There has been no decrease in the client's agitation or preoccupation with auditory hallucinations since the medication was started. What should the nurse's priority intervention be?

Making certain that the client is swallowing the medication

A nurse determines that a client is pretending to be ill. What does this behavior usually indicate?

Malingering

A client with dementia has been cared for by the spouse for 5 years. During the last month the client has become agitated and aggressive and is incontinent of urine and feces. What is the priority nursing care while this client is in an inpatient mental health facility?

Managing the behavior

A client with a history of substance abuse is brought to the emergency department. The client is having seizures, is hypertensive, and has hyperthermia. What drug should the nurse consider that the client may have been abusing?

Methamphetamine

Paradoxical effect

Methylphenidate (Ritalin) is prescribed to treat a 7-year-old child's attention deficit-hyperactivity disorder (ADHD). The nurse understands that methylphenidate is used in the treatment of this disorder in children for its:

3. Paradoxical effect **Methylphenidate (Ritalin), a stimulant, has an opposite effect on hyperactive children; the reason for this action is as yet totally unexplained

Methylphenidate (Ritalin) is prescribed to treat a 7-year-old child's attention deficit-hyperactivity disorder (ADHD). The nurse understands that methylphenidate is used in the treatment of this disorder in children for its: 1. Diuretic effect 2. Synergistic effect 3. Paradoxical effect 4. Hypotensive effect

A student is anxious about an upcoming examination but is able to study intently and does not become distracted by a roommate's talking and loud music. What level of anxiety is demonstrated by the student's ability to shut out the distractions?

Mild

What should a nurse identify as the most important factor in rehabilitation of a client addicted to alcohol?

Motivational readiness

A delusional client verbalizes the belief that others are out to harm him. A nurse notes the client's worsening pacing and agitation. What is the best nursing intervention?

Moving the client to a quiet place on the unit

A client with schizophrenia repeatedly says to the nurse, "No moley, jandu!" The nurse determines that the client is exhibiting:

Neologism

2 Diaphoresis 3 Hyperrigidity 4 Hyperthermia

Neuroleptic malignant syndrome is a potentially fatal reaction to antipsychotic therapy. What signs and symptoms of this syndrome should the nurse identify? (Select all that apply.)

Diaphoresis Hyperrigidity Hyperthermia

Neuroleptic malignant syndrome is a potentially fatal reaction to antipsychotic therapy. What signs and symptoms of this syndrome should the nurse identify? (Select all that apply.)

2. Diaphoresis 3. Hyperrigidity 4. Hyperthermia **Diaphoresis, hyperrigidity, and hyperthermia occur with neuroleptic malignant syndrome as a result of dopamine blockade in the hypothalamus.

Neuroleptic malignant syndrome is a potentially fatal reaction to antipsychotic therapy. What signs and symptoms of this syndrome should the nurse identify? (Select all that apply.) 1. Jaundice 2. Diaphoresis 3. Hyperrigidity 4. Hyperthermia 5. Photosensitivity

A client who attempted suicide by slashing her wrists is transferred from the emergency department to a mental health unit. What important nursing interventions must be implanted when the client arrives on the unit? (Select all that apply.)

Obtaining vital signs Assessing for suicidal thoughts Instituting continuous monitoring Initiating a therapeutic relationship Inspecting the bandages for bleeding

On the fifth day of hospitalization the nurse notes that a depressed client remains lying on her bed when the clients are called to the dining room for lunch. What should the nurse do to encourage the client to eat?

Offer to accompany the client to the dining room

How should the nursing staff fulfill the nutritional needs of a client experiencing periods of extreme mania and hyperactivity?

Offering high-calorie snacks frequently that the client can hold

Sit up slowly. -Olanzapine (Zyprexa), a thienobenzodiazepine, can cause orthostatic hypotension. Blurred, not double, vision may occur. Decreased salivation is an effect of Olanzapine. It may also cause nausea and other gastrointestinal upsets and should be taken with fluid or food.

Olanzapine (Zyprexa) is prescribed for a client with bipolar disorder, manic episode. What cautionary advice should the nurse give the client?

Stating that this behavior is unacceptable When clients enter a new milieu, limits should be set on unacceptable behavior and acceptable behavior should be reinforced. Neither clients nor unacceptable behavior should ever be ignored. Moving the client to his room for a short time-out is punishment. Unacceptable attention-getting behavior must be addressed immediately; also, the focus should be on appropriate behavior

On the afternoon of admission to a psychiatric unit, an adolescent boy with the diagnosis of schizophrenia exposes his genitals to a female nurse. What should the nurse's immediate therapeutic response be? Ignoring the client at this time Stating that this behavior is unacceptable Moving him to his room for a short time-out Telling the client to come to the office later to discuss the behavior

"We're concerned that you might try to harm yourself."

On the day after admission a suicidal client asks a nurse, "Why am I being watched around the clock, and why can't I walk around the whole unit?" Which reply is most appropriate?

Day 13

On the first day of the month a practitioner prescribes an antipsychotic medication for a client with schizophrenia. The initial dosage is 25 mg once a day, to be titrated in increments of 25 mg every other day to a desired dosage of 175 mg daily. On what day of the month will the client reach the desired daily dose of 175 mg?

3.Day 13

On the first day of the month a practitioner prescribes an antipsychotic medication for a client with schizophrenia. The initial dosage is 25 mg once a day, to be titrated in increments of 25 mg every other day to a desired dosage of 175 mg daily. On what day of the month will the client reach the desired daily dose of 175 mg? 1.Day 7 2.Day 9 3.Day 13 4.Day 15

3. Day 13 **The client will reach the desired dosage of 175 mg on the 13th day of the month; on the first day it is 25 mg, on the third day it is 50 mg, on the fifth day it is 75 mg, on the seventh day it is 100 mg, on the ninth day it is 125 mg, on the 11th day it is 150 mg, and on the 13th day it is 175 mg.

On the first day of the month a practitioner prescribes an antipsychotic medication for a client with schizophrenia. The initial dosage is 25 mg once a day, to be titrated in increments of 25 mg every other day to a desired dosage of 175 mg daily. On what day of the month will the client reach the desired daily dose of 175 mg? 1. Day 7 2. Day 9 3. Day 13 4. Day 15

Akathisia -Akathisia , a side effect of haloperidol (Haldol), develops early in therapy and is characterized by restlessness and agitation.

On the psychiatric unit a client has been receiving high doses of haloperidol (Haldol) for 2 weeks. The client says, "I just can't sit still, and I feel jittery." Which side effect does the nurse suspect that the client is experiencing?

1. Akathisia **Akathisia, a side effect of haloperidol (Haldol), develops early in therapy and is characterized by restlessness and agitation.

On the psychiatric unit a client has been receiving high doses of haloperidol (Haldol) for 2 weeks. The client says, "I just can't sit still, and I feel jittery." Which side effect does the nurse suspect that the client is experiencing? 1. Akathisia 2. Torticollis 3. Tardive dyskinesia 4. Parkinsonian syndrome

Administering the prescribed medication to the client to subdue the agitated behavior -The nurse must administer the prescribed medication to the client to subdue the agitated behavior in this life-threatening situation. The client's central nervous system (CNS) is overstimulated, and seizures and death can occur. CNS-depressant medications, usually benzodiazepines, are needed to blunt the withdrawal effects.

On the third day of hospitalization, a client with a history of heavy drinking begins experiencing alcohol withdrawal delirium. What is the most appropriate response by the nurse when the client begins experiencing hallucinations?

There is a strong correlation between successful retirement and good health.

On which principle should the nurse's role be based in the maintenance or promotion of the health of older adults?

Approaching the client in a nonthreatening manner to determine the cause of the agitation

One afternoon a nurse sees a client rushing down the hall of the mental health unit, rapidly tapping his fingers against the wall. What is the most appropriate nursing action at this time?

1.Approaching the client in a nonthreatening manner to determine the cause of the agitation

One afternoon a nurse sees a client rushing down the hall of the mental health unit, rapidly tapping his fingers against the wall. What is the most appropriate nursing action at this time? 1.Approaching the client in a nonthreatening manner to determine the cause of the agitation 2.Summoning additional staff members to forcefully subdue the client and stop the acting-out behavior 3.Observing the client to see whether the behavior escalates and whether it may pose a risk to other clients or staff 4.Immediately obtaining staff assistance to enable administration of medication prescribed for the client's agitatio

1. Approaching the client in a nonthreatening manner to determine the cause of the agitation

One afternoon a nurse sees a client rushing down the hall of the mental health unit, rapidly tapping his fingers against the wall. What is the most appropriate nursing action at this time? 1. Approaching the client in a nonthreatening manner to determine the cause of the agitation 2. Summoning additional staff members to forcefully subdue the client and stop the acting-out behavior 3. Observing the client to see whether the behavior escalates and whether it may pose a risk to other clients or staff 4. Immediately obtaining staff assistance to enable administration of medication prescribed for the client's agitation

Displacement -Displacement reduces anxiety by transferring the emotions associated with an object or person to another emotionally safer object or person.

One afternoon the nurse on the unit overhears a young female client having an argument with her boyfriend. A while later the client complains to the nurse that dinner is always late and the meals are terrible. The nurse identifies the defense mechanism that the client is using as:

"When I look at you I see a person, not a devil."

One day the nurse and a young adult client sit together and draw. The client draws a face with horns and says, "This is me. I'm a devil." What is the best response by the nurse?

"You're frightened. Come with me to your room, and we can talk about it."

One evening a nurse finds a client who has been experiencing persecutory delusions trying to get out the door. The client begs, "Please let me go. I trust you. The Mafia is going to kill me tonight." Which response is most therapeutic?

Depersonalization =The state in which the client feels unreal or believes that parts of the body are distorted is known as depersonalization or loss of personal identity.

One morning a client tells the nurse, "My legs are turning to rubber because I have an incurable disease called schizophrenia." The nurse identifies that this as an example of:

Depersonalization The state in which the client feels unreal or believes that parts of the body are distorted is known as depersonalization or loss of personal identity. This is not an example of a hallucination; a hallucination is a sensory experience for which there is no external stimulus. The client's statement does not indicate any feelings that others are out to do harm, are responsible for what is happening, or are in control of the situation. The statement is not an example of autistic verbalization.

One morning a client tells the nurse, "My legs are turning to rubber because I have an incurable disease called schizophrenia." The nurse identifies that this as an example of: Hallucinations Paranoid thinking Depersonalization Autistic verbalization

"You really seem to be upset about this."

One morning a client with the diagnosis of acute depression says to the nurse, "God is punishing me for my past sins." What is the best response by the nurse?

The client is feeling more anxious today.

One morning a nurse on the psychiatric unit finds a client curled up in the fetal position in the corner of the dayroom. What is an appropriate initial inference for the nurse to make about the client?

Administering chlordiazepoxide as indicated by the client's CIWA score

Oral chlordiazepoxide (Librium) 100 mg/ hr is prescribed for a client with a Clinical Institute Withdrawal Assessment (CIWA) score of 25. The client has had 300 mg in 3 hours but is still displaying acute alcohol withdrawal symptoms. What is the next nursing action?

2.Administering chlordiazepoxide as indicated by the client's CIWA score

Oral chlordiazepoxide (Librium) 100 mg/ hr is prescribed for a client with a Clinical Institute Withdrawal Assessment (CIWA) score of 25. The client has had 300 mg in 3 hours but is still displaying acute alcohol withdrawal symptoms. What is the next nursing action? 1.Informing the client that the limit of chlordiazepoxide has been reached 2.Administering chlordiazepoxide as indicated by the client's CIWA score 3.Requesting a prescription for another medication to replace the chlordiazepoxide. 4.Informing the health care provider that the maximum dose of chlordiazepoxide has been reached

2. Administering chlordiazepoxide as indicated by the client's CIWA score **Medication of clients in acute withdrawal from alcohol should be based on withdrawal symptoms, not medication dosage.

Oral chlordiazepoxide (Librium) 100 mg/ hr is prescribed for a client with a Clinical Institute Withdrawal Assessment (CIWA) score of 25. The client has had 300 mg in 3 hours but is still displaying acute alcohol withdrawal symptoms. What is the next nursing action? 1. Informing the client that the limit of chlordiazepoxide has been reached 2. Administering chlordiazepoxide as indicated by the client's CIWA score 3. Requesting a prescription for another medication to replace the chlordiazepoxide. 4. Informing the health care provider that the maximum dose of chlordiazepoxide has been reached

A nurse is caring for a client during the manic phase of bipolar disorder. What should the nurse do to best help meet the nutritional needs of this client?

Order foods that the client can hold in the hand to eat while moving around

A client with schizophrenia is demonstrating waxy flexibility. Which intervention is the best way to manage the possible outcome of this behavior?

Passive range-of-motion exercises three times a day for effective joint health

2 Denial 4 Confusion 5 Helplessness -Rationale: Shock and disbelief are the initial responses to a traumatic experience; a situational crisis usually is unexpected, and its impact causes disequilibrium. Disequilibrium results in confusion, disorganization, and difficulty making decisions. When a person is unable to cope, helplessness and regression often emerge; a crisis occurs when there is a painful, frightening event that is so overwhelming an individual's usual coping mechanisms are inadequate.

People who are involved in a bioterrorism attack exhibit immediate reactions to the traumatic event. Which responses can a nurse expect in survivors during the period immediately following a traumatic event? (Select all that apply.)

A nurse concludes that a client has successfully achieved the long-term goal of mobilizing effective coping responses when the client states that when he feels himself getting anxious he will:

Perform a relaxation exercise

A client with a diagnosis of bipolar disorder, manic episode, is admitted to the mental health unit. Because the environment is important, what should the nurse do?

Place the client in a private room to provide a quiet atmosphere

A hyperactive, acting-out 9-year-old boy is started on a behavior modification program in which tokens are given for acceptable behavior. When he begins to lose a game he is playing with other children, he begins to kick the other children under the table and call them names. What is the most appropriate behavior modification technique for the nurse to use?

Placing the child in a short time-out

When intimate partner violence (IPV) is suspected, the nurse plays an important role as an advocate for the victim. The advocate role includes several important components. (Select all that apply.)

Planning for future safety Validating the experiences Promoting access to community services

A nurse is caring for a preschool-aged child with a history of physical and sexual abuse. What type of therapy will be the most advantageous for this child?

Play

The health care provider prescribes donepezil (Aricept) 5 mg by mouth once a day for a client exhibiting initial signs of dementia of the Alzheimer type. The client is already taking digoxin (Lanoxin) 0.125 mg in the morning and alprazolam (Xanax) 0.5 mg twice a day. What should the nurse teach the client's spouse to do?

Prefill a weekly drug box with the medications for the spouse to self-administer

Which activity is most appropriate for a nurse to introduce to a depressed client during the early part of hospitalization?

Project involving drawing

An older depressed person at an independent living facility constantly complains about her health problems to anyone who will listen. One day the client says, "I'm not going to any more activities. All these old crabby people do is talk about their problems." What defense mechanism does the nurse conclude that the client is using?

Projection

Which nursing intervention is indicated for a client with an anxiety disorder?

Promoting verbalization of feelings by the client

A nurse is working with an adolescent client with conduct disorder. Which strategies should the nurse implement while working on the goal of increasing the client's ability to meet personal needs without manipulating others? (Select all that apply.)

Provide physical outlets for aggressive feelings Establish a contract regarding manipulative behavior Develop activities that provide opportunities for success

What is important when the nurse plans care for a client with paranoid ideation?

Providing the client with opportunities for nonthreatening social interaction

A client with the diagnosis of alcoholism explains to the nurse that alcohol has a calming effect and states, "I function better when I'm drinking than when I'm sober." What defense mechanism does the nurse identify?

Rationalization

What should the nurse do when a client with the diagnosis of schizophrenia talks about being controlled by others?

React to the feeling tone of the client's delusion

A 30-year-old female client asks the nurse to change her room, stating that she hates her roommate and can't stand to be in the same room with her. Just as she finishes speaking, her roommate enters and the client tells her she missed her and has been all over the unit looking for her. The nurse recognizes that the client is using:

Reaction formation

A client who is hallucinating actively approaches the nurse and reports, "I'm hearing voices that are saying bad things about me." What should the nurse do?

Recognizing that the behavior is part of the illness but setting limits on it

Many clients who call a crisis hotline are extremely anxious. The nurse answering the hotline phone considers that the characteristic distinguishing posttraumatic stress disorders from other anxiety disorders is:

Reexperiencing the trauma in dreams and flashbacks

A 13-year-old boy who recently was suspended from school for consistently bullying other children is brought to the pediatric mental health clinic by his mother. The child is assessed by the psychiatrist and referred to a psychologist for psychological testing. The day after the tests are completed, the mother returns to the clinic and asks the nurse for results of the tests. The nurse should:

Refer the mother to the psychiatrist

It is determined that a staff nurse has a drug abuse problem. As an initial intervention the staff nurse should be:

Referred to the employee assistance program

A 6-year-old child has been wetting the bed at night since the birth of a baby brother. What defense mechanism is the child using to cope with the stress of accepting a new family member?

Regression

What defense mechanism should the nurse anticipate that a client with the diagnosis of schizophrenia, undifferentiated type, will most often exhibit?

Regression

What should be the nurse's first intervention in the care of a client with a generalized anxiety disorder?

Removing as many stimuli from the client's environment as possible

A 3-year-old child is found to have a pervasive developmental disorder not otherwise specified (autistic disorder). What should the nurse consider most unusual for the child to demonstrate?

Responsiveness to the parents

A 17-year-old client is admitted to the hospital because of weight loss and malnutrition, and the health care provider diagnoses anorexia nervosa. After the client's physical condition is stabilized, the provider, in conjunction with the client and parents, decides to institute a behavior-modification program. What component of behavior modification verbalized by one of the parents leads the nurse to conclude that the parent has an understanding of the therapy?

Rewarding positive behavior

A constructive but lengthy method of confronting the stress of adolescence and preventing a negative and unhealthy developmental outcome is:

Role experimentation

What should a nurse who is caring for a hospitalized older client with dementia consider before planning care?

Routines provide stability for clients with dementia.

A client is admitted to the mental health unit after attempting suicide. When a nurse approaches, the client is tearful and silent. What is the best initial nursing intervention?

Saying, "I see that you're crying. Tell me what's going on in your life, and we can work on helping you."

A 12-year-old child who has a history of school failure and destructive acting out is admitted to a child psychiatric unit with the diagnosis of conduct disorder. The youngest of three children, the child is identified by both the parents and the siblings as the family problem. The nurse recognizes the family's pattern of relating to the child as:

Scapegoating

At times a client's anxiety level is so high that it blocks attempts at communication and the nurse is unsure of what is being said. To clarify understanding, the nurse says, "Let's see whether we mean the same thing." What communication technique is being used by the nurse?

Seeking consensual validation

What is the primary concern for a nurse caring for a client who is grossly impaired by stimulants?

Seizure activity

What is the basic therapeutic tool used by the nurse to foster a client's psychological coping?

Self

A mother of a 6-year-old boy with the diagnosis of attention deficit-hyperactivity disorder (ADHD) tells the nurse that when she is reading storybooks to her son, about halfway through the story he becomes distracted, fidgets, and stops paying attention. The nurse suggests that the mother:

Shorten the rest of the story

1. Flushing 2. Headache 3. Dyspepsia - Flushing is a common central nervous system response to sildenafil (Viagra). Headache is a common central nervous system response to sildenafil. Dyspepsia is a common gastrointestinal response to sildenafil. Diarrhea, not constipation, is a common gastrointestinal response to sildenafil. Hypotension, not hypertension, is a cardiovascular response to sildenafil. It should not be taken with antihypertensives and nitrates, because drug interactions can precipitate cardiovascular collapse.

Sildenafil (Viagra) is prescribed for a man with erectile dysfunction. A nurse teaches the client about common side effects of this drug. (Select all that apply.)

At mealtime a client with schizophrenia moves to the counter to choose food but is unable to decide what to do next. The nurse, recognizing the client's ambivalence, assists by using:

Simple declarative statements

What is a therapeutic nursing action in the care of a depressed client?

Sitting down next to the client at frequent intervals

A severely depressed client is to have electroconvulsive therapy (ECT). What should a nurse include when discussing this therapy with the client?

Sleep will be induced and the treatment will not cause pain.

On the afternoon of admission to a psychiatric unit, an adolescent boy with the diagnosis of schizophrenia exposes his genitals to a female nurse. What should the nurse's immediate therapeutic response be?

Stating that this behavior is unacceptable

A depressed client has been sitting alone in a chair most of the day and displays no interest in eating. How should the nurse plan to meet this client's nutritional needs?

Stay with the client during meals

A client comes to a mental health center with severe anxiety, evidenced by crying, hand-wringing, and pacing. What should the first nursing intervention be?

Staying physically close to the client

A hyperactive 9-year-old child with a history of attention deficit-hyperactivity disorder is admitted for observation after a motor vehicle collision. On what should nursing actions be focused when the nurse is teaching about personal safety?

Talking with the child about the importance of using a seat belt

A client with the diagnosis of bipolar disorder, depressive episode, has been hospitalized on a psychiatric unit for 1 week. What is the most appropriate activity for this client?

Talking with the nurse several times during the day

The nurse refers a client to a self-help group. What does the nurse anticipate that a self-help groups such as Alcoholics Anonymous (AA) will help its members learn?

That their problems are not unique

A client with depression is to be given fluoxetine (Prozac). What precaution should the nurse consider when initiating treatment with this drug?

The blood level may not be sufficient to cause noticeable improvement for 2 to 4 weeks.

A 7-year-old boy is brought to the clinic by the mother, who tells the nurse that her child has been having trouble in school, has difficulty concentrating, and is falling behind in schoolwork since she and her husband separated 6 months ago. The mother reports that lately her child has not been eating dinner, and she often hears him crying when he is alone. What basis for these behaviors should the nurse consider?

The child may be blaming himself for his parents' breakup.

An adolescent with anorexia nervosa frequently telephones home just before mealtimes. The client uses the phone calls to avoid eating. What client behavior supports the nurse's conclusion that the nursing plan to set limits on this avoidance behavior has been effective?

The client arrives on time for meals without being told

Evaluation of clients with anorexia nervosa requires reassessment of behaviors after admission. Which finding indicates that the therapy is beginning to be effective?

The client has gained 6 lb since admission 3 weeks ago.

A client confides to the nurse, "I've been thinking about suicide lately." What conclusion should the nurse make about the client?

The client is fearful of the impulses and is seeking protection from them.

Informing the client in a matter-of-fact tone that everyone must remain with the group

The clients on a mental health unit go on a supervised day trip to a baseball game. When returning to the bus, a client with a narcissistic personality disorder insists on leaving the group to get an autograph from a player. What is the most appropriate response by the nurse?

Encourage the expression of their feelings

The day after the birth of their baby, the parents are upset to learn that the baby has a heart defect. At this time it is most helpful for the nurse to:

Empathy

What characteristic is most essential for the nurse caring for a client undergoing mental health care?

Statements by the client about the sexual assault and the rapist

The emergency department nurse is conducting an interview and assisting with the physical examination of a female sexual assault victim. What is most important for the nurse to document on this client's record?

2.Statements by the client about the sexual assault and the rapist

The emergency department nurse is conducting an interview and assisting with the physical examination of a female sexual assault victim. What is most important for the nurse to document on this client's record? 1.Observations about the client's reaction to male staff members 2.Statements by the client about the sexual assault and the rapist 3.Information about the client's previous knowledge of the rapist 4.Summary statement about the client's description of the assault and the rapist

2. Statements by the client about the sexual assault and the rapist

The emergency department nurse is conducting an interview and assisting with the physical examination of a female sexual assault victim. What is most important for the nurse to document on this client's record? 1. Observations about the client's reaction to male staff members 2. Statements by the client about the sexual assault and the rapist 3. Information about the client's previous knowledge of the rapist 4. Summary statement about the client's description of the assault and the rapist

Feelings of guilt

The grieving spouse of a client who has just died says to the nurse, "We should have spent more time together. I always felt that my work came first." What should the nurse conclude that the spouse is experiencing?

Prefill a weekly drug box with the medications for the spouse to self-administer -Clients with early dementia of the Alzheimer type usually have some short-term memory loss. A prefilled box of medications eliminates the need to determine what drugs need to be taken. Also, it provides the spouse with objective proof that the medications have or have not been taken.

The health care provider prescribes donepezil (Aricept) 5 mg by mouth once a day for a client exhibiting initial signs of dementia of the Alzheimer type. The client is already taking digoxin (Lanoxin) 0.125 mg in the morning and alprazolam (Xanax) 0.5 mg twice a day. What should the nurse teach the client's spouse to do?

4. Informing him that he should seek emergency intervention for his wife **The inability to care for herself or her infant is a significant sign that the wife is depressed and in need of immediate intervention

The husband of a woman who gave birth to a baby 2 weeks ago calls the postpartum unit at the hospital, seeking assistance for his wife. He reports that he found his wife in bed and that the baby was wet, dirty, and crying in the crib. He says, "She says she just can't do it." What is the best response by the nurse? 1. Encouraging him to express his feelings about the situation 2. Telling him to schedule an appointment with the gynecologist 3. Asking whether he can afford a home health aide for several weeks 4. Informing him that he should seek emergency intervention for his wife

What is the most important information for a nurse to teach to prevent relapse in a client with a psychiatric illness?

The need to follow the prescribed medication regimen

Fear of the other clients Because the client has paranoid feelings that other people are out to do harm, assignment to a four-bed room may be threatening. Concern about family at home seems unlikely because the disruption appears to have started with the transfer to a four-bed room. Watching for an opportunity to escape is possible but unlikely; planning an escape is usually not part of a schizophrenic pattern of behavior. Trying to work out emotional problems is possible but not likely; clients with schizophrenia have difficulty solving problems.

The night nurse reports that a young client with paranoid schizophrenia has been awake for several nights. The day nurse reviews the client's record and finds that this client did not have an interrupted sleep pattern disorder before transfer from a private room to a four-bed room 3 days ago. What factor should the nurse identify as most likely related to the client's sleeplessness? Fear of the other clients Concern about family at home Watching for an opportunity to escape Trying to work out emotional problems

Lorazepam (Ativan)

The nurse anticipates that the medication that will be used to prevent symptoms of withdrawal in clients with a long history of alcohol abuse is:

1. Lorazepam (Ativan) **Lorazepam (Ativan) is most effective in preventing the signs and symptoms associated with withdrawal from alcohol.

The nurse anticipates that the medication that will be used to prevent symptoms of withdrawal in clients with a long history of alcohol abuse is: 1. Lorazepam (Ativan) 2. Phenobarbital (Luminal) 3. Chlorpromazine (Thorazine) 4. Methadone hydrochloride (Methadone)

Desensitization involving relaxation techniques -The most successful therapy for people with phobias consists of behavior modification techniques involving desensitization.

The nurse determines that the therapy that has the highest success rate for people with phobias is:

Fits within standards accepted by one's society

The nurse explains to a nursing assistant that behavior usually is viewed and accepted as normal if it:

1.Fits within standards accepted by one's society

The nurse explains to a nursing assistant that behavior usually is viewed and accepted as normal if it: 1.Fits within standards accepted by one's society 2.Helps the person reduce the need for coping skills 3.Expresses the individual's feelings and thoughts accurately 4.Allows achievement of short-term and long-term goals by the individua

1. Fits within standards accepted by one's society

The nurse explains to a nursing assistant that behavior usually is viewed and accepted as normal if it: 1. Fits within standards accepted by one's society 2. Helps the person reduce the need for coping skills 3. Expresses the individual's feelings and thoughts accurately 4. Allows achievement of short-term and long-term goals by the individual

"I didn't hear anyone talking; come with me to your room." -The nurse is focusing on reality and trying to distract and refocus the client's attention.

The nurse finds a client with schizophrenia lying under a bench in the hall. The client says, "God told me to lie here." What is the best response by the nurse?

Active participant -To intervene in a crisis the nurse must assume a direct, active role because the client's ability to cope is lessened and help is needed to problem-solve.

The nurse is caring for a client experiencing a crisis. What role is most important for the nurse to assume when providing therapeutic crisis intervention?

3. Active participant

The nurse is caring for a client experiencing a crisis. What role is most important for the nurse to assume when providing therapeutic crisis intervention? 1. Passive listener 2. Friendly adviser 3. Active participant 4. Participant observer

For attempts at eating inedible objects -Hyperorality is the compulsive need to taste and chew inedible objects.

The nurse is caring for a client with Alzheimer's disease who exhibits behaviors associated with hyperorality. To meet the client's need for a safe milieu, the nurse will instruct staff to monitor the client:

Disruptions in cerebral blood flow, resulting in thrombi or emboli -Vascular dementia results from the sudden closure of the lumen of arterioles, causing infarction of the brain tissue in the affected area.

The nurse is caring for a client with vascular dementia. What does the nurse identify as the cause of this problem?

Select health team members -Clients may not select the members of the health care team when admitted to an inpatient setting that delivers care 24 hours a day, 7 days a week.

The nurse is explaining the Client Bill of Rights to a female client whose psychiatrist has admitted her to an inpatient facility. Her admission is voluntary. The statement that is not a client right is the right to:

4. Select health team members **Clients may not select the members of the health care team when admitted to an inpatient setting that delivers care 24 hours a day, 7 days a week. The other rights are included in the Client Bill of Rights.

The nurse is explaining the Client Bill of Rights to a female client whose psychiatrist has admitted her to an inpatient facility. Her admission is voluntary. The statement that is not a client right is the right to: 1. Personal mail 2. Refuse treatment 3. Written treatment plans 4. Select health team members

Prominent delusions Auditory hallucinations Prominent delusions are the essential feature of paranoid schizophrenia; delusions are typically persecutory, grandiose, or both, but delusions with other themes, such as jealousy, religiosity, or somatization, also may occur. Auditory hallucinations are a characteristic associated with paranoid schizophrenia; usually they are related to the delusional theme. Mutism is associated more commonly with the subtype of catatonic schizophrenia. Psychomotor retardation and posturing are associated with catatonic schizophrenia. A flat affect is associated more commonly with the subtype of disorganized schizophrenia. Extreme negativism is associated more commonly with the subtype of catatonic schizophrenia.

The nurse manager of a psychiatric unit informs the primary nurse that a client will be admitted to the unit within an hour. The client's admission diagnosis is paranoid schizophrenia. What classic clinical findings should the nurse anticipate? Select all that apply. Mutism Posturing Flat affect Extreme negativism Prominent delusions Auditory hallucinations

1. Attempts to minimize the illness

The nurse should suspect that a client who had a recent myocardial infarction is experiencing denial when the client: 1. Attempts to minimize the illness 2. Lacks an emotional response to the illness 3. Refuses to discuss the condition with the client's spouse 4. Expresses displeasure with the prescribed activity program

"My stomach has disintegrated."

The nurse notes that a client has been experiencing a somatic delusion. Which statement led to this conclusion?

That their problems are not unique -Sharing problems with others who have similar problems can help one explore feelings and begin to enhance coping abilities. The Twelve Steps of AA guide alcoholics to seek help from a higher power, which may be religious, based in nature, or the group itself. Problem drinking usually is caused by how the drinker feels about him- or herself. Although AA is a support group, it is a self-help support group. The only one who can stop someone from drinking is the person who is drinking.

The nurse refers a client to a self-help group. What does the nurse anticipate that a self-help groups such as Alcoholics Anonymous (AA) will help its members learn?

1. That their problems are not unique **Sharing problems with others who have similar problems can help one explore feelings and begin to enhance coping abilities.

The nurse refers a client to a self-help group. What does the nurse anticipate that a self-help groups such as Alcoholics Anonymous (AA) will help its members learn? 1. That their problems are not unique 2. That they do not need a crutch to lean on 3. That their problems are caused by alcohol 4. That the group can stop them from drinking

Orientation phase, when a contract is established

The nurse should first discuss terminating the nurse-client relationship with a client during the:

Attempts to minimize the illness

The nurse should suspect that a client who had a recent myocardial infarction is experiencing denial when the client:

1.Attempts to minimize the illness

The nurse should suspect that a client who had a recent myocardial infarction is experiencing denial when the client: 1.Attempts to minimize the illness 2.Lacks an emotional response to the illness 3.Refuses to discuss the condition with the client's spouse 4.Expresses displeasure with the prescribed activity program

Identify personal feelings about child abusers

To therapeutically relate to parents who are known to have maltreated their child, the nurse must first:

Exhibits lack of empathy for others -Self-motivation and self-satisfaction are of paramount concern to people with antisocial personality disorder, and they have little or no concern for others.

The nursing staff is discussing the best way to develop a relationship with a new client who has antisocial personality disorder. What characteristic of clients with antisocial personality should the nurses consider when planning care?

"It must be frustrating to deal with your child's behavior." -Stating that it must be frustrating acknowledges the parent's distress and encourages verbalization of feelings.

The parent of a child with a tentative diagnosis of attention deficit-hyperactivity disorder (ADHD) arrives at the pediatric clinic insisting on getting a prescription for medication that will control the child's behavior. What is best response by the nurse?

Denial is being used as a defense.

The parents of a toddler with recently diagnosed moderate cognitive impairment discuss their child's possibility of future independent function. What should the nurse conclude?

2. Denial is being used as a defense.

The parents of a toddler with recently diagnosed moderate cognitive impairment discuss their child's possibility of future independent function. What should the nurse conclude? 1. They accept the child's diagnosis. 2. Denial is being used as a defense. 3. They want to explore their child's limitations. 4. Intellectualization helps them put the diagnosis into perspective.

Explore the client's response to the parents' behavior

The parents of a young adult client visit regularly. After one visit the client becomes very agitated. What should the nurse do to relieve the client's distress?

Way to manage overwhelming feelings

The parents of an adolescent who engages in self-injurious cutting behavior ask the nurse why their child self-mutilates. What should the nurse give as the reason for the cutting?

"That hurts a lot when you want to be liked."

The parents of an overweight 12-year-old bring their child to the mental health clinic. One parent says, "You've got to do something to help us—just look how huge he is." The child tells the nurse, "I hate school. The other kids tease me about my weight. I'm always last when they pick teams in gym." What is the most therapeutic response by the nurse?

7.5 mL

The practitioner prescribes valproic acid (Depakene) 750 mg daily to be administered in two divided doses. The medication is supplied as a syrup of 250 mg/5 mL. How many milliliters of solution should the nurse administer per dose? Record your answer using one decimal place. __________ mL

2. Basic personality

The way individuals cope with an unexpected hospitalization depends on many factors. However, the one that is most significant is: 1. Cognitive age 2. Basic personality 3. Financial resources 4. General physical health

Benztropine (Cogentin), 2 mg intramuscularly -Benztropine (Cogentin) is an anticholinergic, antiparkinsonian drug used to treat drug-induced extrapyramidal symptoms associated with phenothiazine therapy; the intramuscular (IM) route will relieve symptoms more rapidly.

Thirty minutes after administering fluphenazine (Prolixin) to a client, the nurse notes that the client's jaw is rigid, the client is drooling, and her speech is slurred. There are a number of as-needed prescriptions in the client's chart. What should the nurse administer?

4.Benztropine (Cogentin), 2 mg intramuscularly7

Thirty minutes after administering fluphenazine (Prolixin) to a client, the nurse notes that the client's jaw is rigid, the client is drooling, and her speech is slurred. There are a number of as-needed prescriptions in the client's chart. What should the nurse administer? 1.Diazepam (Valium), 10 mg by mouth 2.Trihexyphenidyl (Artane), 1 mg by mouth 3.Haloperidol (Haldol), 2 mg intramuscularly 4.Benztropine (Cogentin), 2 mg intramuscularly7

4. Benztropine (Cogentin), 2 mg intramuscularly7 **Benztropine (Cogentin) is an anticholinergic, antiparkinsonian drug used to treat drug-induced extrapyramidal symptoms associated with phenothiazine therapy; the intramuscular (IM) route will relieve symptoms more rapidly.

Thirty minutes after administering fluphenazine (Prolixin) to a client, the nurse notes that the client's jaw is rigid, the client is drooling, and her speech is slurred. There are a number of as-needed prescriptions in the client's chart. What should the nurse administer? 1. Diazepam (Valium), 10 mg by mouth 2. Trihexyphenidyl (Artane), 1 mg by mouth 3. Haloperidol (Haldol), 2 mg intramuscularly 4. Benztropine (Cogentin), 2 mg intramuscularly7

Repression

Three days after a stressful incident a client can no longer remember why it was stressful. The nurse, in relating to this client, can be most therapeutic by identifying that the inability to recall the situation is an example of the defense mechanism of:

3.Repression

Three days after a stressful incident a client can no longer remember why it was stressful. The nurse, in relating to this client, can be most therapeutic by identifying that the inability to recall the situation is an example of the defense mechanism of: 1.Denial 2.Regression 3.Repression 4.Dissociation

Interpersonal theory

To provide appropriate psychosocial support to clients, a nurse must understand development across the life span. What theory is the nurse using in considering relationships and resulting behaviors the central factors that influence development?

Interpersonal theory -The interpersonal theory of human development by Harry Stack Sullivan highlights interpersonal behaviors and relationships as the central factors influencing child and adolescent development across six "eras"; the need to satisfy social attachments and a longing to meet biological and psychological needs are two dimensions associated with this theory.

To provide appropriate psychosocial support to clients, a nurse must understand development across the life span. What theory is the nurse using in considering relationships and resulting behaviors the central factors that influence development?

A nurse notes that a client in the detoxification unit is exhibiting early signs of alcohol withdrawal. What clinical manifestations might the nurse have noticed? (Select all that apply.)

Tremors Anorexia

What characteristic uniquely associated with psychophysiological disorders differentiates them from somatoform disorders?

Underlying pathophysiology

What is the prognosis for a normal productive life for a child with autism?

Unlikely because of interference with so many parameters of function

A 6-year-old child who has autism exhibits frequent spinning and hand-flapping behaviors. What should the nurse teach the parents to do to limit these actions?

Use another activity to distract the child

A nurse is in the process of developing a therapeutic relationship with a client who has an addiction problem. What client communication permits the nurse to conclude that they are making progress in the working stage of the relationship? (Select all that apply.)

Verbalizes difficulty identifying personal strengths Acknowledges the effects of the addiction on the family Addresses how the addiction has contributed to family distress

What behavior by a client with a long history of alcohol abuse is an indication that the client may be ready for treatment?

Verbalizing an honest desire for help

A client on a psychiatric unit misses breakfast because of an elaborate hand-washing ritual. What is the most important therapeutic intervention during the early period of the client's hospitalization?

Waking the client early so the ritual can be completed before breakfast

While watching television in the dayroom a client who has demonstrated withdrawn, regressed behavior suddenly screams, bursts into tears, and runs from the room to the far end of the hallway. What is the most therapeutic intervention by the nurse?

Walking to the end of the hallway where the client is standing

Amnesia, apraxia, agnosia, aphasia -Neurofibrillary tangles in the hippocampus cause recent memory loss (amnesia); temporoparietal deterioration causes cognitive deficiencies in speech (aphasia), purposeful movements (apraxia), and comprehension of visual, auditory, and other sensations (agnosia).

What are the "four A's" for which nurses should evaluate clients with suspected Alzheimer disease?

Develop a sense of satisfaction when considering past achievements

What can the nurse do to help older adult to successfully complete Erikson's major task of this stage?

1. Empathy **Empathy—understanding and to some extent sharing the emotions of another—encourages the expression of feelings. Empathy is an essential tool in caring for emotionally ill clients.

What characteristic is most essential for the nurse caring for a client undergoing mental health care? 1. Empathy 2. Sympathy 3. Organization 4. Authoritarianism

Sets limits -Having poor control, these individuals cannot set limits for themselves and require an environment in which appropriate limits for behavior are set for them.

What characteristic of an environment should the nurse consider important for a confused older adult with socially aggressive behavior?

Based on realistic limits -Realistic guidelines reduce anxiety, increase feelings of security, and increase adherence to the therapeutic regimen.

What characteristic of the environment is most therapeutic for clients with the diagnosis of bulimia nervosa?

Underlying pathophysiology -The psychophysiological response (e.g., hyperfunction or hypofunction) produces actual tissue change. Somatoform disorders are unrelated to organic changes. There is an emotional component in both instances. There is a feeling of illness in both instances. There may be a restriction of activities in both instances.

What characteristic uniquely associated with psychophysiological disorders differentiates them from somatoform disorders?

Anger

What client response should the nurse anticipate when an attempt is made to prevent a client from carrying out ritualistic behavior?

Autonomy -Testing the self both physically and psychologically occurs during the toddler stage, after trust has been achieved. Trust is the task of infancy. The task of industry is accomplished between the ages of 6 and 12.

What developmental task should the nurse consider when caring for toddlers?

3. Autonomy

What developmental task should the nurse consider when caring for toddlers? 1. Trust 2. Industry 3. Autonomy 4. Identification

Sitting down next to the client at frequent intervals

What is a therapeutic nursing action in the care of a depressed client?

2. Taking responsibility for the hostile behavior

What is an initial client objective in relation to anger management? 1. Expressing remorse over aggressive actions 2. Taking responsibility for the hostile behavior 3. Developing alternative methods to release feelings 4. Teaching others how to avoid triggering the angry behavior

Using a firm but caring and consistent approach

What is essential for the nurse to do when approaching a client during a period of overactivity?

Helping the couple express to each other their feelings about his terminal illness

What is most important for the nurse to do to assist a couple to cope with their feelings about the husband's terminal illness?

Self -The self is often the most important tool available to the nurse to help a client cope; to be therapeutic, the nurse must be present, actively listening, and attentive.

What is the basic therapeutic tool used by the nurse to foster a client's psychological coping?

1.Self

What is the basic therapeutic tool used by the nurse to foster a client's psychological coping? 1.Self 2.Milieu 3.Helping process 4.Client's intellect

1. Self

What is the basic therapeutic tool used by the nurse to foster a client's psychological coping? 1. Self 2. Milieu 3. Helping process 4. Client's intellect

Supporting rituals while setting realistic limits

What is the best nursing intervention during the working phase of the therapeutic relationship with which to meet the needs of individuals who demonstrate obsessive-compulsive behavior?

Focusing on nonthreatening subjects -Nursing care involves a steady attempt to draw the client into some response. This can best be accomplished by focusing on nonthreatening subjects that do not demand a specific response

What is the best nursing intervention to encourage a socially withdrawn client to talk?

Recognizing that the behavior is part of the illness but setting limits on it

What is the best nursing intervention when the language of a client in the manic phase of a bipolar disorder becomes vulgar and profane?

Developing an awareness of self and the professional role in the relationship -The nurse's major tool in mental health nursing is the therapeutic use of self. Mental health nurses must learn to identify their own feelings and understand how they affect the situation. Although remaining therapeutic and professional and being able to understand and accept a client's behavior may be difficult, an awareness of self is still the most difficult part of developing a nurse-client relationship. Accepting responsibility for identifying and evaluating the real needs of a client implies that the nurse is working alone in caring for the client.

What is the most difficult initial task in the development of a nurse-client relationship?

3. Developing an awareness of self and the professional role in the relationship **The nurse's major tool in mental health nursing is the therapeutic use of self. Mental health nurses must learn to identify their own feelings and understand how they affect the situation.

What is the most difficult initial task in the development of a nurse-client relationship? 1. Remaining therapeutic and professional 2. Being able to understand and accept a client's behavior 3. Developing an awareness of self and the professional role in the relationship 4. Accepting responsibility for identifying and evaluating the real needs of a client

4.Follow the prescribed medication regimen This is important because side effects and denial of illness may cause clients to stop taking their medications; this is a common cause of relapse.

What is the most important information a nurse should teach to prevent relapse in a client with a psychiatric illness? 1.Develop close support systems 2.Create a stress-free environment 3.Refrain from activities that cause anxiety 4.Follow the prescribed medication regimen

4. The need to follow the prescribed medication regimen

What is the most important information for a nurse to teach to prevent relapse in a client with a psychiatric illness? 1. The need to develop a close support system 2. The need to create a stress-free environment 3. The need to refrain from activities that cause anxiety 4. The need to follow the prescribed medication regimen

The self and a desire to help

What is the most important tool a nurse brings to the therapeutic nurse-client relationship?

3. Assisting the individual with plans for the future **Helping an individual maintain an interest in the future is therapeutic. It is forward looking and fosters a positive attitude.

What is the most therapeutic nursing intervention to help a late-middle-aged individual cope with the emotional aspects of aging? 1. Focusing on the individual's past experiences 2. Having the individual attend lectures on aging 3. Assisting the individual with plans for the future 4. Encouraging the individual to focus on his or her career

To compete with opioids for receptors that control respiration

What is the planned effect of naloxone when it is administered for a heroin overdose?

Seizure activity -Stimulants increase the excitatory neurotransmitters (e.g., adrenaline and dopamine), lowering the seizure threshold.

What is the primary concern for a nurse caring for a client who is grossly impaired by stimulants?

Alcoholism involves the entire family. -Research indicates that alcoholism is a family disease, with its roots in the family of origin

What is the primary reason that the nurse encourages the family of an alcoholic to become involved in the treatment program?

Restoring the client's psychological equilibrium

What is the priority goal in the planning of care for a client in crisis?

3.Restoring the client's psychological equilibrium

What is the priority goal in the planning of care for a client in crisis? 1.Referring the client for occupational therapy 2.Arranging follow-up counseling for the client 3.Restoring the client's psychological equilibrium 4.Having the client work to gain insight into the problem

Confabulating

What should a nurse conclude that a client is doing when he makes up stories to fill in blank spaces of memory?

Clients do not want to repeat their rituals but feel compelled to do so.

What should a nurse consider when planning care for a client who is using ritualistic behavior?

Help the client feel accepted by the staff on the unit

What should a nurse do when caring for a client whose behavior is characterized by pathological suspicion?

1. Realistic limits are set.

What should a nurse ensure when creating an environment that is conducive to psychological safety? 1. Realistic limits are set. 2. Passive acceptance is promoted. 3. The client's physical needs are met. 4. The physical environment is kept in order

Channeling unacceptable impulses into socially approved behavior

What should a nurse recognize that a client who uses the defense mechanism of sublimation is doing?

3. Channeling unacceptable impulses into socially approved behavior

What should a nurse recognize that a client who uses the defense mechanism of sublimation is doing? 1. Acting out in reverse something already done or thought 2. Returning to an earlier, less mature stage of development 3. Channeling unacceptable impulses into socially approved behavior 4. Excluding from consciousness thoughts that are psychologically disturbing

It is important to include the family in the treatment plan.

What should nurses consider when working with depressed young children?

Foster a trusting relationship

What should the nurse do to achieve a primary objective of providing a therapeutic daycare environment for a client who is withdrawn and reclusive?

Offer support and encourage safety during play activities

What should the nurse do to develop a trusting relationship with a disturbed child who acts out?

3. Offer support and encourage safety during play activities

What should the nurse do to develop a trusting relationship with a disturbed child who acts out? 1. Ask the child's feelings about the parents 2. Implement one-on-one interactions every half hour 3. Offer support and encourage safety during play activities 4. Begin setting limits and explain the rules that must be followed

Compensation -Compensation is replacing a weak area or trait with a more desirable one

When a person who is nonathletic and uncoordinated is successful in a musical career, it may be related to the defense mechanism of:

React to the feeling tone of the client's delusion. Reacting to the feeling tone of the client's delusion helps the client explore underlying feelings and allows the client to see the message that his verbalizations are communicating. Expressing disbelief about the client's delusion denies the client's feelings rather than accepting and working with them. Attempting to divert the client rather than accepting and working with him denies the client's feelings. Responding to the verbal content of the client's delusion focuses on the delusion itself rather than on the feeling that is causing the delusion.

What should the nurse do when a client with the diagnosis of schizophrenia talks about being controlled by others? Express disbelief about the client's delusion. Divert the client's attention to unit activities. React to the feeling tone of the client's delusion. Respond to the verbal content of the client's delusion

3. React to the feeling tone of the client's delusion

What should the nurse do when a client with the diagnosis of schizophrenia talks about being controlled by others? 1. Express disbelief about the client's delusion 2. Divert the client's attention to unit activities 3. React to the feeling tone of the client's delusion 4. Respond to the verbal content of the client's delusion

Set limits The client's security is increased by the setting of limits; guidelines remove responsibility for behavior from the client and increase compliance with the regimen.

What should the nurse do when interacting with an adolescent client with the diagnosis of anorexia nervosa?

Biological perspective -The biological factors, including genetics, neuroanatomy, and abnormal neurotransmitter-endocrine interactions, prevail as the origin of schizophrenia as a result of studies conducted during the twentieth century.

What should the nurse identify as the foremost basis for the development of schizophrenia?

"I'm not the least bit worried." -Not being worried indicates potential denial and possible failure to address the problem emotionally.

What statement by a 45-year-old woman scheduled for an abdominal hysterectomy and bilateral salpingo-oophorectomy should alert the nurse to the potential for postoperative coping difficulties?

1."I'm not the least bit worried."

What statement by a 45-year-old woman scheduled for an abdominal hysterectomy and bilateral salpingo-oophorectomy should alert the nurse to the potential for postoperative coping difficulties? 1."I'm not the least bit worried." 2."I don't want any more children." 3."I get along very well with my husband." 4."I'll be glad not to have any more periods."

1. "I'm not the least bit worried." **Not being worried indicates potential denial and possible failure to address the problem emotionally

What statement by a 45-year-old woman scheduled for an abdominal hysterectomy and bilateral salpingo-oophorectomy should alert the nurse to the potential for postoperative coping difficulties? 1. "I'm not the least bit worried." 2. "I don't want any more children." 3. "I get along very well with my husband." 4. "I'll be glad not to have any more periods."

Electroconvulsive therapy

What treatment should a nurse anticipate will be prescribed for a client with severe, persistent, intractable depression and suicidal ideation?

Telling the client that these words cannot be understood

When a disturbed client who has a history of using neologisms says to the nurse, "My lacket huss kelong mon," the nurse should respond by:

2. Listen to what the client is saying

When a newly admitted client with paranoid ideation tells the nurse about people coming through the doors to commit murder, the nurse should: 1. Ignore the client's stories 2. Listen to what the client is saying 3. Explain that no one can get through the door 4. Ask for an explanation of where the information was obtained

Saying, "I'll be back in a few minutes so we can talk." -Saying, "I'll be back in a few minutes so we can talk," allows the angry client time to regain self-control; announcing a plan to return will ease fears of abandonment or retribution.

When a nurse enters a room to administer an oral medication to an agitated and angry client with schizophrenia, paranoid type, the client shouts, "Get out of here!" What is the most therapeutic response?

"I want to talk with you because you are important to me." -The response "I want to talk with you because you are important to me" is an expression of the nurse's positive thoughts about the client and lets the client know that the nurse is concerned.

When a nurse sits next to a depressed client and begins to talk, the client responds, "I'm stupid and useless. Talk with the other people who are more important." Which response is most therapeutic?

May become exhausted from excessive activity

When caring for clients who are demonstrating manic behavior, the nurse must constantly reevaluate these clients' physical needs. What characteristic about these clients makes this particularly important?

In what situation should a nurse anticipate that a client will experience a phobic reaction?

When coming into contact with the feared object

Productive

When considering Erikson's psychosocial developmental tasks, a nurse should focus care for middle-aged adults around their need to be:

1. Planning for future safety 3. Validating the experiences 4 .Promoting access to community services

When intimate partner violence (IPV) is suspected, the nurse plays an important role as an advocate for the victim. The advocate role includes several important components. (Select all that apply.) 1. Planning for future safety 2. Normalizing victimization 3. Validating the experiences 4 .Promoting access to community services

Capacity to handle life's stresses

When planning care for an older client, the nurse remembers that aging has little effect on a client's:

1. Denial 2. Anger 3. Bargaining 4. Depression 5. Acceptance

When planning nursing care for clients who are grieving the potential death of a family member, it is helpful to draw on the understanding of the five stages of grieving identified and described by Elisabeth Kübler-Ross. Place these stages in order of progression from first to last.

Selective serotonin reuptake inhibitors -Selective serotonin reuptake inhibitors have better safety profiles and do not carry the risk of substance abuse and tolerance. Anticholinergics are administered concurrently with antipsychotics to minimize extrapyramidal side effects. Lithium carbonate is a drug used to treat bipolar disorder. Antipsychotics are administered to clients with thought disorders.

When reviewing the medications for a group of clients on a psychiatric unit, the nurse concludes that the pharmacotherapy for anxiety disorders is moving away from benzodiazepines and moving toward:

1 Projection 5 Rationalization

When talking with a client who has alcoholism, the nurse notes that the client becomes irritable, makes excuses, and blames family and friends for the drinking problem. Which defense mechanisms does the nurse conclude that the client is using? (Select all that apply.)

1. Projection 5. Rationalization **Projection is the unconscious denial of unacceptable feelings and emotions in one's self while attributing them to others. This defense mechanism commonly is used by clients with alcoholism because it helps make reality more acceptable. Rationalization is making acceptable excuses for behavior; this defense is used by people with alcoholism because it makes reality more acceptable.

When talking with a client who has alcoholism, the nurse notes that the client becomes irritable, makes excuses, and blames family and friends for the drinking problem. Which defense mechanisms does the nurse conclude that the client is using? (Select all that apply.) 1. Projection 2. Suppression 3. Sublimation 4. Identification 5. Rationalization

"I've been on the medication for 8 days now, and I don't feel any better."

When talking with a client who has been receiving Paroxetine (Paxil), the nurse determines that more clarification is needed when the client says:

The client is trying to divide the staff, and the behavior should be reported to the other staff members

When talking with one of the day nurses, a client with the diagnosis of anorexia nervosa states that the day nurses give better care and are nicer than the night nurses. The client also asks a question that the day nurse knows was already answered by one of the night nurses. What conclusion should the nurse make about the client?

Project involving drawing -An art-type project that may be worked on successfully at one's own pace is appropriate for a depressed client. Board games and card games with three other clients require too much concentration and may increase the client's feelings of despair.

Which activity is most appropriate for a nurse to introduce to a depressed client during the early part of hospitalization?

1.Active 5.Goal-directed 1. The nurse should assume an active role in assessing the current situation and facilitate the interview with authority. 5. During crisis intervention the nurse should be goal-directed to assist the client with coping with the crisis

Which approaches should a nurse use during crisis intervention? Select all that apply. 1.Active 2.Passive 3.Reflective 4.Interpretative 5.Goal-directed

Suspicious feelings

Which client characteristic is an initial concern for the nurse when caring for a client with the diagnosis of paranoid schizophrenia?

Suspicious feelings The nurse must consider the client's suspicious feelings and establish basic trust to promote a therapeutic milieu. Continual pacing is not a problem because the nurse can walk back and forth with the client. Inability to socialize with others and disturbed relationship with the family may be of long-range importance but have little influence on the nurse-client relationship at this time.

Which client characteristic is an initial concern for the nurse when caring for a client with the diagnosis of paranoid schizophrenia? Continual pacing Suspicious feelings Inability to socialize with others Disturbed relationship with the family

Interacting appropriately with others in the therapeutic milieu -Interacting appropriately with others in the therapeutic milieu is a goal related to the identified problem and is appropriate and measurable.

Which goal specific to a client with impaired verbal communication related to a psychological barrier should be documented in the client's clinical record?

Toddler -The toddler is learning autonomy, but because of the nature of development there is still physical and emotional dependence on the parents. The major task during infancy is the development of trust. School-aged children cope with the task of industry and developing skills for working in and relating to the world. Preschool-aged children cope with developing a sense of initiative.

Which individual is coping with issues concerning dependence versus independence?

2.Toddler

Which individual is coping with issues concerning dependence versus independence? 1.Infant 2.Toddler 3.School-aged child 4.Preschool-aged child

1.Milieu therapy

Which nursing intervention involves providing, structuring, and maintaining a safe and therapeutic environment in collaboration with patients, families, and other health care providers? 1.Milieu therapy 2.Coordination of care 3.Health teaching and health maintenance 4.Pharmacologic, biologic, and integrative therapies

1. Milieu therapy **Milieu therapy involves providing, structuring, and maintaining a safe and therapeutic environment in collaboration with psychiatric patients, families, and other health care providers.

Which nursing intervention involves providing, structuring, and maintaining a safe and therapeutic environment in collaboration with patients, families, and other health care providers? 1. Milieu therapy 2. Coordination of care 3. Health teaching and health maintenance 4. Pharmacologic, biologic, and integrative therapies

Providing clear boundaries and consequences

Which nursing intervention is most important for a client who has the diagnosis of antisocial personality disorder?

Psychoanalytical model -The psychoanalytical model studies the unconscious and uses the strategies of hypnosis, dream interpretation, and free association to encourage the release of repressed feelings. The behaviorist model holds that the self and mental symptoms are learned behaviors that persist because they are consciously rewarding to the individual; this model deals with behaviors on a conscious level of awareness. The psychobiological model views emotional and behavioral disturbances as stemming from a physical disease; abnormal behavior is directly attributed to a disease process. This model deals with behaviors on a conscious level of awareness. The social-interpersonal model affirms that crucial social processes are involved in the development and resolution of disturbed behavior; this model deals with behavior on a conscious level of awareness.

Which psychotherapeutic theory uses hypnosis, dream interpretation, and free association as methods to release repressed feelings?

2. Psychoanalytical model

Which psychotherapeutic theory uses hypnosis, dream interpretation, and free association as methods to release repressed feelings? 1. Behaviorist model 2. Psychoanalytical model 3. Psychobiological model 4. Social-interpersonal model

"I need to take every dose of my medication as prescribed."

Which statement by the client indicates to the nurse that the teaching about taking an antidepressant medication has been understood?

Nursing Interventions Classification (NIC) -Nursing Interventions Classification (NIC) is a tool that helps to define nursing interventions, as well as helps to standardize and measure the nursing care provided.

Which tool is used to standardize and measure nursing treatments?

3. Nursing Interventions Classification (NIC) **Nursing Interventions Classification (NIC) is a tool that helps to define nursing interventions, as well as helps to standardize and measure the nursing care provided.

Which tool is used to standardize and measure nursing treatments? 1. Nursing Outcomes Classification (NOC) 2. NANDA-I-Approved Nursing Diagnoses 3. Nursing Interventions Classification (NIC) 4. Diagnostic and Statistical Manual of Mental Disorders (DSM-5)

Neologism -Neologism is the invention of new words with meanings understood only by the client.

While caring for a client, a nurse notes that the client has begun to create new words. What term does the nurse use to document this finding?

"I'd like to understand what you're saying, but I'm having difficulty following you."

While speaking with a client with schizophrenia, the nurse notes that the client keeps interjecting sentences that have nothing to do with the main thoughts being expressed. The client asks whether the nurse understands. What is the best response by the nurse?

Walking to the end of the hallway where the client is standing -Walking to the end of the hallway where the client is standing lets the client know that the nurse is available. It also demonstrates an acceptance of the client.

While watching television in the dayroom a client who has demonstrated withdrawn, regressed behavior suddenly screams, bursts into tears, and runs from the room to the far end of the hallway. What is the most therapeutic intervention by the nurse?

2. Are better equipped to assist the client

With the client's permission, the nurse should inform the family about what is happening. The main reason for this action is that informed families: 1. Ease the client's anxiety 2. Are better equipped to assist the client 3. Appear more relaxed with the situation 4. Commonly cause fewer nursing problems

Apologize and leave the room

Without knocking, a nurse enters the room of a young male client with the diagnosis of panic disorder and finds him masturbating. What should the nurse do?

What is the greatest difficulty for nurses caring for the severely depressed client?

quality of depression


Kaugnay na mga set ng pag-aaral

Republican, Democratic, Third & Green Parties Review

View Set

Project Management Chapter3 Key Terms

View Set

Communication 30 Final Study Guide

View Set

Clinical Application of the Oxyhemoglobin Dissociation Curve

View Set

New Business Venture - (Quiz4-Final)

View Set

chapter 4 ( fill-in the blanks / Explanations )

View Set

Chapter 3 Test Review Part 2-Spanish

View Set